teoria dos números - uepa

204

Upload: alex-pereira-bezerra

Post on 29-Jul-2015

379 views

Category:

Documents


6 download

TRANSCRIPT

Marília Brasil Xavier

REITORA

Prof. M. Sc. Rubens Vilhena Fonseca

COORDENADOR GERAL DOS CURSOS DE MATEMÁTICA

MATERIAL DIDÁTICO

COLABORAÇÃO

Maria da Glória Costa Lima

Cleyton Isamu Muto

EDITORAÇÃO ELETRONICA

Odivaldo Teixeira Lopes

ARTE FINAL DA CAPA

Odivaldo Teixeira Lopes

REALIZAÇÃO

BELÉM – PARÁ – BRASIL

- 2011 -

SUMÁRIO Capítulo 1:...............................................................................................................................................9

NÚMEROS INTEIROS – NOÇÕES FUNDAMENTAIS ...................................................................................9 1.1 – NÚMEROS INTEIROS ....................................................................................................................................... 10 1.2 – PROPRIEDADES DOS INTEIROS ........................................................................................................................ 11 1.3 – VALOR ABSOLUTO DE UM INTEIRO ................................................................................................................ 12 1.4 – REPRESENTAÇÃO DOS INTEIROS EM OUTRAS BASES ...................................................................................... 14 1.5 – FATORIAL E PRINCÍPIO FUNDAMENTAL DA CONTAGEM .................................................................................. 15 1.6 – PRINCÍPIO FUNDAMENTAL DA CONTAGEM - PFC ............................................................................................ 16 1.7 – NÚMERO BINOMIAL ........................................................................................................................................ 17 1.8 – NÚMEROS BINOMIAIS COMPLEMENTARES ...................................................................................................... 18 1.9 – NÚMEROS BINOMIAIS CONSECUTIVOS ............................................................................................................ 18 1. 10 – PISO, TETO E NINT DE UM NÚMERO REAL. ................................................................................................ 20 1.11 – O PRINCÍPIO DA CASA DOS POMBOS (PRINCÍPIO DAS GAVETAS DE DIRICHLET) .............................................. 24 1.12 – CAOS FATORIAL: !N. ..................................................................................................................................... 25 1.13 – LEFT FATORIAL: L!N ..................................................................................................................................... 26 EXERCÍCIOS ............................................................................................................................................................ 27

Capítulo 2:.............................................................................................................................................30

INDUÇÃO MATEMÁTICA ........................................................................................................................30 2.1 – ELEMENTO MÍNIMO DE UM CONJUNTO DE INTEIROS ...................................................................................... 30 2.2 – PRINCÍPIO DA BOA ORDENAÇÃO .................................................................................................................... 31 2.3 – PRINCÍPIO DE INDUÇÃO FINITA. ...................................................................................................................... 32 2.4 – INDUÇÃO MATEMÁTICA ................................................................................................................................ 33 2.5. EXEMPLOS DE DEMONSTRAÇÃO POR INDUÇÃO MATEMÁTICA .......................................................................... 35 2.6 . OUTRAS FORMAS DA INDUÇÃO MATEMÁTICA ................................................................................................ 37 EXERCÍCIOS ............................................................................................................................................................ 42

Capítulo 3:.............................................................................................................................................43

SOMATÓRIOS E PRODUTÓRIOS .............................................................................................................43 3.1 . SOMATÓRIOS ................................................................................................................................................. 43 3.2. PROPRIEDADES DOS SOMATÓRIOS ................................................................................................................... 44 3.3. PRODUTÓRIOS ................................................................................................................................................. 45 3.4. PROPRIEDADES DOS PRODUTÓRIOS ................................................................................................................. 46

Capítulo 4 ..............................................................................................................................................48

DIVISIBILIDADE .....................................................................................................................................48 4.1. RELAÇÃO DE DIVISIBILIDADE EM Z ................................................................................................................. 48 4.2. CONJUNTO DOS DIVISORES DE UM INTEIRO ..................................................................................................... 50 4.3. DIVISORES COMUNS DE DOIS INTEIROS ........................................................................................................... 50 4.4. TEOREMA DA DIVISÃO ................................................................................................................................... 51 4.5. PARIDADE DE UM INTEIRO .............................................................................................................................. 54 EXERCÍCIOS ............................................................................................................................................................ 56

Capítulo 5 ..............................................................................................................................................58

MÁXIMO DIVISOR COMUM ...................................................................................................................58 5.1. MÁXIMO DIVISOR COMUM DE DOIS INTEIROS .................................................................................................. 58 5.2. EXISTÊNCIA E UNICIDADE DO MDC. ................................................................................................................ 59 5.3. INTEIROS RELATIVAMENTE PRIMOS (COPRIMOS OU PRIMOS ENTRE SI) ............................................................ 61 5.4. CARACTERIZAÇÃO DO MDC DE DOIS INTEIROS ................................................................................................ 64 5.5. MDC DE VÁRIOS INTEIROS .............................................................................................................................. 64 EXERCÍCIOS ............................................................................................................................................................ 65

Capítulo 6 ............................................................................................................................................. 67

ALGORITMO DE EUCLIDES – MÍNIMO MÚLTIPLO COMUM ................................................................. 67 6.1. ALGORITMO DE EUCLIDES .............................................................................................................................. 67 6.2 . MÚLTIPLOS COMUNS DE DOIS INTEIROS ......................................................................................................... 74 6.3. MÍNIMO MÚLTIPLO COMUM DE DOIS INTEIROS ................................................................................................ 75 6.5. MMC DE VÁRIOS INTEIROS .............................................................................................................................. 76 EXERCÍCIOS ............................................................................................................................................................ 78

Capítulo 7 ............................................................................................................................................. 79

NÚMEROS PRIMOS ................................................................................................................................ 79 7.1. INTRODUÇÃO .................................................................................................................................................. 79 7.2. NÚMEROS PRIMOS (DO LAT. PRIMUS, PRINCIPAL. PRIME EM INGLÊS) .............................................................. 81 7. 3. TEOREMA FUNDAMENTAL DA ARITMÉTICA. ................................................................................................... 82 7.4. A SEQÜÊNCIA DOS NÚMEROS PRIMOS .............................................................................................................. 84 7.5. O CRIVO DE ERATÓSTENES. .............................................................................................................................. 86 7.6. SEQÜÊNCIA DE INTEIROS CONSECUTIVOS COMPOSTOS .................................................................................... 94 7.7 . CONJECTURAS ................................................................................................................................................ 96 7.8. FÓRMULAS QUE GERAM ALGUNS NÚMEROS PRIMOS ........................................................................................ 98 7.9. DECOMPOSIÇÃO DO FATORIAL EM FATORES PRIMOS ..................................................................................... 101 7.10. MÉTODO DA FATORAÇÃO DE FERMAT .......................................................................................................... 105 7. 11 – ALGORITMO DE FERMAT ............................................................................................................................ 105 EXERCÍCIOS .......................................................................................................................................................... 107

Capítulo 8: .......................................................................................................................................... 110

EQUAÇÕES DIOFANTINAS LINEARES ................................................................................................. 110 3.1. GENERALIDADES ........................................................................................................................................... 111 3.2. CONDIÇÃO DE EXISTÊNCIA DE SOLUÇÃO ........................................................................................................ 112 3.3. SOLUÇÕES DA EQUAÇÃO AX + BY = C. ........................................................................................................... 113 EXERCÍCIOS .......................................................................................................................................................... 115

Capítulo 9 ........................................................................................................................................... 117

CONGRUÊNCIAS .................................................................................................................................. 117 9.1. CONGRUÊNCIAS ............................................................................................................................................ 117 9.2. CARACTERIZAÇÃO DE INTEIROS CONGRUENTES ............................................................................................ 117 9.3. PROPRIEDADES DAS CONGRUÊNCIAS ............................................................................................................. 118 9.4. SISTEMAS COMPLETOS DE RESTOS ................................................................................................................ 121 9.5 – ARITMÉTICA MÓDULO M .............................................................................................................................. 122

9.6. ADIÇÃO E MULTIPLICAÇÃO EM m .............................................................................................................. 124

9.7. SUBTRAÇÃO EM m ...................................................................................................................................... 130

9.8. DIVISÃO EM m ............................................................................................................................................ 131

9.9. POTENCIAÇÃO EM m .................................................................................................................................. 135

EXERCÍCIOS .......................................................................................................................................................... 139

Capítulo 10 ......................................................................................................................................... 141

TEOREMAS DE FERMAT, WILSON E EULER .......................................................................... 141 10.1. PEQUENO TEOREMA DE FERMAT ....................................................................................................... 141 EXERCÍCIOS .......................................................................................................................................................... 145 10.2. TEOREMA DE WILSON .................................................................................................................................. 146 EXERCÍCIOS ............................................................................................................................................................................. 149

10.3. TEOREMA DE EULER .................................................................................................................................... 150 10.4. FUNÇÃO TOTIENT (N) ................................................................................................................................. 151

10.5 – CÁLCULO DE (N) ...................................................................................................................................... 152 10.6. RESOLUÇÃO DE CONGRUÊNCIAS LINEARES PELO TEOREMA DE EULER ......................................................... 155

10. 7. RESOLUÇÃO DA EQUAÇÃO (N). ................................................................................................................. 156

10.8 – VALÊNCIA DA FUNÇÃO TOTIENTE: ( )N m . ............................................................................................ 159

EXERCÍCIOS .......................................................................................................................................................... 160 10.9. TEOREMA CHINÊS DO RESTO (TCR) .............................................................................................................. 161 10.10. POTENCIAÇÃO: UMA APLICAÇÃO DO TEOREMA DE EULER ......................................................................... 165 10.11 – POTENCIAÇÃO: UMA APLICAÇÃO DO TEOREMA CHINÊS DO RESTO (TCR) .................................................. 165

Capítulo 11 ..........................................................................................................................................171

CIFRA DE CÉSAR ..................................................................................................................................171 11.1. FUNÇÕES POLINOMIAIS DE CODIFICAÇÃO .................................................................................................... 174

Capítulo 12 ..........................................................................................................................................179

CIFRA DE VIGENÈRE ...........................................................................................................................179

Capítulo 13 ..........................................................................................................................................182

CIFRA DE HILL.....................................................................................................................................182

Capítulo 14 ..........................................................................................................................................190

RSA .......................................................................................................................................................190 14. 1. PRÉ-CODIFICAÇÃO ...................................................................................................................................... 190 14.2 – CODIFICANDO E DECODIFICANDO ............................................................................................................... 191 14. 3. ASSINATURA DIGITAL UTILIZANDO A CRIPTOGRAFIA RSA .......................................................................... 195

Capítulo 15 ..........................................................................................................................................201

PARTILHA DE SENHAS .........................................................................................................................201

CAPÍTULO 1

NÚMEROS INTEIROS – NOÇÕES FUNDAMENTAIS

9

Capítulo 1:

NÚMEROS INTEIROS – NOÇÕES

FUNDAMENTAIS

INTRODUÇÃO

Teoria dos Números nasceu cerca de 600 anos antes de Cristo quando Pitágoras e

os seus discípulos começaram a estudar as propriedades dos números inteiros. Os

pitagóricos rendiam verdadeiro culto místico ao conceito de número,

considerando-o como essência das coisas. Acreditavam que tudo no universo estava

relacionado com números inteiros ou razões de números inteiros (em linguagem atual,

números racionais). Aliás, na antiguidade a designação número aplicava-se só aos inteiros

maiores do que um.

http://nonio.fc.ul.pt/analise1/cap1/hnum.htm

O conceito de número tomou forma num longo desenvolvimento histórico. A origem e

formulação deste conceito ocorreu simultaneamente com o despontar, entenda-se nascimento,

e desenvolvimento da Matemática. As atividades práticas do homem, por um lado, e as

exigências internas da Matemática por outro determinaram o desenvolvimento do conceito de

número. A necessidade de contar objetos levou ao aparecimento do conceito de número

Natural.

Todas as nações que desenvolveram formas de escrita introduziram o conceito de

número Natural e desenvolveram um sistema de contagem. O desenvolvimento subsequente

do conceito de número prosseguiu principalmente devido ao próprio desenvolvimento da

Matemática. Os números negativos aparecem pela primeira vez na China antiga. Os chineses

estavam acostumados a calcular com duas coleções de barras - vermelha para os números

positivos e preta para os números negativos.No entanto, não aceitavam a idéia de um número

negativo poder ser solução de uma equação. Os Matemáticos indianos descobriram os

números negativos quando tentavam formular um algoritmo para a resolução de equações

quadráticas. São exemplo disso as contribuições de Bramaghupta, pois a aritmética

sistematizada dos números negativos encontra-se pela primeira vez na sua obra. As regras

sobre grandezas eram já conhecidas através dos teoremas gregos sobre subtração, como por

exemplo (a - b)(c - d) = ac + bd - ad - bc, mas os hindus converteram-nas

em regras numéricas sobre números negativos e positivos.

Diofanto (Séc. III) operou facilmente com os números negativos. Eles apareciam

constantemente em cálculos intermédios em muitos problemas do seu "Aritmetika", no

entanto havia certos problemas para o qual as soluções eram valores inteiros negativos como

por exemplo:

4x + 20 = 4 ou 3x – 18 = 5x2

Nestas situações Diofanto limitava-se a classificar o problema de absurdo. Nos séculos XVI e

XVII, muitos matemáticos europeus não apreciavam os números negativos e, se esses

A

CAPÍTULO 1

NÚMEROS INTEIROS – NOÇÕES FUNDAMENTAIS

10

números apareciam nos seus cálculos, eles consideravam-nos falsos ou impossíveis. Exemplo

deste fato seria Michael Stifel (1487- 1567) que se recusou a admitir números negativos como

raízes de uma equação, chamando-lhes de "numeri absurdi". Cardano usou os números

negativos embora chamando-os de "numeri ficti". A situação mudou a partir do (Séc.XVIII)

quando foi descoberta uma interpretação geométrica dos números positivos e negativos como

sendo segmentos de direções opostas.

http://www.somatematica.com.br/historia.php

1.1 – Números Inteiros

Os números inteiros ou apenas os inteiros são:

cujo conjunto representa-se pela letra Z, isto é:

Neste conjunto Z destacam-se os seguintes subconjuntos:

1) Conjunto Z* dos inteiros não nulos ( 0 ):

2) Conjunto Z dos inteiros não negativos ( 0 ):

3) Conjunto Z dos inteiros não positivos ( 0 ):

4) Conjunto *Z dos inteiros positivos (> 0):

5) Conjunto *Z dos inteiros negativos (< 0):

Os inteiros positivos são também denominados inteiros naturais e por isso o conjunto

dos inteiros positivos é habitualmente designado pela letra N (N = *Z ).

*Z {x Z| x 0} = {-1, -2, -3,...}

*Z {x Z| x 0} = {1, 2, 3,...}

Z {x Z| x 0} = {0, -1, -2, -3,...}

Z {x Z| x 0} = {0, 1, 2, 3,...}

Z* = {x Z| x 0} { 1, 2, 3,...}

Z = {..., -3, -2, -1, 0, 1, 2, 3,...}

..., -3, -2, -1, 0, 1, 2, 3,...

CAPÍTULO 1

NÚMEROS INTEIROS – NOÇÕES FUNDAMENTAIS

11

1.2 – Propriedades dos Inteiros

O conjunto Z dos inteiros munido das operações de adição (+) e multiplicação ( . )

possui as propriedades fundamentais que a seguir enumeramos, onde a, b e c são inteiros

quaisquer, isto é, elementos de Z:

1) a + b = b + a e ab = ba;

2) (a + b) + c = a + (b + c) e (ab) c = a (bc);

3) 0 + a = a e 1.a = a;

4) –a = (-1) a e a – a = a + (-a) = 0;

5) a (b + c) = ab + ac;

6) 0.a = 0, e se ab = 0, então a = 0 ou b = 0.

Também existe uma “relação de ordem” entre os inteiros, representada pelo sinal “<

(menor que)”, que possui as seguintes propriedades:

7) Se a 0 , então a > 0 ou a < 0;

8) Se a < b e b < c, então a < c;

9) Se a < b, então a + c < b + c;

10) Se a < b e 0 < c, então ac < bc;

11) Se a < b e c < 0, então bc < ac.

Destas propriedades podem ser deduzidas muitas outras propriedades dos inteiros.

Exemplo 1.1: Demonstrar: -(a + b) = (-a) + (-b).

Com efeito, temos sucessivamente:

-(a + b) = (-1) (a + b) = (Propriedade 4)

= (-1) a + (-1) b = (Propriedade 5)

= (-a) + (-b) (Propriedade 4)

Exemplo 1.2: Demonstrar que , se 0x , então 20 x .

Com efeito:

1) Se 0x , então 0x ou 0 x (Propriedade 7)

2) Se 0x , então 0. .x x x (Propriedade 11)

20 x (Propriedade 6)

3) Se 0 x , então 0. .x x x (Propriedade 10)

20 x (Propriedade 6)

CAPÍTULO 1

NÚMEROS INTEIROS – NOÇÕES FUNDAMENTAIS

12

Nota: Com o mesmo significado de a < b, escreve-se b > a. Indica-se, de modo

abreviado, que a < b ou a = b por a b . Por exemplo, temos 2 3 , porque 2

< 3, e 2 2 , porque 2 - 2.

Com o mesmo significado de a b , escreve-se b a . Em lugar de a b e

b c também se escreve a b c .

1.3 – Valor absoluto de um Inteiro

Definição 1.1: Chama-se valor absoluto de um inteiro a, o inteiro que se indica por | a |, e tal

que:

Assim, por exemplo:

Consoante a definição de | a |, para todo inteiro a, temos:

O valor absoluto | a | de um inteiro a também pode ser definido pelas igualdades:

onde a² denota a raiz quadrada não negativa de a² e máx [-a, a] indica o maior dos dois

inteiros –a e a.

Assim, por exemplo:

Teorema 1.1: Se a e b são dois inteiros, então:

| ab | | a | . | b |

| 4 | ( 4)² 16 4

| 6 | = máx [-6, 6] = 6

| a | a² , | a | = máx [-a, a]

| a | 0 , | a | ² a² , | a | | a | , a | a |

| 3| 3 e | 5| ( 5) 5

a, se a 0| a |

a, se a < 0

CAPÍTULO 1

NÚMEROS INTEIROS – NOÇÕES FUNDAMENTAIS

13

Demonstração:

Com efeito:

Teorema 1.2: Se a e b são dois inteiros, então:

Demonstração:

Com efeito, pela definição de | a |, temos:

Somando ordenadamente estas desigualdades, obtemos:

o que implica:

* Usou –se o fato de que x a a x a .

Corolário 1.1: Se a e b são dois inteiros, então:

Demonstração:

Com efeito:

| a b | | a ( b) | | a | | b | | a | | b |

| a b | | a | | b |

| a b | | a | | b |

(| a | | b |) a b | a | | b | *

| a | a | a | , | b | b | b |

| a b | | a | | b |

| ab | (ab)² a²b² a². b² | a | . | b |

CAPÍTULO 1

NÚMEROS INTEIROS – NOÇÕES FUNDAMENTAIS

14

1.4 – Representação dos Inteiros em outras Bases

Teorema 1.3: Dado um inteiro qualquer b 2, todo inteiro positivo n admite uma única

representação da forma:

onde os ai são tais que 0 ai < b , i = 0, 1, ... , m

Demonstração:

Assim, dado um inteiro qualquer 2b , todo inteiro positivo n pode ser representado por um

polinômio inteiro em b do grau m (porque 0ma ), ordenado segundo as potencias

decrescentes de b, e cujos coeficientes ia são inteiros que satisfaçam as condições:

Este polinômio representa-se, de modo abreviado, pela notação:

em que os coeficientes ia são indicados pela ordem respectiva, figurando o inteiro b como um

índice.

O inteiro b chama-se base e é costume dizer que n está escrito no sistema de base b.

Exemplos:

a) Escrever 105 no sistema binário

105 = 1.26 + 1.2

5 + 0.2

4 + 1.2

3 + 0.2

2 + 0.2 + 1 = (1101001)2

Por outro lado, (100111)2 = 1.25 + 0.2

4 + 0.2

3 + 1.2

2 + 1.2 + 1 = 39

b) Escrever 31415 no sistema de base 8

Temos, sucessivamente:

31415 = 8.3926 + 7

3926 = 8.490 + 6

490 = 8.61 + 2

61 = 8.7 + 5

7 = 8.0 + 7

Portanto 31415 = 7.84 + 5.8

3 + 2.8

2 + 6.8 + 7 = (75267)8

1 2 1 0( )m m bn a a a a a

0 ( 0,1,2, , )ia b i m , sendo 0ma

1 2

1 2 1 0

m m

m mn a b a b a b a b a

CAPÍTULO 1

NÚMEROS INTEIROS – NOÇÕES FUNDAMENTAIS

15

c) Escrever (3531)6 no sistema de base 10

Temos, (3531)6 = 3.63 + 5.6

2 + 3.6 + 1 = 847

d) Escrever (6165)7 no sistema de base 12

Temos, (6165)7 = 6.73 + 1.7

2 + 6.7 + 5 = 2154

Vamos escrever 2154 (base 10) na base 12:

2154 = 12.179 + 6

179 = 12.14 + 11

14 = 12.1 + 2

1 = 12.0 + 1

No sistema de base 12 é hábito designar 10 e 11 por a e b, respectivamente, de modo que os

algarismos deste sistema são: 0, 1, 2, 3, 4, 5, 6, 7, 8, 9, a, b. Portanto,

2154 = 1.123 + 2.12

2 + b.12 + 6 = (12b6)12

Assim, no sistema de numeração decimal, dado um inteiro n, temos que,

é a representação no sistema decimal do inteiro positivo n.

Podemos também dizer que todo inteiro positivo n pode ser expresso sob a forma:

Onde a0 é o algarismo das unidades de n

1.5 – Fatorial e Princípio Fundamental da Contagem

Foi a necessidade de calcular o número de possibilidades existentes nos chamados jogos de

azar que levou ao desenvolvimento da Análise Combinatória, parte da Matemática que estuda

os métodos de contagem. Esses estudos foram iniciados já no século XVI, pelo matemático

italiano Niccollo Fontana (1500-1557), conhecido como Tartaglia. Depois vieram os

franceses Pierre de Fermat (1601-1665) e Blaise Pascal (1623-1662).

A Análise Combinatória visa desenvolver métodos que permitam contar - de uma forma

indireta - o número de elementos de um conjunto, estando esses elementos agrupados sob

certas condições.

Definição 1.2: Chama-se fatorial de um inteiro não negativo n ( n 0 ), o inteiro que se indica

por n!, e tal que:

Assim, por exemplo:

1, se n = 0 ou n = 1n!

n(n 1)(n 2)...3.2.1 se n 2

n = 10k + a0

n = am. 10m

+ am-1. 10m-1

+ ... + a1. 10 + a0, 0 ≤ ak ≤ 10

CAPÍTULO 1

NÚMEROS INTEIROS – NOÇÕES FUNDAMENTAIS

16

7! = 7.6.5.4.3.2.1 = 5040

Observe-se que n! = n.(n-1)!.

Exemplo 1.4: Escrever, usando o símbolo de fatorial, o produto dos n primeiros inteiros

positivos pares e o produto dos n primeiros inteiros positivos ímpares.

Os n primeiros inteiros positivos pares são:

2,4,6, ..., 2n – 2, 2n

Isto é:

2.1,2.2,2.3, ..., 2 . (n – 1), 2n

Portanto:

2,4,6, ..., 2n – 2, 2n = 2n (1.2.3... (n -1).n) = 2

n . n!

Os n primeiros inteiros positivos ímpares são:

1,3,5, ..., 2n – 3, 2n - 1

Portanto:

Exemplo 1.5: Calcular a soma:

1.1! + 2.2 ! + 3.3! + ... + n.n!

Tomemos a igualdade:

k.k! = (k + 1)! – k!

e nela façamos sucessivamente k = 1, 2, 3,..., n, o que dá:

Somando ordenadamente todas essas n igualdades e simplificando, obtemos:

1.6 – Princípio fundamental da contagem - PFC

1.1! + 2.2! + 3.3! +...+ n.n! = (n + 1)! – 1

1.1! = 2! – 1

2.2! = 3! – 2!

3.3! = 4! – 3!

n.n! = (n + 1)! – n!

1.2.3.4...(2 2).(2 1).2 (2 )!1.3.5...(2 3).(2 1)

2.4.6...(2 2).2 2 . !n

n n n nn n

n n n

CAPÍTULO 1

NÚMEROS INTEIROS – NOÇÕES FUNDAMENTAIS

17

Se determinado acontecimento ocorre em n etapas diferentes, e se a primeira etapa pode

ocorrer de k1 maneiras diferentes, a segunda de k2 maneiras diferentes, e assim

sucessivamente , então o número total T de maneiras de ocorrer o acontecimento é dado por:

T = k1. k2 . k3 . ... . kn

1.7 – Número Binomial

Definição 1.3: Sejam n > 0 e k dois inteiros tais que 0 k n . Chama-se número binomial

de numerador n e classe k, o inteiro que se indica por n

k, e tal que:

Obviamente, também podemos escrever:

Em particular, para k = 0 ou k = n, temos:

Assim, por exemplo:

8 8! 8.7.6.5.4.3.2.1 8.7.656

3 3!5! 3.2.1.5.4.3.2.1 3.2.1

7 7.6.5 7.6.535

4 (7 4)! 3.2.1

n n1

0 n

n n(n 1)...(k 1) n(n 1)...(n k 1)

k (n k)! k!

n n!

k k!(n k)!

CAPÍTULO 1

NÚMEROS INTEIROS – NOÇÕES FUNDAMENTAIS

18

1.8 – Números Binomiais Complementares

Definição 1.4: Chamam-se números binomiais complementares dois números binomiais que

têm o mesmo numerador e cuja soma das suas classes respectivas é igual ao numerador

comum.

Assim, por exemplo, 20

7 e

20

13 são números binomiais complementares, pois, têm o

mesmo numerador 20 e 7 + 13 = 20.

Teorema 1.4: Dois números binomiais complementares são iguais.

Demonstração:

Sejam n

k e

n

h dois números binomiais complementares. Então, k + h = n e k = n – h.

Portanto:

1.9 – Números Binomiais Consecutivos

Definição 1.5: Chamam-se números binomiais consecutivos dois números binomiais que têm

o mesmo numerador e cujas classes respectivas são inteiros consecutivos.

Assim, por exemplo, 18

9 e

18

10 são números binomiais consecutivos, pois, têm o mesmo

numerador 18 e as suas classes respectivas são os inteiros consecutivos 9 e 10.

Teorema 1.5: Entre dois números binomiais consecutivos n

k 1 e

n

k, com 1 k n ,

subsiste a relação de Stifel:

n n n 1

k 1 k k

n n nn! n!

k n h h(n h)!(n (n h))! (n h)!h!

CAPÍTULO 1

NÚMEROS INTEIROS – NOÇÕES FUNDAMENTAIS

19

Demonstração: Com efeito:

Assim, por exemplo:

Corolário 1.2: n n 1 n 2 k k 1

...k k 1 k 1 k 1 k 1

Demonstração:

Com efeito, mudando na relação de Stifel n sucessivamente por n – 1, n – 2, n – 3,..., k,

obtemos:

n n 1 n 1

k k 1 k

n 1 n 2 n 2

k k 1 k

n 2 n 3 n 3

k k 1 k

...........................................

n 1 k k

k k 1 k

18 18 19

9 10 10

13 12 12

8 8 7

n n n! n!

k 1 k (k 1)!(n k 1)! k!(n k)!

n! n!

(k 1)!(n k 1)(n k)! k(k 1)!(n k)!

n! 1 1

(k 1)!(n k)! n k 1 k

n! n 1

(k 1)!(n k)! k(n k 1)

n 1(n 1)!

kk!(n 1 k)!

CAPÍTULO 1

NÚMEROS INTEIROS – NOÇÕES FUNDAMENTAIS

20

Além disso, é evidente: k k 1

k k 1.

Somando ordenadamente todas essas igualdades e suprimindo os termos comuns aos dois

membros acha-se a relação desejada.

Substituindo, nesta relação, cada número binomial pelo seu complementar, obtemos:

Corolário 1.3: n n 1 n 2 n k n k 1

...k k k 1 1 0

Demonstração: Consoante a relação de Stifel, temos:

Além disso, temos:

Somando ordenadamente todas essas igualdades e suprimindo os termos comuns aos dois

membros acha-se a relação desejada.

1. 10 – Piso, Teto e Nint de um número real.

É fácil perceber que qualquer número real está entre dois números inteiros, um inteiro menor

que o dado número real e um inteiro maior que esse número real. Por exemplo, o número real

5 , está entre os inteiros 2 e 3 ( 2 5 3 ); o número real 3

2, está entre os inteiros -5 e -

4 (3

5 42

), etc.. Veremos a seguir que o inteiro à esquerda será chamado de Piso

(floor) e o inteiro à direita será chamado de Teto(ceiling).

n k n k 1

0 0

n n 1 n 1

k k 1 k

n 1 n 2 n 2

k 1 k 2 k 1

n 2 n 3 n 3

k 2 k 3 k 2

...........................................

n k 1 n k n k

1 0 1

n n 1 n 2 k k 1...

n k n k n k 1 1 0

CAPÍTULO 1

NÚMEROS INTEIROS – NOÇÕES FUNDAMENTAIS

21

Definição 1.6: Chamam-se partes inteiras de um número real r, os inteiros n e n+1 que

verificam às condições:

A todo número real r podemos associar dois números inteiros chamados piso e teto. Keneth

Iverson introduziu esses nomes, assim como a notação que será usada, no início da década de

1960.

Definição 1.7: Chama-se piso de um número real r, ao maior número inteiro menor ou igual a r.

Definição 1.8: Chama-se teto de um número real r, ao menor número inteiro maior ou igual a r.

Definição 1.9: Chama-se nint de um número real r, o valor inteiro mais próximo de r. Para

evitar ambigüidades, no caso de valores de r iguais à metade de um inteiro, convenciona-se

arredondar o valor de nint sempre para o inteiro par.

Notação: Usaremos as seguintes notações:

Assim, o piso e o teto de um número real r são os inteiros definidos pelas desigualdades:

Em linguagem da Teoria dos Conjuntos:

Observe que r r r se, e somente se, r é um número inteiro, e que todo número real r

pode ser escrito sob a forma:

r r k , onde 0 1k r r

e

1r r k , onde 0 1 1k r r

O número real k chama-se parte não-inteira de r.

max{ | } e min{ | }r n n r r n n r

1 1r r r r r

r = piso de r

r = teto de r

r = nint de r

1n r n

CAPÍTULO 1

NÚMEROS INTEIROS – NOÇÕES FUNDAMENTAIS

22

Exemplos: piso e teto de r.

a) 2 1 e 2 2 d) 1 1

0 e 13 3

b) 3 e 4 e) 1 1

1 e 02 2

c) 3 3

2 e 12 2

f) 7 7 e 7 7

Exemplos: nint de r.

a) [2,3] = 2 e [2,7] = 3 d) [3,5] = 4 e [4,5] = 4

b) 1 23

0 e 43 6

e) 1

0 e 1,5 22

c) [ ] = 3 e [e] = 3 f) [-3, 4] = -3 e [-3, 7] = -4

CAPÍTULO 1

NÚMEROS INTEIROS – NOÇÕES FUNDAMENTAIS

23

Abaixo, estão ilustrados os gráficos das funções piso, teto e nint, respectivamente.

( )f x x

( )f x x

CAPÍTULO 1

NÚMEROS INTEIROS – NOÇÕES FUNDAMENTAIS

24

( )f x x

1.11 – O Princípio da Casa dos Pombos (Princípio das Gavetas de Dirichlet)

O princípio da Casa dos Pombos é a afirmação de que se n pombos devem ser postos em m

casas, sendo n > m então pelo menos uma casa irá conter mais de um pombo.

É também conhecido como Princípio das Gavetas de Dirichlet, acredita-se que o primeiro

relato deste principio foi feito pôr Dirichlet em 1834, com o nome de Schubfachprinzip

("Princípio das Gavetas").

O princípio da casa do pombo é um exemplo de um argumento de calcular que pode ser

aplicado em muitos problemas formais, incluíndo aqueles que envolvem um conjunto infinito.

Exemplo: Quantas pessoas são necessárias para se ter certeza que haverá pelo menos duas

delas façam aniversário no mesmo mês?

Resposta: 13 pessoas. Pelo princípio da casa dos pombos se houver mais pessoas (13) do que

meses (12) é certo que pelos menos duas pessoas terão nascido no mesmo mês.

Embora o princípio da casa dos pombos seja uma observação trivial, pode ser usado para

demonstrar resultados possivelmente inesperados . Por exemplo, em toda grande cidade,

digamos com mais de 1 milhão de habitantes existem pessoas com o mesmo número de fios de

cabelo. Demonstração: Tipicamente uma pessoa tem cerca de 150 mil fios de cabelo. É

razoavel supor que ninguém tem mais de 1.000.000 de fios de cabelo em sua cabeça. Se há

mais habitantes do que o número máximo de fios de cabelo, necessariamente pelo menos duas

pessoas terão exatamente o mesmo número de fios de cabelo.

CAPÍTULO 1

NÚMEROS INTEIROS – NOÇÕES FUNDAMENTAIS

25

1.12 – Caos Fatorial: !n.

Suponha que queremos calcular todos os anagramas da palavra ESCOLA, de modo que

nenhuma letra ocupe o seu lugar original, ou primitivo. Um deles seria SEOCAL, uma vez

que nenhuma letra ocupa seu lugar inicial. Esse tipo de permutação é chamada de caótica ou

desordenada e o caos fatorial n ( também chamado de subfatorial ou derangements em

Inglês), simbolizado por !n , é usado para calcular o número dessas permutações caóticas.

Lembre-se que o fatorial calcula o total de permutações de um conjunto.

Definição 1.11.: Chama-se caos fatorial de um inteiro não negativo n ( n 0 ), o inteiro que

se indica por !n, e tal que:

n nn

k 0

1 1 1 1 ( 1) ( 1)!n n! ... n!

0! 1! 2! 3! n! k!

Para 1n , temos: n nn

k 2

1 1 ( 1) ( 1)!n n! ... n!

2! 3! n! k!

Pode-se provar que n!

!ne

.

M. Hassani deu outras formas para o caos fatorial:

n! 1!n , n 1

e

e

1!n e e n! en! ,n 1

Os 10 primeiros valores !n, são:

n n!

0 1

1 0

2 1

3 2

4 9

5 44

6 265

7 1854

8 14833

9 133496

10 1334961

CAPÍTULO 1

NÚMEROS INTEIROS – NOÇÕES FUNDAMENTAIS

26

Voltando ao problema comentando no início, podemos afirmar que o número de permutações

caóticas da palavra ESCOLA é !6 = 265.

Exemplos:

a) 1 1 1 1 1 1 1 1 1 1

!6 6! 6!2! 3! 4! 5! 6! 2 6 24 5! 6!

9 1 1 9 1 1

!6 6! 6!24 5! 6! 4! 5! 6!

6.5.4!.9 6.5! 6!

!6 270 6 14! 5! 6!

!6 265

b) 6! 720

!6 264,87... 265e 2,718...

c) 6! 1 721

!6 265,241... 265e 2,718...

d) 1

!6 2,718... .720 2,718... .720 3,086... .720 2,718... .7202,718...

!6 2221,92 1956,96 2221 1956 265

1.13 – Left Fatorial: L!n

Dura Kurepa , em 1971 publicou a o conceito de L!n, o left factorial, definido como

Um famoso problema em aberto na Teoria dos Números, é uma conjectura feita por Kurepa

de que o MDC (n!, L!n) = 2 para todo n maior que 1.

Abaixo colocamos os 10 primeiros valores do left fatorial. Por definição, L!0 = 0.

n L!n

0 0

1 1

2 2

1

0

! 0! 1! 2! ... ( 1)! !n

k

L n n k

CAPÍTULO 1

NÚMEROS INTEIROS – NOÇÕES FUNDAMENTAIS

27

3 4

4 10

5 34

6 154

7 874

8 5914

9 46234

10 409114

O left fatorial é sempre par para qualquer inteiro maior que 1. Se dividirmos o left fatorial por

2, obtemos alguns valores primos. Veja

n !

2

L n

3 2

4 5

5 17

8 2957

9 23117

10 204557

Uma questão em aberto é saber se existem infinitos primos da forma !

2

L n.

EXERCÍCIOS

1) Sem usar P.A., calcule a soma dos “n”

primeiros inteiros positivos.

2) Calcular o inteiro positivo n, sabendo que

3n+2

. 2n+3

= 2592.

3) Calcule o inteiro positivo n, sabendo-se que:

3n + 3

n+1 + 3

n+2 + 3

n+3 = 1080.

4) Com uma calculadora, achar os valores de n <

10 para os quais n! + 1 é um quadrado perfeito.

5) Sendo m e n inteiros positivos, dizer se é

verdadeiro ou falso:

a) (mn)! = m!. n!

b) (m + n)! = m! + n!

6) Demonstrar: (n – 1)! [(n + 1)! – n!] = (n!)2

7) Sendo n > 2, demonstrar: (n2)! > (n!)

2.

8) Decompor o inteiro 565 numa soma de cinco

inteiros ímpares consecutivos.

9) Achar todas as soluções inteiras e positivas da

equação (x + 1)(y + 2) = 2xy.

10) Determinar todos os inteiros positivos de dois

algarismos que sejam igual ao quádruplo da

soma dos seus algarismos.

11) Achar o menor e o maior inteiro positivo de n

algarismos.

12) Resolva a equação: (x + 2)! = 72.x!

13) Resolver a equação: 2

7 7

2 2xx x

14) Demonstrar : n 1

k 1

nn k

kk

CAPÍTULO 1

NÚMEROS INTEIROS – NOÇÕES FUNDAMENTAIS

28

15) Achar todas as soluções inteiras e positivas da

equação: x2 – y

2 = 88.;

16) Verificar se o quadrado de um inteiro pode

terminar em 2, 3, 7 ou 8.

17) Hilbert escreveu os inteiros de 1 até 1000

(inclusive), em ordem decrescente. Sem usar

P.A, determine qual foi o 0333 inteiro

escrito?

18) Calcular o número de algarismos necessários

para ser escrever os números positivos de 1, 2.

3, 4, ......, n algarismos.

19) O produto de um inteiro positivo de três

algarismos por 7 termina à direita por 638.

Achar esse inteiro.

20) Determinar quantos algarismos se emprega

para numerar todas as páginas de um livro de

2748 páginas.

21) Dois homens estavam conversando num bar

quando um virou para o outro e disse:

- Tenho três filhas a soma de suas idades é

igual ao número da casa em frente e o

produto é 36.

- Posso determinar as idades de suas filhas

apenas com esses dados?

- Não. Dar-lhe-ei um dado fundamental:

minha filha mais velha toca piano.

Determine as idades das filhas e o número da

casa em frente.

22) Calcular a soma dos três maiores números

inteiros de, respectivamente, três, quatro e

cinco algarismos.

23) Determinar a diferença entre o maior número

inteiro com seis algarismos diferentes e o

maior inteiro com cinco algarismos também

diferentes.s

24) Um livro tem 1235 páginas. Determinar o

número de vezes que o algarismo 1 aparece na

numeração da páginas deste livro.

25) Os números abaixo estão dispostos em linhas e

colunas.

1 2

8 9

15 16

22 23

29 30

Determine a posição (linha e coluna) ocupada

pelo número 107.

26) Mostrar que o produto de quatro algarismos

consecutivos, aumentado de 1, é um quadrado

perfeito.

27) A soma dos quadrados de dois inteiros é 3332 e

um deles é o quádruplo do outro. Achar os dois

inteiros.

28) Escrever os inteiros de 1 a 1993, inclusive,

quantas vezes o algarismo 1 é escrito?

29) Determinar o inteiro n > 1 de modo que a soma

1! + 2! + 3! + ... + n! seja um quadrado

perfeito.

30) A média aritmética de dois inteiros positivos é

5 e a média geométrica é 4. Encontre esses

números.

31) Achar cinco inteiros positivos consecutivos

cuja soma dos quadrados é igual a 2010.

32) O resto por falta da raiz quadrada de um inteiro

positivo é 135 e o resto por excesso é 38.

Achar esse inteiro.

33) Resolver a equação ! 3( 2)! 31

! 3( 2)! 29

x x

x x

34) Achar o inteiro que deve ser somado a cada um

dos inteiros 2, 6 e 14 para que, nesta ordem,

formem uma proporção contínua.

35) Coloque em ordem crescente: 602 ;

403 ; 207

.

36) Achar o valor mínimo de uma soma de 10

inteiros positivos distintos, cada um dos quais

se escreve com três algarismos.

37) O menor número natural n, diferente de zero,

que torna o produto de 3888 por n um cubo

perfeito é:

38) Um estudante ao efetuar a multiplicação de

7432 por um certo inteiro achou o produto

1731656, tendo trocado, por engano, o

algarismo das dezenas do multiplicador,

tomando 3 em vez de 8. Achar o verdadeiro

produto.

39) Achar o menor inteiro cujo produto por 21 é

um inteiro formado apenas por 4 algarismo.

CAPÍTULO 1

NÚMEROS INTEIROS – NOÇÕES FUNDAMENTAIS

29

40) Escreve-se a seqüência natural dos inteiros

positivos, sem separar os algarismos:

123456789101112131415...

Determinar:

a) o 435º algarismo

b) o 1756º algarismo.

c) o 12387º algarismo.

41) Escreve-se a seqüência natural dos inteiros

positivos pares, sem separar os algarismos:

24681012141618...

Determinar o 2574º algarismo que se escreve.

42) As representações decimais dos números 19992

e 19995 são escritos lado a lado. O número de

dígitos escritos é igual a:

43) Mostrar que o produto de dois fatores entre 10

e 20 é o décuplo da soma do primeiro com as

unidades do segundo mais o produto das

unidades dos dois.

44) Achar o menor inteiro positivo que

multiplicado por 33 dá um produto cujos

algarismos são todos 7.

45) Os inteiros a e b são tais que 4 < a < 7 e 3 < b

< 4. Mostrar que 0 < a – b < 4.

46) Os inteiros a e b são tais que –1 < a < 3 e –2 <

b < 0. Mostrar que –1 < a – b < 5.

47) Os inteiros a e b são tais que -2 < a < 2 e -

2 < b < 2. Mostrar que –4 < a – b < 4.

48) Em um quartel existem 100 soldados e, todas

as noites, três deles são escolhidos para

trabalhar de sentinela. É possível que após

certo tempo um dos soldados tenha trabalhado

com cada um dos outros exatamente uma vez?

49) Um jogo consiste de 9 botões luminosos (de

cor verde ou vermelha) dispostos da seguinte

forma:

1 2 3

4 5 6

7 8 9

Apertando um botão do bordo do retângulo,

trocam de cor ele e seus vizinhos (do lado ou

em diagonal). Apertando o botão do centro,

trocam de cor todos os seus 8 vizinhos porém

ele não.

Exemplos:

Apertando 1, trocam de cor 1, 2, 4 e 5.

Apertando 2, trocam de cor 1, 2, 3, 4, 5 e 6.

Apertando 5, trocam de cor 1, 2, 3, 4, 6, 7, 8 e 9.

Inicialmente todos os botões estão verdes. É

possível, apertando sucessivamente alguns

botões, torná-los todos vermelhos?

50) Escrevemos abaixo os números naturais de 1 a

10.

1 2 3 4 5 6 7 8 9 10.

Antes de cada um deles, coloque sinais “+” ou

“–” de forma que a soma de todos seja zero.

51) Escrevemos abaixo os números naturais de 1 a

11.

1 2 3 4 5 6 7 8 9 10 11

Antes de cada um deles, coloque sinais “+” ou

“–” de forma que a soma de todos seja zero.

52) Para numerar as páginas de um livro foram

utilizados 663 algarismos. Quantas páginas

tinha o livro?

53) Seja Q = 1! + 2! + 3! + ... + n!. Para quantos

valores de n tem-se Q quadrado perfeito?

54) Quantos são os números naturais de 4 dígitos

que possuem pelo menos dois dígitos iguais?

55) Quantos são os números de 5 algarismos, na

base 10:

a) Nos quais o algarismo 2 figura?

b) Nos quais o algarismo 2 não figura?

56) Permutam-se de todos os modos possíveis os

algarismos 1, 2, 4, 6, 7 e escrevem-se os

números assim formados em ordem crescente.

a) Que lugar ocupa o número 62417?

b) Qual o número que ocupa o 66º lugar?

c) Qual o 200º algarismo escrito?

d) Qual a soma dos números assim

formados?

30

Capítulo 2:

INDUÇÃO MATEMÁTICA

INTRODUÇÃO

s ciências naturais utilizam o método chamado indução empírica para formular leis

que devem reger determinados fenômenos a partir de um grande número de

observações particulares, selecionadas adequadamente. Esse tipo de procedimento,

embora não seja uma demonstração de que um dado fato é logicamente verdadeiro, é

frequentemente satisfatório. Por exemplo: ninguém duvidaria de que quando um corpo é

liberado ao seu próprio peso, no vácuo, na superfície da terra, ele cai segundo a vertical do

local.

A validade de um teorema matemático se estabelece de forma totalmente diferente. Verificar

que uma certa afirmação é verdadeira num grande número de casos particulares não nos

permitirá concluir que ela é válida.

Para demonstrar a verdade de uma sequência infinita de proposições, uma para cada inteiro

positivo, introduziremos o chamado método de recorrência ou indução matemática.

2.1 – Elemento mínimo de um conjunto de inteiros

Definição 2.1: Seja A um conjunto de inteiros. Chama-se elemento mínimo de A um elemento

a A tal que a x para todo x A.

Representa-se pela notação “minA”, que se lê: “mínimo de A”. Portanto, simbolicamente:

Teorema 2.1: Se a é elemento mínimo de A, então esse elemento é único.

Demonstração:

Com efeito, se existisse um outro elemento mínimo b de A, teríamos:

i) a b , porque a = minA.

ii) b a , porque b = minA..

A

minA = a ( a A e ( x A) ( a x ))

CAPÍTULO 2

INDUÇÃO MATEMÁTICA

31

Logo, pela propriedade anti-simétrica da relação de ordem natural “ ” em Z, temos a = b.

O elemento mínimo de A, se existe, denomina-se também primeiro elemento de A ou menor

elemento de A

Exemplo 2.1: O conjunto N = {1, 2, 3,...} dos inteiros positivos tem o elemento mínimo, que

é 1 (minN = 1), porque 1 N e 1 n para todo n N .

Exemplo 2.2: O conjunto A x |x 12 tem o elemento mínimo, que é 13 (minA =

13), porque 13 A e 13 x para todo x A .

Exemplo 2.3: O conjunto 0, 1, 2, 3,... dos inteiros não positivos não tem o elemento

mínimo, porque não existe -Za tal que a x para todo

-Zx .

Exemplo 2.4: O conjunto 2A x |3divide x tem o elemento mínimo 3 (min A = 3),

porque 3 A (3 divide 9) e 3 x para todo x A (1 A e 2 A).

2.2 – Princípio da boa ordenação

Todo conjunto não vazio A de inteiros não negativos possui o elemento mínimo.

Em outros termos, todo subconjunto não vazio A do conjunto

dos inteiros não negativos (+A Z ) possui o elemento mínimo, isto é, simbolicamente:

Exemplo 2.5: O conjunto A = {1, 3, 5, 7,...} dos inteiros positivos ímpares é um subconjunto

não vazio de

Logo, pelo “Princípio da boa ordenação”, A possui o elemento mínimo (minA = 1).

Exemplo 2.6: O conjunto P = {2,3,5,7,11, ...} dos inteiros primos é um subconjunto não vazio

de Z+ ( P Z+). Logo, pelo “Principio da boa ordenação”, P possui o elemento mínimo

(minP = 2).

Teorema 2.2 (de Archimedes): Se a e b são dois inteiros positivos quaisquer, então existe um

inteiro positivo n tal que na b .

Demonstração:

+Z (+A Z ).

( ,A Z A ) min A

+Z ={0 ,1, 2, 3, ...}

CAPÍTULO 2

INDUÇÃO MATEMÁTICA

32

Suponhamos que a e b são dois inteiros positivos para os quais na b para todo inteiro

positivo n. Então, todos os elementos do conjunto:

são inteiros positivos e, pelo “Princípio da boa ordenação”, S possui o elemento mínimo,

digamos minS = b – ka.

E como b – (k + 1)a pertence a S, porque S contém todos os inteiros positivos desta forma,

temos:

isto é, b – ka não é o elemento mínimo de S, o que é uma contradição. Logo, a propriedade

archimediana é verdadeira.

Assim, por exemplo:

i) se a = 2 e b = 11, então n = 6, porque 6.2 > 11;

ii) se a = 9 e b = 5, então n =1, porque 1.9 > 5.

2.3 – Princípio de Indução Finita.

Quando uma proposição é enunciada em termos de números naturais, o Princípio de indução

finita constitui um eficiente instrumento para demonstrar a proposição no caso geral.

Na prática, o método pode ser entendido por um artifício muito simples. Vamos supor que

temos uma série de dominós idênticos colocados em fila, que começa por um deles e

prossegue indefinidamente. Nosso objetivo é - empurrando apenas um dominó - garantir que

todos caiam. Como derrubar todos os dominós? Para isso, basta nos assegurarmos de que:

1) O primeiro dominó cai;

2) Os dominós estão dispostos de tal modo que qualquer um deles - toda vez que cai -,

automaticamente, empurra o dominó seguinte e o faz cair também.

Assim, mesmo que a fila se estenda indefinidamente, podemos afirmar que todos os

dominós cairão.

b – (k + 1) a = (b – ka) – a < b – ka

S = {b – na | n N }

CAPÍTULO 2

INDUÇÃO MATEMÁTICA

33

Vamos estabelecer matematicamente esses procedimentos.

Teorema 2.3 Seja S um subconjunto do conjunto N dos inteiros positivos ( S N ) que

satisfaz as duas seguintes condições:

i) 1 pertence a S (1 S );

ii) para todo inteiro positivo k, se k S , então ( 1)k S .

Nestas condições, S é o conjunto N dos inteiros positivo: S = N.

Demonstração:

Suponhamos, por absurdo, que S não é o conjunto N dos inteiros positivos ( S N ) e seja X o

conjunto de todos os inteiros positivos que não pertencem a S, isto é:

Então, X é um subconjunto não vazio de N ( X N ) e, pelo “Princípio da boa

ordenação”, existe o elemento mínimo 0x de X (minX =

0x ).

Pela primeira condição, 1 S , de modo que 0x > 1 e, portanto,

0x - 1 é um inteiro positivo que

não pertence a X. Logo, (x0 - 1) S e, pela segunda condição, segue-se que (0x - 1) + 1 =

0x

S , o que é uma contradição, pois, 0x X N S , isto é,

0x S . Assim sendo, X e S

= N.

Consoante este “Princípio de indução finita”, o único subconjunto de N que satisfaz às duas

condições é o próprio N.

2.4 – Indução Matemática

Em matemática, conclusões como as que se obtêm a seguir são inadmissíveis. Por quê? Em

que pecam os raciocínios utilizados? Vamos examiná-los...

1) Suponha que desejemos obter uma fórmula que dá o valor da soma Sn = 1 + 3 + 5 + 7 + ...

+ (2n - 1), para qualquer inteiro positivo de n.

É fácil ver que:

n = 1 S1 = 1 = 12;

n = 2 S2 = 1 + 3 = 4 = 22;

n = 3 S3 = 1 + 3 + 5 = 9 = 32

n = 4 S4 = 1 + 3 + 5 + 7 = 16 = 42

X = {x | x N e x S } = N – S

CAPÍTULO 2

INDUÇÃO MATEMÁTICA

34

Por meio de um raciocínio indutivo, os resultados obtidos nos levam a afirmar que para

todo inteiro positivo n tem-se Sn = n2.

2) Consideremos o trinômio P(n) = n2 + n + 41. Considerando n = 0, obtemos P(0) = 41, que

é um número primo. Substituindo n por 1, chegamos a outro número primo, o 43.

Substituindo sucessivamente n por 2, 3, 4, 5, 6, 7, 8, 9 e 10, conseguimos como resultados

outros números primos (47, 53, 61, 71, 83, 97, 113, 131 e 151, respectivamente). Então, os

resultados obtidos nos induzem a afirmar que, para todo n natural, o trinômio P(n) = n2 + n

+ 41, sempre produz como resultado um número primo.

Nos dois exemplos, propôs-se um resultado geral, supostamente válido para todo n, com

base no fato de que ele é correto para alguns valores particulares de n: tal procedimento,

entretanto, pode conduzir a conclusões falsas.

Assim, ainda que em no primeiro caso a proposição geral enunciada resulte correta - por

mero acaso! -, a proposição geral do segundo exemplo é falsa. De fato, P(n) gera números

primos para n= 0, 1, 2, 3, ..., 39, mas para n = 40, ele vale 412, que não é um número

primo. Portanto, no exemplo 2), encontramos uma proposição que - apesar de válida em

40 casos particulares - não é válida em geral.

Note bem: Uma proposição pode ser válida em uma série de casos particulares, mas,

mesmo assim, não o ser de maneira geral.

Coloca-se, então, o seguinte problema: temos uma proposição que se mostrou correta em

muitos casos particulares. No entanto, é impossível verificar todos os casos particulares.

Assim sendo, como podemos saber se a proposição é correta de modo geral? O Teorema

abaixo, esclarece essa questão.

Teorema 2.4: Seja P(n) uma proposição associada a cada inteiro positivo n e que satisfaz às

duas seguintes condições:

i) P(1) é verdadeira;

ii) para todo inteiro positivo k, se P(k) é verdadeira, então P(k + 1) também é verdadeira.

Nestas condições, a proposição P(n) é verdadeira para todo inteiro positivo n.

Demonstração:

Seja S o conjunto de todos os inteiros positivos n para os quais a proposição P(n) é

verdadeira, isto é:

Pela primeira condição, P(1) é verdadeira e, portanto, 1 S . Pela segunda condição, para todo

inteiro positivo k, se k S , então ( 1)k S . Logo, o conjunto S satisfaz às duas condições

do “Princípio de indução finita” e, portanto, S = N, isto é, a proposição P(n) é verdadeira para

todo inteiro positivo n.

S = { n N | P(n) é verdadeira}

CAPÍTULO 2

INDUÇÃO MATEMÁTICA

35

Nota: O teorema 2.4 é geralmente denominado “Teorema da indução matemática” ou

“Princípio de indução matemática”, e a demonstração de uma proposição usando-se este

teorema chama-se “demonstração por indução matemática” ou “demonstração por indução

sobre n”.

Na “demonstração por indução matemática” de uma dada proposição P(n) é obrigatório

verificar que as condições i e ii são ambas satisfeitas. A verificação da condição i é

geralmente muito fácil, mas a verificação da condição ii implica em demonstrar o teorema

auxiliar cuja hipótese é:

denominada “hipótese de indução”, e cuja tese ou conclusão é:

2.5. Exemplos de demonstração por Indução Matemática

Exemplo 2.7: Demonstrar a proposição:

Demonstração:

i) P(1) é verdadeira, visto que 1 = 1².

ii) A hipótese de indução é que a proposição:

é verdadeira.

Adicionando (2k + 1) a ambos os membros desta igualdade, obtemos:

e isto significa que a proposição P(k + 1) é verdadeira.

Logo, pelo “Teorema da indução matemática”, a proposição P(n) é verdadeira para todo

inteiro positivo n.

1 + 3 + 5 + ... + (2k – 1) + (2k + 1) = k² + (2k + 1) = (k + 1)²

P(k): 1 + 3 + 5 + ... + (2k – 1) = k², k N

P(n): 1 + 3 + 5 + ... + (2n – 1) = n², n N

T: proposição P(k + 1) é verdadeira.

H: proposição P(k) é verdadeira, k N .

CAPÍTULO 2

INDUÇÃO MATEMÁTICA

36

Exemplo 2.8: Demonstrar a proposição:

Demonstração:

1) P(1) é verdadeira, visto que 1 1

1.2 1 1

2) A hipótese de indução é que a proposição:

é verdadeira.

Adicionando 1

k 1 k 2 a ambos os membros desta igualdade, obtemos:

e isto significa que a proposição 1P k é verdadeira. Logo, pelo “Teorema da indução

matemática”, a proposição P n é verdadeira para todo inteiro positivo n.

Exemplo 2.9: Demonstrar a proposição:

Demonstração:

1) P (1) é verdadeira, visto que 23| 2 1 .

2) A hipótese de indução é que a proposição: 2kP k :3 | 2 1 ,k N é verdadeira.

Portanto:

22k

– 1 = 3q, com q Z

2( ) :3 | 2 1 ,nP n n N

2

1 1 1 1 1...

1.2 2.3 3.4 k(k 1) k 1 k 2

k 1 k 2k 1 k 1

k 1 k 1 k 2 (k 1) k 2 k 2

1 1 1 1 kP(k) : ... , k N

1.2 2.3 3.4 k(k 1) k 1

1 1 1 1 nP(n) : ... , n N

1.2 2.3 3.4 n(n 1) n 1

CAPÍTULO 2

INDUÇÃO MATEMÁTICA

37

o que implica:

2 2 2k 2k

2k 2k

2 k 1 1 2 .2 1 4.2 1

4.2 4 4 1 4 2 1 3

4.3q 3 3(4q 1)

isto é, a proposição 1P k é verdadeira. Logo, pelo “teorema da indução matemática”, a

proposição P n é verdadeira para todo inteiro positivo n.

Exemplo 2.10: Demonstrar a proposição:

( ) : 2 ,nP n n n N

Demonstração:

1) P(1) é verdadeira, visto que 2¹ = 2 > 1.

2) A hipótese de indução é que a proposição:

P(k): 2k k , k N

é verdadeira. Portanto:

2.2k > 2k ou 2

k+1 > k + k k + 1

o que implica: 12 1k k , isto é, a proposição P(k+1) é verdadeira. Logo, pelo “Teorema da

indução matemática”, a proposição P(n) é verdadeira para todo inteiro positivo n.

2.6 . Outras formas da indução matemática

Teorema 2.5 Seja r um inteiro positivo fixo e seja P(n) uma proposição associada a cada

inteiro n r e que satisfaz às duas seguintes condições:

i) P(r) é verdadeira;

ii) para todo inteiro k r, se P(k) é verdadeira, então P(k + 1) também é verdadeira.

Nestas condições, P(n) é verdadeira para todo inteiro n r..

CAPÍTULO 2

INDUÇÃO MATEMÁTICA

38

Demonstração:

Seja S o conjunto de todos os inteiros positivos n para os quais a proposição P(r + n – 1) é

verdadeira, isto é:

S = {n N | P(r + n – 1) é verdadeira}

Pela primeira condição, P(r) = P(r + 1 – 1) é verdadeira, isto é, 1 S. E, pela segunda

condição, se P(r + k – 1) é verdadeira, então:

P((r + k – 1) + 1) = P(r + (k + 1) – 1)

também é verdadeira, isto é, se k S, então (k + 1) S. Logo, pelo “Princípio da indução

finita”, S é o conjunto dos inteiros positivos: S = N, isto é, a proposição P(r + n – 1) é

verdadeira para todo n N , ou seja, o que é a mesma coisa, a proposição P(n) é verdadeira

para todo inteiro n r .

Exemplo 2.11: Demonstrar a proposição:

P(n): 2 !n n , 4n

Demonstração:

1) P(4) é verdadeira, visto que 42 16 4! 24 .

2) Suponhamos, agora, que é verdadeira a proposição:

Então, por ser

multiplicando termo a termo ( I ) e ( II ):

isto é, a proposição P(k + 1) é verdadeira. Logo, pelo teorema 2.5, a proposição P(n) é

verdadeira para todo inteiro 4n .

Observe-se que a proposição P(n) é falsa para n = 1, 2, 3, pois, temos:

Exemplo 2.12: Demonstrar a proposição:

2¹ > 1! , 2² > 2! , 2³ > 3!

12 !.( 1)k k k ou 12 ( 1)!k k

2 < k + 1 para 4k ( II ),

P(k): 2 !k k , 4k ( I )

CAPÍTULO 2

INDUÇÃO MATEMÁTICA

39

P(n) : n2 > 2n + 1, n 3

Demonstração:

1) P (3) é verdadeira, visto que 32 = 9 > 2. 3 + 1= 7.

2) Suponhamos, agora, que é verdadeira a proposição:

P(k) : k2 > 2k + 1, k 3

Então, temos:

k2 + (2k+ 1) > (2k+1) + (2k+1)

ou

(k +1)2 > 2 (k + 1) + 2k > 2 (k + 1) + 2 > 2 (k +1) + 1, k 3

e, portanto:

(k +1)2 > 2 (k +1) + 1, k 3.

Isto é, a proposição 1P k é verdadeira. Logo, pelo teorema 2.5, a proposição P n é

verdadeira para todo inteiro 3n .

Observa-se que a proposição P n é falsa para n = 1 e n = 2, pois, temos:

12 < 2.1+1 e 2

2 < 2.2 + 1

Teorema 2.6 Seja P(n) uma proposição associada a cada inteiro positivo n e que satisfaz às

duas seguintes condições:

i) P(1) é verdadeira;

ii) para todo inteiro positivo k, se

são todas verdadeiras, então P(k + 1) também é verdadeira.

Nestas condições, a proposição P(n) é verdadeira para todo inteiro positivo n.

P(1), P(2),..., P(k)

CAPÍTULO 2

INDUÇÃO MATEMÁTICA

40

Demonstração:

Seja S o conjunto de todos os inteiros positivos n para os quais a proposição P(n) é

verdadeira, isto é:

Suponhamos por absurdo, que S N e seja X o conjunto de todos os inteiros positivos que

na pertencem a S, isto é:

Então, X é um subconjunto não vazio de N e, pelo “Princípio da boa ordenação”, existe o

elemento mínimo j de X (minX = j).

Pela primeira condição, 1 S , de modo que j > 1, e como j é o menor inteiro positivo que não

pertence a S, segue-se que as proposições P(1), P(2),..., P(j – 1) são todas verdadeiras. Então,

pela segunda condição, a proposição P(j) é verdadeira e j S , o que é uma contradição, pois

j X , isto é, j S . Assim sendo, S = N e a proposição P(n) é verdadeira para todo inteiro

positivo n.

Teorema 2.7 Seja r um inteiro positivo fixo e seja P(n) uma proposição associada a cada

inteiro n r e que satisfaz às duas seguintes condições:

1) P(r) é verdadeira;

2) para todo inteiro k > r, se P(m) é verdadeira para todo inteiro m tal que r m k , então

P(k) é verdadeira.

Nestas condições, a proposição P(n) é verdadeira para todo inteiro n r .

Demonstração:

Seja S o conjunto de todos os inteiros n r para os quais a proposição P(n) é falsa, isto é:

Suponhamos, por absurdo, que S não é vazio ( S ). Então, pelo “Princípio da boa

ordenação”, existe o elemento mínimo j de S (minS = j).

Pela primeira condição, r S , de modo que j > r, e, por conseguinte P(m) é verdadeira para

todo inteiro m tal que r m j . Assim sendo, pela segunda condição, P(j) é verdadeira e

j S , o que é uma contradição, pois, j S . Logo, o conjunto S é vazio ( S ), e a

proposição P(n) é verdadeira para todo inteiro n r .

S = { n N | n r e P(n) é falsa}

X = {x | x N e x S } = N – S

S = { n N | P(n) é verdadeira}

CAPÍTULO 2

INDUÇÃO MATEMÁTICA

41

Nota Histórica

Era ideia assente na comunidade matemática do século XIX, que a indução era obra do

matemático francês Blaise Pascal , tendo em conta diversas demonstrações que apresenta no

seu Traité du Triangle Arithmétique.

Essa situação seria integralmente modificada, vinte anos após a formulação moderna de

indução matemática fixada por Giuseppe Peano , quando Giovanni Vacca , em 1909, num

artigo de três páginas publicado no Bulletin of American Mathematical Society, vem defender

que o italiano Francesco Maurolico , pelos trabalhos que desenvolveu no primeiro livro de

aritmética incluído na sua Opuscula Mathematica, escrita em 1557 e publicado em Veneza no

ano de 1575, como "the first discoverer of the principle of mathematical induction".

O artigo de Vacca encontrou eco, ainda que eventualmente sem verificação posterior, em

autores importantes como Moritz Cantor ou Siegmund Günther . M. Cantor, por exemplo,

que atribuiu inicialmente a Pascal a principal origem do método de indução completa (em

Vorlesungen uber Geschichte der Mathematik, vol. 2, p. 749), viria a transferir esse atributo

para Maurolico (em Zeichrift fur Mathematischen und Naturwissenschaftlichen Unterricht,

vol 33, 1902, p. 536), segundo conta devido a uma informação oral que lhe foi prestada pelo

próprio Vacca.

Passar-se-iam mais de quarenta anos sem que o artigo de Vacca fosse alvo de qualquer crítica.

Até que Hans Freudenthal (em Zur Geschichte der vollständigen Induktion, Archive

Internationale d'Histoire des Sciences 6 (1953) 17-37) depois de um exame detalhado dos

trabalhos de Maurolico, vem sustentar que em apenas três pontos conseguiu reconhecer uma

certa forma de indução matemática: uma forma arcaica, contudo, ao contrário do que

observou em Pascal, onde a indução é formulada pela primeira vez de uma maneira abstrata.

CAPÍTULO 2

INDUÇÃO MATEMÁTICA

42

EXERCÍCIOS

1) Demonstrar por "indução matemática":

a) 12 + 2

2 + 3

2 + ... + n

2 =

6

)1n2)(1n(n

n N

b) 13 + 2

3 + 3

3 + ... + n

3 =

4

)1n(n 22

n

N

c) 12 + 3

2 + 5

2 + ... + (2n – 1)

2 =

3

)1n4(n 2

n N

d) 13 + 3

3 + 5

3 + ... + (2n –1)

3 = n

2(2n

2 – 1)

e) 1.2 + 2.3 + 3.4 + ... + n(n + 1) =

3

)2n)(1n(n

f) 1 1 1

1 1 1 1 ... 1 12 3

nn

, n N

g) a + aq + aq2 + ...+aq

n =

1q

)1q(a 1n

, q 1

2) Demonstrar por "indução matemática"

a) 2n < 2

n+1 n N

b) 2n > n

2 n 5

c) 4n > n

4 n 5

d) 2n > n

3 n 10

e) n ! > n2 n 4

f) n! > n3 n 6

g) 2n

1...

9

1

4

11 2 –

n

1, n N

3) Demonstrar por "indução matemática"

a) 2 | (3n – 1) n N

b) 6 | (n3 – n) n N

c) 5 | (8n – 3

n) n N

d) 24 | (52n

– 1) n

e) 7 | (23n

– 1) n N

f) 8 | 32n

+ 7, n N.

4) Demonstrar que 10n + 1

– 9n – 10 é um múltiplo

de 81 para todo inteiro positivo n

5) Demonstrar que 15

n7

5

n

3

n 53

é um inteiro

positivo para todo n N

6) Prove que, para todo inteiro 1n , o número

4 1

3

n

na é inteiro e ímpar.

7) Para 1n , mostre que 1

!n

n

k

S k é um

inteiro ímpar.

8) Para 0n , mostre que 2 2 111 12n n

na

é um inteiro divisível por 133.

9) Para 3n , mostre que

a) n n 1n 1 n

b) 2 nn! n . .

10) Mostre que é sempre possível pagar, sem

receber troco, qualquer quantia inteira de $,

maior que $7, com notas de $3 e $5.

43

Capítulo 3:

SOMATÓRIOS E PRODUTÓRIOS

3.1 . Somatórios

Sejam os n > 1 inteiros 1 2 na ,a ,...,a . Para indicar, de modo abreviado, a soma

1 2 na a ... a

desses n inteiros usa-se a notação:

que se lê: “somatório de ia de 1 a n”.

Em particular, para n = 2, 3,..., temos:

A letra i chama-se o índice do somatório e pode ser substituída por qualquer outra diferente de

a e de n – é um índice mudo. E os inteiros 1 e n que figuram abaixo e acima da letra grega

maiúscula (sigma) chamam-se respectivamente limite inferior e limite superior do índice i.

O número de parcelas de um somatório é sempre igual à diferença entre os limites superior e

inferior do seu índice mais uma unidade.

Se m e n são dois inteiros, com m n , então, por definição:

Exemplo 3.1: Temos:

7

1

5 5.1 5.2 5.3 5.4 5.5 5.6 5.7

5 10 15 20 25 30 35 140

i

i

n

i m m 1 m 2 n

i m

a a a a ...a

2

i 1 2

i 1

a a a , 3

i 1 2 3

i 1

a a a a , ...

n

i

i 1

a

CAPÍTULO 3

SOMATÓRIOS E PRODUTÓRIOS

44

4

1

8 3 8.1 3 8.2 3 8.3 3 8.4 3

5 13 21 29 68

j

j

83 4 5 6 7 8

3

.2 3.2 4.4 5.2 6.2 7.2 8.2

24 64 160 384 896 2048 3576

k

k

k

Exemplo 3.2: Temos: 6

i

i 1

2 4 8 16 32 64 2

15

1

1 3 5 ... 29 2 1j

j

3.2. Propriedades dos somatórios

Teorema 3.1: n n n

i i i i

i 1 i 1 i 1

(a b ) a b

Demonstração:

Com efeito, desenvolvendo-se o primeiro membro, temos:

Teorema 3.2 n

i 1

a na

Demonstração:

Seja ia a para i = 1, 2,..., n. Então, temos:

Teorema 3.3 n n

i i

i 1 i 1

(a a) a na

Demonstração:

n n

i 1 2 n

i 1 i 1

a a a a ... a a a ... a na

n

i i 1 1 2 2 n n

i 1

(a b ) (a b ) (a b ) ... (a b )

n n

1 2 n 1 2 n i i

i 1 i 1

(a a ... a ) (b b ... b ) a b

CAPÍTULO 3

SOMATÓRIOS E PRODUTÓRIOS

45

Consoante os dois teoremas anteriores, temos:

Teorema 3.4 n n

i i

i 1 i 1

ka k a

Demonstração:

Com efeito, desenvolvendo o primeiro membro, temos:

Exemplo 3.3: Calcular 20

i 1

(5i 2)

Consoante os teoremas anteriores temos, sucessivamente:

3.3. Produtórios

Sejam os n > 1 inteiros 1 2 na ,a ,...,a . Para indicar, de modo abreviado, o produto

1 2 na a ...a

desses n inteiros usa-se a notação:

que se lê: “produtório de ia de 1 a n”.

Em particular, para n = 2, 3,..., temos:

2

i 1 2

i 1

a a a , 3

i 1 2 3

i 1

a a a a , ...

n

i

i 1

a

20 20 20 20

i 1 i 1 i 1 i 1

(5i 2) 5i 2 5 i 20.2 5(1 2 ... 20) 40

15. (1 20)20 40 5.210 40 1090

2

n n

i 1 2 n 1 2 n i

i 1 i 1

ka ka ka ... ka k(a a ... a ) k a

n n n n

i i i

i 1 i 1 i 1 i 1

(a a) a a a na

CAPÍTULO 3

SOMATÓRIOS E PRODUTÓRIOS

46

A letra i chama-se o índice do produtório e pode ser substituída por qualquer outra diferente

de a e de n – é um índice mudo. E os inteiros 1 e n que figuram abaixo e acima da letra grega

maiúscula (pi) chamam-se respectivamente limite inferior e limite superior do índice i.

O número de fatores de um produtório é sempre igual à diferença entre os limites superior e

inferior do seu índice mais uma unidade.

Se m e n são dois inteiros, m n , então, por definição:

Exemplo 3.4: Temos:

Exemplo 3.5: Temos:

3.4. Propriedades dos Produtórios

Teorema 3.5 n n n

i i i i

i 1 i 1 i 1

a b a . b

Demonstração:

Com efeito, desenvolvendo o primeiro membro, temos:

n n n

i i 1 1 2 2 n n 1 2 n 1 2 n i i

i 1 i 1 i 1

a b (a b )(a b )...(a b ) (a a ...a )(b b ...b ) a . b

6i

i 1

3.9.27.81.243.729 3

16

1

1.3.5.7....31 2 1j

j

n

i=1

1.2.3... 1 !=n n n i

6

1

4

1

3 3.1 3.2 3.3 3.4 3.5 3.6

=3.6.9.12.15.18=524880

5 3 5.1 3 5.2 3 5.3 3 5.4 3

=2.7.12.17 2856

i

j

i

j

n

i m m 1 m 2 n

i m

a a .a .a ...a

CAPÍTULO 3

SOMATÓRIOS E PRODUTÓRIOS

47

Teorema 3.6 n

n

i 1

a a

Demonstração:

Seja ia a para i = 1, 2,..., n. Então, temos:

Teorema 3.7 n n

n

i i

i 1 i 1

ka k a

Demonstração:

Com efeito, desenvolvendo o primeiro membro, temos:

Exemplo 3.6: Calcular 4

i 1

(2i 1)²

Consoante o teorema 3.7, temos:

Exemplo 3.7: Demonstrar

n n n

ij ij

i, j 1 i 1 j 1

a a

Com efeito, desenvolvendo o primeiro membro, temos:

n

ij 11 12 1n 21 22 2n n1 n2 nn

i, j 1

a (a a ...a )(a a ...a )...(a a ...a )

n n n n n

1j 2 j nj ij

j 1 j 1 j 1 i 1 j 1

a . a ... a a

4 4

i 1 i 1

(2i 1)² (2i 1) ² (3.5.7.9)² 945² 893025

n nn n

i 1 2 n 1 2 n i

i 1 i 1

ka (ka )(ka )...(ka ) k (a a ...a ) k a

n nn

i 1 2 n

i 1 i 1

a a a a ...a a.a...a a

48

Capítulo 4

DIVISIBILIDADE

Um conceito chave em Teoria dos Números é o conceito de divisibilidade. Existem muitos

aspectos interessantes referentes à divisão de números inteiros. Antes que possam ser

analisados, é necessário que conceitos básicos como divisor e divide estejam bem

estabelecidos.

4.1. RELAÇÃO DE DIVISIBILIDADE EM Z

Definição 4.1: Sejam a e b dois inteiros, com a 0. Diz-se que a divide b se, e somente se,

existe um inteiro q tal que b = aq

Se a divide b também se diz que a é divisor de b, que b é múltiplo de a, que a é um fator de b

ou que b é divisível por a.

Notação: a | b ( a divide b)

Observação: Se a | b , então –a | b

Teorema 4.1: Quaisquer que sejam os inteiros a, b e c tem-se:

1) a | 0 a 0 , 1 | a e a | a a 0

2) Se a | 1 , então a = 1

3) Se a | b e se c | d , então ac | bd

4) Se a | b e se b | c , então a | c

5) Se a | b e se b | a , então a = b

6) Se a | b com b 0 , então | a | | b |

7) Se a | b e se a | c , então a |(bx + cy) para todo x e y em Z

CAPÍTULO 4

DIVISIBILIDADE

49

Demonstração:

Em todas as demonstrações estaremos aplicando a Definição 1 e considerando aceitas todas

as propriedades operatórias dentro do conjunto .

1) De fato:

2) De fato, se a|1, então 1 = a.q, o que implica a = 1 e q = 1 ou a = -1 e q = -1, ou seja:

a = 1.

3) De fato,

Portanto:

4) De fato:

Logo, 1.( . ) |c a q q a c .

5) De fato:

Logo:

6) De fato, nas condições da proprieade, temos:

Como 0q , temos que | | 1q , desse modo temos | | | |b a .

7) De fato:

| .a b b a q

1| .a c c a q

| . , ou seja | | | | . | |a b b a q b a q

1 1 1 1( ) 1 |1 1a a qq qq q q a b

| .a b b a q

1| .b a a b q

| .a b b a q

1| .b c c b q

1.( . ) |bd ac q q ac bd

| .a b b a q

1| .c d d c q

0 = a.0; a = 1.a; a = a.1

CAPÍTULO 4

DIVISIBILIDADE

50

Logo, quaisquer que sejam os inteiros x e y:

Esta propriedade (7) pode ser generalizada; ou seja, se

então, quaisquer que sejam os inteiros

temos:

De acordo com as propriedades (1) e (4), a relação de divisibilidade em é reflexiva e

transitiva, mas não é simétrica.

4.2. Conjunto dos divisores de um inteiro

O conjunto de todos os divisores de um inteiro qualquer a indica-se por

É imediato que, para todo inteiro a, se tem D(a) = D(-a).

Qualquer que seja o inteiro 0a , se |x a , então:

significando que qualquer inteiro 0a tem um número finito de divisores.

4.3. Divisores comuns de dois inteiros

Definição 4.2 Chama-se divisor comum de dois inteiros a e b todo inteiro d 0 tal que

d | a e d | b.

( ) [ | |,| |]a x a D a a a

D(a) = {x Z* | x | a }

1 2| ( ... )na bx bx bx

1 2, ,..., nx x x

| , 1,2,3,...,ka b k n

1 1( ) | ( )bx cy aqx aq y a qx q y a bx cy

CAPÍTULO 4

DIVISIBILIDADE

51

Notação: D(a,b) = { x * | x | a e x | b} ou seja, D(a,b) = D(a) D(b)

Obs.: D(a,b) ; D(0,0) = *

Exemplo 4.1: Sejam os inteiros a = 12 e b = -15. temos:

Portanto:

4.4. Teorema da Divisão

O Teorema da divisão, que veremos a seguir, usado por Euclides no seu livro Elementos,

estabelece uma divisão com resto. É um teorema que foi "provado" uma vez através de um

algoritmo que explica como se processa a divisão, por esse motivo ficou conhecido como

Algoritmo de Euclides.

Teorema 4.2 Se a e b são dois inteiros, com b > 0, então existem e são únicos os inteiros q e r

que satisfazem às condições: a = bq + r e 0 r < b

Demonstração:

Existência

Seja S o conjunto de todos os inteiros não-negativos que são da forma a – bx, com x , isto

é:

Este conjunto S não é vazio, porque, sendo b > 0, temos b 1 e, portanto, para x = - | a |,

resulta:

Assim sendo, pelo “Princípio da boa ordenação”, existe o elemento mínimo r de S tal que

0 r e r = a – bq ou a = bq + r, com q

a – bx = a + b |a | a + | a | 0

S = {a – bx ; x , a – bx 0 }

12, 15 12 15 1, 3D D D

12 1, 2, 3, 4, 6, 12

15 1, 3, 5, 15

D

D

CAPÍTULO 4

DIVISIBILIDADE

52

Além disso, temos r < b, pois, se fosse r b, teríamos:

isto é, r não seria o elemento mínimo de S.

Unicidade

Para demonstrar a unicidade de q e r, suponhamos que existem dois outros inteiros q1 e r1 tais

que

Então, teremos:

por outro lado, temos:

o que implica:

isto é

Assim, b | (r1 – r) e | r1 – r | < b e, portanto: r1 – r = 0, e como b 0, também temos

q – q1 0. Logo, r1 = r e q1 = q.

Nota: Aqui cabe uma pergunta: Por que o resto deve ser positivo? A resposta é

simples: Quando se divide a por b, o que se procura é o maior múltiplo de b que é

menor do que a, de modo que se a =b q + r, então r = a – bq é positivo

porque a.b é menor do que a. Mas, não poderíamos definir divisão de modo que o

resto fosse negativo? Neste caso b.q seria o menor múltiplo de b maior do que a, e

teríamos situações como a do seguinte exemplo: para dividir R$10,00 entre 3

pessoas, cada uma delas receberia 4 reais e haveria um resto de -2(dívida de 2

reais? Quem iria pagar?) O exemplo mostra que esta maneira de se fazer a

divisão não teria muito valor prático. (RPM 8)

Corolário 4.1 Se a e b são dois inteiros com b 0, existem e são únicos os inteiros q e r que

satisfazem às condições: a = bq + r , 0 r < | b |

| r1 – r | < b

- b < r – r1 < b,

- b < - r 0 e 0 r1 < b

bq1 + r1 = bq + r r1 – r = b(q – q1) b | (r1 – r)

a = bq1 + r1 e 0 r1 < b

0 r – b = a – bq – b = a – b( q+1 ) < r

CAPÍTULO 4

DIVISIBILIDADE

53

Demonstração:

Com efeito, se b > 0, nada há que demonstrar, e se b < 0, então | b | > 0, e por conseguinte

existem e são únicos os inteiros q1 e r tais que

ou seja, por ser | b | = - b:

Portanto, existem e são únicos os inteiros q = - q1 e r tais que

Os inteiros a, b, q e r chamam-se respectivamente o dividendo, o divisor, o quociente e o

resto na divisão de a por b.

Nota: As demonstrações acima garantem a validade do Teorema de

Eudoxius: Sejam a e b 0 inteiros, então,

I ) a é um múltiplo de b e, portanto, a= bq, q ;

II ) a está situado entre dois múltiplos consecutivos de b, isto é, existe um

inteiro q tal que, para b > 0, bq a < b(q+1) e, para b < 0, bq a <

b(q-1).

Exemplo 4.2: Achar o quociente q e o resto r na divisão de a = 59 por b = -14 que satisfazem

as condições do algoritmo da divisão.

Efetuamos a divisão usual dos valores absolutos de a e b, obtemos:

o que implica:

Logo, o quociente 4q e o resto 3r .

Exemplo 4.3: Achar o quociente q e o resto r na divisão de a = -79 por b = 11 que satisfazem

as condições do algoritmo da divisão.

Efetuamos a divisão usual dos valores absolutos de a e b, obtemos:

o que implica:

-79 = 11 7 2

79 = 11.7 + 2

59 14 4 3 e 0 3 14

59 14.4 3

a = bq + r e 0 r < | b |.

a = b(- q1 ) + r e 0 r < | b |

a = | b |q1 + r e 0 r < | b |

CAPÍTULO 4

DIVISIBILIDADE

54

Como o termo r = 2 < 0 não satisfaz a condição 0 11r , somando e subtraindo o valor 11

de b ao segundo membro da igualdade anterior, obtemos:

com 0 9 11. Logo, o quociente q = -8 e o r = 9.

Exemplo 4.4: Sejam os inteiros a = 1, -2, 61, -59 e b = -7. Temos:

4.5. Paridade de um Inteiro

Na divisão de um inteiro qualquer a por 2 os possíveis restos são r = 1 e r = 0.

Se r = 0 , então o inteiro a = 2q é denominado par; e se r = 1, então o inteiro a = 2q + 1 é

denominado ímpar, q .

Dois inteiros que são ambos pares ou ambos ímpares dizem-se de mesma paridade, a dois

inteiros tais que um é par e o outro é ímpar, dizemos que tem paridades diferentes.

De modo geral, dado um inteiro 2a , pode-se sempre escrever um inteiro qualquer n, de

modo único, na forma n aq r , onde ,k r e r a .

Teorema 4.3

1) A soma ou a diferença de dois números pares é par.

2) A soma ou a diferença de dois números ímpares é par.

3) A soma ou a diferença de um número par com um número ímpar é ímpar.

Demonstração:

1) Sejam a = 2k1 e b = 2k2, então a b = 2k1 2k2 = 2(k1 k2).

2) Sejam a = 2k1 +1 e b = 2k2 +1, então a b = (2k1 +1) (2k2 +1) = 2(k1 + k2 + 1) ou 2(k1-

k2).

3) Sejam a = 2k1 e b = 2k2 +1, então a b = 2k1 (2k2 +1) = 2(k1 k2) +1.

1 7 .0 1 e 0 1 7 0q e r =1

2 7 .1 5 e 0 5 7 1q e r = 5

61 7 8 5 e 0 5 7 8q e r = 5

59 7 .9 4 e 0 4 7 9q e r = 4

79 11 7 11 11 2 11 8 9

CAPÍTULO 4

DIVISIBILIDADE

55

Exemplo 4.5: Mostrar que o quadrado de qualquer inteiro ímpar é da forma 8k+1.

Com efeito, pelo algoritmo da divisão, qualquer inteiro é de uma das seguintes formas:

Nesta classificação, somente os inteiros das formas 4q +1 e 4q +3 são ímpares e , portanto, os

seus quadrados são da forma:

Assim, por exemplo, 7 e 13 são inteiros ímpares, e temos:

2

2

7 49 8.6 1

13 169 8.21 1

2 2

2 2

4 1 8 2 1 8 1

4 3 8 2 3 1 1 8 1

q q q k

q q q k

4 ,4 1,4 2,4 3q q q q

CAPÍTULO 4

DIVISIBILIDADE

56

EXERCÍCIOS

1) Mostrar que se a | b, então (-a) | b, a | (-b) e

(-a) | (-b).

2) Sejam a, b e c inteiros. Mostrar que:

a ) se a | b, então a | bc.

b ) se a | b e se a | c, então a2 | bc.

c ) a | b se e somente se ac | bc (c 0).

3) Verdadeiro ou falso:

se a | (b + c), então a | b ou a | c.

4) Mostrar que, se a é um número inteiro

qualquer, então um dos inteiros a, a + 2, a + 4 é

divisível por 3.

5) Sendo a um inteiro qualquer, mostrar:

a ) 2 | a(a + 1).

b ) 3 | a(a + 1)(a + 2) .

6) Mostrar que um inteiro qualquer da forma 6k +

5 também é da forma 3t + 2.

7) Mostrar que todo inteiro ímpar é da forma 4k +

1 ou 4k + 3.

8) Mostrar que o quadrado de um inteiro qualquer

é da forma 3k ou 3k + 1.

9) Mostrar que o cubo de um inteiro qualquer é de

uma das formas 9k, 9k + 1 ou 9k + 8.

10) Mostrar que:

a) n(n + 1)(2n + 1)/6 é um inteiro, qualquer

que seja o inteiro positivo n.

b) Se “a “ é um inteiro ímpar, então 24 | a

(a2 – 1).

11) Mostrar que se a | (2x – 3y) e se a | (4x – 5y),

então a | y.

12) Sendo a e b dois inteiros quaisquer, mostrar

que os inteiros a e a + 2b têm sempre a mesma

paridade.

13) Sendo m e n dois inteiros quaisquer, mostrar

que os inteiros m + n e m – n têm sempre a

mesma paridade.

14) Demonstrar que: Se “a” e “b” são inteiros

ímpares, então 8 | a2 – b

2.

15) Determinar os inteiros positivos que divididos

por 17 deixam um resto igual ao quadrado do

quociente.

16) Verdadeiro ou falso: se a | c e se b | c, então a

| b.

17) Mostrar que a diferença entre os cubos de dois

inteiros consecutivos nunca é divisível por 2.

18) Na divisão do inteiro a = 427 por um inteiro

positivo “b”, o quociente é 12 e o resto é r.

Achar o divisor “b” e o resto “r”.

19) Na divisão do inteiro 525 por um inteiro

positivo o resto é 27. Achar os inteiros que

podem ser o divisor e o quociente.

20) Na divisão de dois inteiros positivos o

quociente é 16 e o resto é o maior possível.

Achar os dois inteiros, sabendo-se que sua

soma é 341.

21) Achar os inteiros positivos menores que 150 e

que divididos por 39 deixam um resto igual ao

quociente.

22) Seja d um divisor de n (d | n). Mostrar que cd |

n se e somente se c | (n/d).

23) Sejam n, r e s inteiros tais que 0 < r < n e 0 < s

< n. Mostrar que se n | (r – s) então r = s.

24) Mostrar que o produto de dois inteiros ímpares

é um inteiro ímpar.

25) Demonstrar que se m e n são inteiros ímpares,

então 8 | (m4 + n

4 – 2).

26) Demonstrar que 30 | (n5 – n)

27) Mostrar que, para todo inteiro n, existem

inteiros k e r tais que n = 3k + r e r = -1, 0, 1.

CAPÍTULO 4

DIVISIBILIDADE

57

28) Mostrar que (1 + 2 + . . . + n) | 3(12

+ 22 + . . .

+ n2) para todo n > 1.

29) Mostre que todo inteiro ímpar, quadrado

perfeito, é da forma 4n + 1.

30) Na divisão de 392 por 45, determinar:

a) o maior inteiro que se pode somar ao

dividendo sem alterar o quociente.

b) o maior inteiro que se pode subtrair ao

dividendo sem alterar o quociente.

31) Numa divisão de dois inteiros, o quociente é 16

e o resto 167. Determinar o maior inteiro que

se pode somar ao dividendo e ao divisor sem

alterar o quociente.

32) Achar o maior inteiro de quatro algarismos

divisível por 13 e o menor inteiro de cinco

algarismos divisível por 15.

33) Achar um inteiro de quatro algarismos,

quadrado perfeito, divisível por 27 e terminado

em 6.

34) Mostre que se a, b e c são inteiros ímpares, a

equação ax2bx c 0 não tem raiz

racional.

35) Um tabuleiro 6 6 está coberto com dominós

2 1. Mostre que existe uma reta que separa as

peças do tabuleiro sem cortar nenhum dominó.

36) Dividindo-se o número 245 por um número

natural b, obtém-se quociente 5 e resto r.

Determine o valor da soma dos valores

possíveis para b.

37) A divisão de um certo número inteiro N por

1994 deixa resto 148. Calcule o resto da

divisão de N + 2000 pelo mesmo número 1994.

38) Considere quatro números inteiros a, b, c e d.

Prove que o produto: (a-b) . (c-a) . (d-a) . (d-c).

(d-b). (c-b) é divisível por 12.

39) Prove que n na b é divisível por a+b se n é

ímpar.

58

Capítulo 5

MÁXIMO DIVISOR COMUM

5.1. Máximo Divisor Comum de Dois Inteiros

Definição 5.1 Sejam a e b dois inteiros não conjuntamente nulos (a 0 ou b 0). Chama-se

máximo divisor comum de a e b o inteiro positivo d (d 0) que satisfaz às condições:

1) d | a e d | b;

2) se c | a e se c | b, então c d.

Observe-se que, pela condição (1), d é um divisor comum de a e b, e pela condição

(2), d é o maior dentre todos os divisores comuns de a e b.

O máximo divisor comum de a e b indica-se pela notação mdc(a,b).

É imediato que o mdc(a,b) = mdc(b,a). Em particular:

(i) o mdc(0,0) não existe.

(ii) o mdc(a,1) = 1

(iii) se a 0, então o mdc(a,0) = | a |

(iv) se a | b, então o mdc(a,b) = | a |

Assim, por exemplo:

mdc(8,1) = 1 mdc(-3,0) = | -3 | = 3 mdc(-6,12) = | -6 | = 6.

Exemplo 5.1 Sejam os inteiros a = 16 e b = 24. Os divisores comuns positivos de 16 e 24 são

1, 2, 4 e 8, e como o maior é 8, segue-se que o mdc(16,24) = 8.

Observa-se que

mdc(-16,24) = mdc(16,-24) = mdc(-16,-24) = 8.

Exemplo 5.2 Sejam os inteiros a = -24 e b = 60. Os divisores comuns positivos de –24 e 60

são 1, 2, 3, 4, 6 e 12, e como o maior deles é 12, segue-se que o mdc(-24,60) = 12.

CAPÍTULO 5

MÁXIMO DIVISOR COMUM

59

5.2. Existência e unicidade do MDC.

Teorema 5.1 (Identidade de Bézout ) Se a e b são dois inteiros não conjuntamente nulos ( a

0 ou b 0), então existe e é único o mdc(a,b); além disso, existem inteiros x e y tais que

Isto é, o mdc(a,b) é uma combinação linear de a e b.

Nota: Algumas fontes creditam este teorema ao matemático francês Claude

Gaspard Bachet de Méziriac e não ao também francês, Etienne Bézout.

Demonstração:

Seja S o conjunto de todos os inteiros positivos da forma au + bv, com u, v Z, isto é:

Este conjunto S não vazio (S ), porque, por exemplo, se a 0, então um dos dois inteiros:

é positivo e pertence a S. Logo, pelo “Princípio da boa ordenação”, existe e é único o

elemento mínimo d de S: minS = d 0. E, consoante a definição de S, existem inteiros x e y

tais que d = ax + by.

Posto isto, vamos mostrar que d = mdc(a,b). Com efeito, pelo algoritmo da divisão, temos:

a = dq + r, com 0 r d

O que dá:

r = a – dq = a – (ax + by)q = a(1 – qx) + d(-qy)

Isto é, o resto r é uma combinação linear de a e b. Como 0 r d e d 0 é o elemento

mínimo de S, segue-se que r = 0 e a = dq, isto é, d | a.

Com raciocínio inteiramente análogo se conclui que também d | b. Logo, d é um divisor

comum positivo de a e b.

Finalmente, se c é um divisor comum positivo qualquer de a e b ( c | b, c 0), então:

Isto é, d é o maior divisor comum positivo de a e b, ou seja:

e o teorema fica demonstrado.

mdc(a,b) = d = ax + by x, y Z

c | (ax + by) c | d c d

a = a.1 + b.0 e -a = a.(-1) + b.0

S = { au + bv | au + bv 0 e u, v Z }

mdc(a,b) = ax + by

CAPÍTULO 5

MÁXIMO DIVISOR COMUM

60

Nota: A demonstração do teorema 1 deixa ver que o mdc(a,b) é o menor

inteiro positivo da forma ax + by, isto é, que pode ser expresso como

combinação linear de a e b. Mas, esta representação do mdc(a,b) como

combinação linear de a e b não pe punica, pois, temos:

qualquer que seja o inteiro t.

Importa ainda notar que, se

Para algum par de inteiros r e s, então d não é necessariamente o mdc(a,b). Assim, por

exemplo, se:

mdc(a,b) = ax + by

então

t.mdc(a,b) = atx + bty

Para todo inteiro t, isto é:

onde d = t.mdc(a,b), r = tx e s = ty.

Exemplo 5.3 Sejam os inteiros a = 6 e b = 27. Temos:

qualquer que seja o inteiro t.

Exemplo 5.4 Sejam os inteiros a = -8 e b = -36. Temos:

Teorema 5.2 Se a e b são dois inteiros não conjuntamente nulos (a 0 ou b 0), então o

conjunto de todos os múltiplos do mdc(a,b) = d é

T = { ax + by | x,y Z }

mdc(-8,-36) = 4 = (-8)4 + (-36)(-1)

mdc(6,27) = 3 = 6(-4 + 27t) + 27(1 – 6t)

d = ar + bd

d = ar + bs.

mdc(a,b) = d = a(x + bt) + b(y - at)

CAPÍTULO 5

MÁXIMO DIVISOR COMUM

61

Demonstração:

Como d | a e d | b, segue-se que d | (ax + by), quaisquer que sejam os inteiros x e y, e por

conseguinte todo elemento do conjunto T e um múltiplo de d.

Por outro lado, existem inteiros x0 e y0 tais que

de modo que todo múltiplo kd de d é da forma:

isto é, kd é uma combinação linear de a e b e, portanto, kd é elemento do conjunto T.

5.3. Inteiros Relativamente Primos (coprimos ou primos entre si)

Definição: Sejam a e b dois inteiros não conjuntamente nulos (a 0 e b 0). Diz-se que a e b

são relativamente primos se, e somente se, o mdc(a,b) = 1.

Assim, por exemplo, são relativamente primos os inteiros: 2 e 5, -9 e 16, -27 e –35, pois,

temos:

Dois inteiros a e b coprimos admitem como únicos divisores comuns 1 e –1.

Teorema 5.3 Dois inteiros a e b, não conjuntamente nulos (a 0 e b 0), são primos entre si

se, e somente se, existem inteiros x e y tais que ax + by = 1.

Demonstração:

( ) Se a e b são relativamente primos, então o mdc(a,b) = 1 e por conseguinte existem

inteiros x e y tais que

ax + by = 1

( ) Reciprocamente, se existem inteiros x e y tais que ax + by = 1 e se o mdc(a,b) = d, então

d | a e d | b. Logo, d | (ax + by) e d | 1, o que implica d = 1 ou mdc(a,b) =1, isto é, a e b são

primos entre si.

Corolário 5.1 Se o mdc(a,b) = d, então o mdc( a/d , b/d ) = 1.

Demonstração:

Preliminarmente, observa-se que a/d e b/d são inteiros, porque d é um divisor comum de a e b.

Posto isso, se o mdc(a,b) = d, então existem inteiros x e y tais que ax + by = d, ou seja,

dividindo ambos os membros desta igualdade por d:

mdc(2,5) = mdc(-9,16) = mdc(-27,-35) = 1

kd = k(ax0 + by0) = a(kz0) + b(ky0)

d = ax0 + by0,

CAPÍTULO 5

MÁXIMO DIVISOR COMUM

62

Logo, pelo teorema anterior, os inteiros a/d e b/d são primos entre si, isto é, o mdc

(a/d ,b/d) = 1.

Assim, por exemplo:

Corolário 5.2 Se a | b e se o mdc(b,c) = 1, então o

Demonstração:

Com efeito:

Portanto:

Corolário 5.3 Se a | c, se b | c e se o mdc(a,b) = 1, então ab | c.

Demonstração:

Com efeito:

Portanto:

Observe-se que somente as condições a | c e b | c não implicam ab | c.

Assim, por exemplo, 6 | 24 e 8 | 24, mas 6.8 | 24 (o mdc(6,8) = 2 1).

Corolário 5.4 Se mdc(a,b) = 1 = mdc(a,c), então o mdc(a,bc) = 1.

c = a(nq2)x = b(aq1)y = ab(q2x + q1y) ab | c

a | c c = aq1, com q1 Z

b | c c = bq2, com q2 Z

mdc(a,b) = 1 ax + by = 1, com x,y Z

acx + bcy = c

a(qx) + cy = 1 mdc(a,c) = 1

a | b b = aq, com q Z

mdc(b,c) = 1 bx + cy = 1, com x, y Z.

mdc (a,c) = 1.

mdc(-12,30) = 6 e mdc(-12/6 , 30/6) = mdc(-2,5) = 1.

(a/d)x + (b/d)y = 1

CAPÍTULO 5

MÁXIMO DIVISOR COMUM

63

Demonstração:

Com efeito:

Portanto:

o que implica mdc(a,bc) = 1.

Corolário 5.5 Se o mdc(a,bc) = 1, então mdc(a,b) = 1 = mdc(a,c).

Demonstração:

Com efeito:

Portanto:

Note-se que esta proposição é a recíproca da anterior.

Teorema 5.4 (de Euclides) Se a | bc e se o mdc(a,b) = 1, então a | c.

Demonstração:

Com efeito:

Portanto:

Note-se que somente a condição a | bc não implica que a | c.

Assim, por exemplo, 12 | 9.8, mas 12 | 9 e 12 | 8 mdc(12,9) 1 e mdc(12,8) 1.

c = acx + aqy = a(cx + qy) a | c

a | bc bc = aq, com q Z

mdc(a,b) = 1 ax + by = 1, com x, y Z

acx + bcy = c

ax + b(cy) = 1 mdc(a,b) = 1

ax + c(by) = 1 mdc(a,c) = 1

mdc(a,bc) = 1 ax + (bc)y = 1, com x,y Z.

1 = ax + by(az + ct) = a(x + byz) + bc(yt)

mdc(a,b) = 1 ax + by = 1, com x,y Z

mdc(a,b) = 1 az + ct = 1, com z,t Z

CAPÍTULO 5

MÁXIMO DIVISOR COMUM

64

5.4. Caracterização do MDC de Dois Inteiros

Teorema 5.5 Sejam a e b dois inteiros não conjuntamente nulos (a 0 ou b 0). Um inteiro

positivo d (d 0) é o mdc(a,b) se e somente se satisfaz às condições:

(1) d | a e d | b

(2) se c | a e se c | b, então c | d

Demonstração:

( ) Suponhamos que o mdc (a, b) = d. Então, d | a e d | b, isto é, a condição (1) é satisfeita.

Por outra parte, existem inteiros x e y tais que ax + by = d e, portanto, se c | a e se c | b, então

c | (ax + by) e c | d, isto é, a condição (2) também é satisfeita.

( ) Reciprocamente, seja d um inteiro positivo qualquer que satisfaz às condições (1) e (2).

Então, pela condição (2), todo divisor comum c de a e b também é divisor de d, isto é, c | d, e

isto implica c d. Logo, d é o mdc(a,b).

5.5. MDC de vários Inteiros

O conceito de máximo divisor comum, definido para dois inteiros a e b, estende-se de maneira

natural a mais de dois inteiros. No caso de três inteiros a, b e c, não todos nulos, o mdc(a,b,c)

é o inteiro positivo d (d 0) que satisfaz às condições:

(1) d | a, d | b e d | c

(2) se e | a, se e | b e se e | c, então e d

Assim, por exemplo:

Importa notar que três inteiros a, b e c podem ser primos entre si, isto é, o mdc(a,b,c) = 1, sem

que sejam primos entre si dois a dois, que é o caso, por exemplo, dos inteiros 6, 10 e 15.

Teorema 5.6 O mdc(a,b,c) = mdc(mdc(a,b),c).

Demonstração:

Com efeito, seja mdc(a,b,c) = d e mdc(a,b) = e. Então, d | a, d | b e d | c, e como existem

inteiros x e y tais que ax + by = e, segue-se que d | (ax + by) ou d | e, isto é, d é um divisor

comum de e e c (d | e e d | c).

mdc(39,42,54) = 3 e mdc(49,210,350) = 7

CAPÍTULO 5

MÁXIMO DIVISOR COMUM

65

Por outro lado, se f é um divisor comum qualquer de e e c (f | e e f | c), então f | a, f | b e f |

c, o que implica f d.

Assim sendo, o mdc(e,c) = d, isto é, p mdc(mdc(a,b),c) = mdc(a,b,c).

Exemplo 5.5 Determinar o mdc(570,810,495).

Pelo teorema anterior, temos:

e como o mdc(570,810) = 30, segue-se que o

EXERCÍCIOS

1. Determinar:

a) mdc(11, 99)

b) mdc(-21,14)

c) mdc(17, 18)

1. Achar os elementos do conjunto A = {1, 2, 3,

4, 5} que são relativamente primos com 8.

2. Seja o conjunto A = {1, 2, 3, 4, 5, 6}.

Enumerar os elementos do conjunto X = {x

A | mdc(x, 6) = 1}.

3. Sabendo que o mdc(a, 0) = 13, achar todos os

valores do inteiro a.

4. Achar o menor inteiro positivo c, da forma c =

22x + 55y, onde x e y são dois inteiros.

5. Sendo n um inteiro qualquer, calcular o mdc(n,

n + 1).

6. Calcular

a) mdc(n, n + 2), sendo n um inteiro par.

b) mdc(n, n + 2), sendo n um inteiro ímpar.

7. Sendo n um inteiro qualquer, achar os

possíveis valores do máximo divisor comum

dos inteiros n e n + 10.

8. Sendo n um inteiro qualquer, calcular o mdc(n

– 1, n2 + n + 1).

9. Sendo a e b dois inteiros não conjuntamente

nulos (a 0 ou b 0), mostrar: mdc (a, b) =

mdc (-a, b) = mdc (a, -b) = mdc (-a, -b).

10. Sejam a, b e c inteiros. Demonstrar:

a) existem inteiros x e y tais que c = ax + by

se e somente se o mdc(a, b) | c.

b) se existem inteiros x e y tais que ax + by =

mdc(a, b) então mdc(x, y) = 1.

11. Sejam a, b e c inteiros. Demonstrar:

a) se o mdc(a, b) = 1 então o mdc (ac, b) =

mdc (b, c)

b) Se o mdc(a, b) = 1 e se c | (a + b), então o

mdc(a, c) = 1 e o mdc(b, c) = 1.

c) se b | c, então o mdc(a, b) = mdc (a + c, b).

d) Se mdc (a, b) = 1, então mdc (am, b

n) = 1.

e) Se mdc (a, b)= 1, então mdc (a+b, a²-

ab+b²) =1 ou 3

f) O mdc (a,b) = mdc (a, b + ac), com c

positivo.

12. Calcular o mdc (a + b, a – b) sabendo que a e b

são inteiros primos entre si.

13. Seja 10 120a e mdc ( a, 120 ) =10.

Determine o valor de a.

mdc(570,810,495) = mdc(30,495) = 15

mdc(570,810,495) = mdc(mdc(570,810),495)

CAPÍTULO 5

MÁXIMO DIVISOR COMUM

66

14. Achar o maior inteiro positivo pelo qual se

devem dividir os inteiros 160, 198 e 370 para

que os restos sejam respectivamente 7, 11 e 13.

15. Determinar os inteiros positivos a e b, sabendo-

se que:

a) a + b = 63 e mdc(a, b) = 9

b) ab = 756 e mdc(a, b) = 6.

16. Os restos das divisões dos inteiros 4933 e 4435

por um inteiro positivo n são respectivamente

37 e 19. Achar o inteiro n.

17. Demonstrar que se n = abc + 1, então o mdc(n,

a) = mdc(n, b) = mdc(n, c) = 1.

18. Demonstrar que mdc(mdc(a, b), b) = mdc(a, b)

19. Demonstrar que o mdc(n + k, k) = 1 se e

somente se o mdc(n, k) = 1.

20. Demonstrar que, se a | bc e se mdc(a, b) = d,

então a | cd.

21. Demonstrar que, se a | c, se b | c e se o mdc(a,

b) = d então ab | cd.

22. Demonstrar que se mdc(a, b) = 1 e se mdc(a,c)

= d,então mdc(a, bc) = d.

23. O inteiro ímpar d é um divisor de a + b e de a –

b. Demontrar que d também é um divisor do

mdc(a, b).

24. Os inteiros positivos a, b e c são tais que o

mdc(a, b) = 1, a | c e c | b. Demonstrar que a =

1.

25. O mdc(n, n + k) = 1 para todo inteiro positivo

n. Demonstrar que k = 1 ou k = -1.

26. Demonstrar que mdc(a, b) = mdc(a + kb, b)

para todo inteiro k.

27. O mdc(a, 4) = 2 = mdc(b, 4). Demonstrar que o

mdc(a + b, 4) = 4.

28. Os inteiros positivos m e n são tais que o

mdc(m, n) = d. Mostrar que o mdc (2m – 1, 2

n –

1) = 2d – 1.

29. Demonstrar que mdc(a, b) = mdc(a, b, a + b).

30. Demonstrar que mdc(a, b) = mdc(a, b, ax +

by), quaisquer que seja os inteiros x e y.

31. O mdc(a, b) = p, sendo p um primo. Achar os

possíveis valores do

a) mdc (a2, b)

b) mdc(a3, b) = p, mesma conclusão acima.

c) mdc(a2, b

3) = p

2. Pois aparecem 2 fatores

iguais a p em a2 e 3 fatores iguais a p em

b3.

32. Sabendo que o mdc(a, p2) = p e que o mdc(b,

p3) = p

2, onde p é um primo, calcular o mdc

(ab, p4) e o mdc(a + b, p

4).

33. Demonstrar que se o mdc(a, b) = d então o mdc

(a2, b

2) = d

2.

34. Demonstrar que mdc(a, b) = mdc(a, c) implica

mdc(a2, b

2) = mdc(a

2, c

2).

35. Sejam a e k inteiros não conjuntamente nulos.

Demonstrar que mdc(a, a + k) | k.

36. Demonstrar que mdc(a, b) = mdc(a, c) implica

mdc(a, b) = mdc(a, b, c).

37. Demonstrar que mdc(a, b, c) = mdc(mdc(a, b),

mdc(a, c).

38. Sejam a e b inteiros positivos tais que ab é um

quadrado perfeito e o mdc(a, b) = 1.

Demonstrar que a e b são quadrados perfeitos.

39. Demonstrar que mdc( a + b, a – b) > mdc(a, b)

40. Mostrar que o mdc (5n + 6, 5

n + 8) = 1 onde n é

um inteiro ímpar.

41. Sejam a, b, c, d (b d) inteiros tais que mdc(a,

b) = mdc(c, d) = 1. Mostrar que a soma a/b +

c/d não é um inteiro.

42. Determinar os inteiros positivos a e b, sabendo

que a2 – b

2 = 7344 e mdc(a, b) = 12.

43. Dividindo-se dois inteiros positivos pelo seu

mdc, a soma dos quocientes é 8. Determinar os

dois inteiros, sabendo-se que sua soma é 384.

44. Um enxadrista quer decorar uma parede

retangular, dividindo-a em quadrados, como se

fosse um tabuleiro de xadrez. A parede mede

4,40 metros por 2,75 metros. Qual o menor

número de quadrados que ele pode colocar na

parede?

67

Capítulo 6

ALGORITMO DE EUCLIDES –

MÍNIMO MÚLTIPLO COMUM

ouco se sabe sobre a vida e a personalidade de Euclides e se desconhece a data de seu

nascimento. É provável que sua formação matemática tenha se dado na escola platônica

de Atenas. Ele foi professor do Museu em Alexandria.

Euclides escreveu cerca de uma dúzia de tratados, cobrindo tópicos desde óptica, astronomia,

música e mecânica até um livro sobre secções cônicas; porém, mais da metade do que ele

escreveu se perdeu. Entre as obras que sobreviveram até hoje temos: Os elementos, Os dados,

Divisão de figuras, Os fenômenos e Óptica.

Os elementos de Euclides não tratam apenas de geometria, mas também de teoria dos

números e álgebra elementar (geométrica). O livro se compõe de quatrocentos e sessenta e

cinco proposições distribuídas em treze livros ou capítulos, dos quais os seis primeiros são

sobre geometria plana elementar, os três seguintes sobre teoria dos números, o livro X sobre

incomensuráveis e os três últimos tratam sobre geometria no espaço.

O livro VII começa com o processo, hoje conhecido como algoritmo euclidiano, para achar o

máximo divisor comum de dois ou mais números inteiros e o usa para verificar se dois

inteiros são primos entre si; encontramos também uma exposição da teoria das proporções

numérica ou pitagórica.

http://www.matematica.br/historia/euclides.html

Comecemos com o seguinte Lema:

Lema: Se a = bq + r, então o mdc(a,b) = mdc(b,r).

Demonstração:

Se o mdc(a,b) = d, então d | a e d | b, o que implica d | (a - bq) ou d | r, isto é, d é um divisor

comum de b e r (d | b e d | r).

Por outro lado, se c é um divisor comum qualquer de b e r (c | b e c | r), então c | (bq + r) ou

c | a, isto é, c é um divisor comum de a e b, o que implica c d. Assim sendo, o mdc(b,r) = d.

6.1. Algoritmo de Euclides

Sejam a e b dois inteiros não conjuntamente nulos ( a 0 ou b 0 ) cujo máximo divisor

comum se deseja determinar.

P

CAPÍTULO 6

ALGORITMO DE EUCLIDES – MÍNIMO MÚLTIPLO COMUM

68

Sejam a e b dois inteiros não conjuntamente nulos ( a 0 ou b 0 ) cujo máximo divisor

comum se deseja determinar.

É imediato:

(1) se a 0, então o mdc(a,0) = | a |

(2) se a 0, então o mdc(a,a) = | a |

(3) se b | a , então o mdc(a,b) = | b |

Além disso, por ser mdc(a,b) = mdc( | a | , | b | ), a determinação do mdc(a,b) reduz-se ao caso

em que a e b são inteiros positivos distintos, por exemplo, com a b, tais que b não divide a,

isto é: a b 0 e b | a. nestas condições, a aplicação repetida do algoritmo da divisão dá-

nos as igualdades:

Como os restos r1, r2, r3, r4, ... são todos inteiros positivos tais que

e existem apenas b – 1 inteiros positivos menores que b, necessariamente se chega a uma

divisão cujo resto rn+1 = 0, isto é, finalmente, teremos:

O último resto rn 0 que aparece nesta sequência de divisões é o máximo divisor comum

procurado de a e b, isto é, o mdc(a,b) = rn, visto que, pelo lema anterior, temos:

este processo prático para o cálculo do máximo divisor comum de dois inteiros positivos a e b

é denominado algoritmo de EUCLIDES ou processo das divisões sucessivas.

É usual o seguinte dispositivo de cálculo no emprego do algoritmo de EUCLIDES:

q1 q2 q3 qn qn+1

a b r1 r2 ... rn-1 rn

r1 r2 r3 r4 0

mdc(a,b) = mdc(b,r1) = mdc(r1,r2) = ... =

= mdc(rn-2,rn-1) = mdc(rn-1,rn) = rn

rn-2 = rn-1qn + rn, 0 rn rn-1

rn-1 = rnqn+1 + rn+1, rn-1 = 0

b r1 r2 r3 r4 ...

a = bq1 + r1, 0 r1 b

b = r1q2 + r2, 0 r2 r1

r1 = r2q3 + r3, 0 r3 r2

r2 = r3q4 + r4, 0 r4 r3

.............................. ........................

CAPÍTULO 6

ALGORITMO DE EUCLIDES – MÍNIMO MÚLTIPLO COMUM

69

Que se traduz na seguinte REGRA: Para se “ achar” o mdc de dois inteiros positivos, dividi-se

o maior pelo menor, este pelo primeiro resto obtido, o segundo resto pelo primeiro, e assim

sucessivamente até encontrar um resto nulo. O último resto não nulo é o máximo divisor

comum procurado.

O algoritmo de EUCLIDES também pode ser usado para achar a expressão do mdc(a,b) = rn

como combinação linear de a e b, para o que basta eliminar sucessivamente os restos rn-1, rn-2,

..., r3, r2, r1 entre as n primeiras igualdades anteriores.

Exemplo 6.1 Achar o mdc(963,657) pelo algoritmo de EUCLIDES e a sua expressão como

combinação linear de 963 e 657,

Temos, sucessivamente:

963 = 657.1 + 306

657 = 306.2 + 45

306 = 45.6 + 36

45 = 36.1 + 9

36 = 9.4 + 0

1 2 6 1 4

963 657 306 45 36 9

306 45 36 9 0

Portanto, o mdc(963,657) = 9 e a sua expressão como combinação linear de 963 e 657 se

obtém eliminando os restos 36, 45 e 306 entre as quatro primeiras igualdades anteriores do

seguinte modo:

9 = 45 – 36 = 45 – (306 – 45.6) =

= - 306 + 7.45 = - 306 + 7(657 – 306.2) =

= 7.657 – 15.306 = 7.657 – 15(963 – 657) =

= 963(-15) + 657.2

isto é:

9 = mdc(963,657) = 963x + 657y

onde x = -15 e y = 22.

Esta respresentação do inteiro 9 = mdc(963,657) como combinação linear de 963 e 657 não é

única. Assim, por exemplo, somando e subtraindo o produto 963.657 ao segundo mebro da

igualdade:

obtemos:

9 = 963(-15 + 657) + 657(22 - 963) =

= 963.642 + 657(-941)

9 = 963(-15) + 657.22

CAPÍTULO 6

ALGORITMO DE EUCLIDES – MÍNIMO MÚLTIPLO COMUM

70

que é uma outra representação do inteiro 9 = mdc(963,657) como combinação linear de 963

e 657.

Exemplo 6.2 Achar o mdc(252,-180) pelo algoritmo de EUCLIDES e a sua expressão como

combinação linear de 252 e –180.

Temos, sucessivamente:

Portanto, o mdc(252,-180) = mdc(252,180) = 36 e como

temos:

onde x = -2 e y = -3, que é a expressão do mdc(252,-180) como combinação linear de 252 e –

180.

Outra representação do inteiro 36 = mdc(252,-180) como combinação linear de 252 e –180 é

a seguinte:

Exemplo 6.3 O mdc de dois inteiros positivos a e b é 74 e na sua determinação pelo algoritmo

de EUCLIDES os quocientes obtidos foram 1, 2, 2, 5, 1 e 3. Calcular a e b.

1 2 2 5 1 3

a b r r1 r2 r3 74

r r1 r2 r3 74 0

Temos, sucessivamente:

a = b + r, b = 2r + r1, r = 2r1 + r2

r1 = 5r2 + r3, r1 = r2 + 74, r3 = 74.3 = 222

36 = 252(-2 + 180) + (-180)(-3 + 252) =

= 252.178 + (-180)249

36 = mdc(252,-180) = 252x + (-180)y

36 = 180 – 72.2 = 180 – (252 – 180)2 =

= 252(-2) + (-180)(-3)

252 = 180.1 + 72

180 = 72.2 + 36

72 = 36.2

CAPÍTULO 6

ALGORITMO DE EUCLIDES – MÍNIMO MÚLTIPLO COMUM

71

Portanto:

Nota: Dispositivo prático para expressar o MDC...(José Paulo Q. Carneiro)

Antes de deduzir o dispositivo prático, vamos vê-lo funcionando no exemplo

citado. Forma-se uma matriz com duas colunas, intituladas “q” e “s, t”.

Primeiramente, preenche-se a coluna “q” com os quocientes obtidos no

algoritmo de Euclides (desprezando o último, correspondente ao resto zero),

na ordem contrária ao de seu aparecimento no algoritmo, e deixando em

branco a primeira linha.

Na coluna “s, t”, coloca-se o número 1 na primeira linha (é sempre 1

mesmo), e na segunda linha repete-se o valor do quociente que

aparece ao seu lado na primeira coluna. No nosso exemplo, fica:

A partir daí, cada valor seguinte da coluna “s, t” vai sendo obtido de acordo com o seguinte

esquema auto-explicativo:

Observa-se que os dois últimos valores obtidos na segunda

coluna são, justamente, em valor absoluto, s = 85e t = 539. Isso

não é coincidência. Na realidade, essa matriz resume as contas

que precisam ser feitas com os números relevantes que figuram

no processo.

Há a questão dos sinais, isto é, o esquema só fornece os valores absolutos de s e t. Para

decidir sobre os sinais, aplica-se a seguinte regra: se o número de quocientes aproveitados

(ou seja, o número de linhas preenchidas na coluna “q”) for ímpar, então s é positivo e t

negativo; se for par, ocorrerá justamente o contrário: s é negativo e t é positivo. Deve ser

notado que se pode também, em vez de decorar mais uma regra, experimentar, comparando

os valores de e . No exemplo, | s | a = 85 x 7248 = 616080 e | t | b = 539 x 1143 =

616077, ficando claro que, para obter o mdc 3, é necessário fazer s = 85 e t = –539.

Outro exemplo: a = 1741 e b = 85

20 2 13 1 2

1741 85 41 3 2 1

41 3 2 1 0

r2 = 222 + 74 = 296, r1 = 5.296 + 222 = 1702

r = 2.1702 + 296 = 3700, b = 2.3700 + 1702 = 9102

a = 9102 + 3700 = 12802

CAPÍTULO 6

ALGORITMO DE EUCLIDES – MÍNIMO MÚLTIPLO COMUM

72

Formando então a matriz:

Como agora o número de quocientes aproveitados é par, toma-

se: s = 29 s = –29 e t = 549, de modo que

( 29) x 1741 + (594) x 85 = 50 489 + 40 490 = 1

Justificativa do dispositivo prático

Vamos agora justificar o dispositivo prático,

por meio da formulação literal do problema.

Supondo > b > 0 inteiros, aplica-se o

algoritmo de Euclides, encontrando:

onde rn = m = mdc (a, b). Os quocientes já foram numerados de trás para frente, isto é, o

último quociente (correspondente ao resto 0) é q0, o penúltimo é q1, etc., e o primeiro é qn,

para manter coerência com o dispositivo prático.

Desprezando, como se fez nos exemplos, o último quociente q0, vê-se, da primeira equação,

que r1 pode ser escrito como uma combinação linear de a e b, ou seja, r1 = a – qnb.

Substituindo esse valor na penúltima equação, vê-se que r2 também pode ser escrito como

uma combinação linear de a e b, a saber, r2 = –qn-1a + (1 + qnqn–1)b. E assim por diante,

para todos os r. Em particular, calcula-se m = n = sa+ tb.

Repare que acabamos de demonstrar o Teorema Fundamental da Teoria dos Números. Se o

leitor não estiver satisfeito com o “e assim por diante”, pode formalizar a demonstração por

indução. É realmente fácil. Basta observar que uma combinação linear de combinações

lineares de a e b é também uma combinação linear de a e b.

n s t

1 1

2

3

4

... ... ...

A lei de formação é clara (e pode ser verificada por indução):

com os valores iniciais: s1 = 1 e t1 = –q1.

1

11

1:t

k k

k kk k

s tpara n

t q S

CAPÍTULO 6

ALGORITMO DE EUCLIDES – MÍNIMO MÚLTIPLO COMUM

73

Observando que cada s repete o t anterior, basta trabalhar com os t e notar que, ao final do

processo, o último valor é o t procurado, enquanto o penúltimo é o s. Foi por isso que

introduzimos a coluna “s, t” na matriz da nossa regra prática. Além disso, levando em conta a

alternância dos sinais, é suficiente lidar com os valores absolutos de t, tomando o cuidado

de, no final, fazer a correção de sinal conveniente, conforme n seja par ou ímpar. Com essas

providências, a lei de formação fica:

que é justamente o que se fez na regra prática.

Alguém pode ainda perguntar: quando se escreve o máximo divisor comum de dois inteiros

como uma combinação linear deles, essa representação é única? A resposta é não (ver [4],

para maiores detalhes). Uma vez encontrados s e t tais que , basta tomar um

inteiro qualquer u, e também será verdade que m = s’ a + t’b, onde s’ = s + bu e t’ = t – au,

como pode ser verificado diretamente por substituição.

Em nosso primeiro exemplo, tomando , temos:

Verificando: s'a + t'b= 1228 x 7 248 + ( 7 787) x 1 143 = 8900544 – 8900541 = 3

Teorema 6.1 Se k 0, então o mdc(ka,kb) = k.mdc(a,b).

Demonstração:

Multiplicando ambos os membros de cada uma das n+1 igualdades que dão o mdc(a,b) = rn

pelo algoritmo de EUCLIDES por k, obtemos:

Obviamente, estas n+1 igualdades outra coisa não são que o algoritmo de EUCLIDES

aplicado aos inteiros ak e bk, e por conseguinte o mdc(ak,bk) é o último resto rnk 0, isto é:

mdc(ak,bk) = rnk = k.mdc(a,b)

ak = (bk)q1 + r1k, 0 r1k bk

bk = (r1k)q2 + r2k, 0 r2k r1k

r1k = (r2k)q3 + r3k, 0 r3k r2k

............................. .....................

rn-2k = (rn-1k)qn + rnk, 0 rnk rn-1k

rn-1k = (rnk)qn+1 + 0

s' = 85 + 1143 = 1228, enquanto t’ = – 539 – 7248 = – 7728

| tx | = qx | tx -1 | + | tx-2 |,

CAPÍTULO 6

ALGORITMO DE EUCLIDES – MÍNIMO MÚLTIPLO COMUM

74

Assim, por exemplo:

Corolário 6.1 Para todo k 0, o mdc(ka,kb) = | k |.mdc(a,b)

Demonstração:

Se k 0, nada há que demonstrar, e se k 0, então –k = | k | 0 e, pelo teorema anterior,

temos:

6.2 . Múltiplos comuns de dois inteiros

O conjunto de todos os múltiplos de um inteiro qualquer a 0 indica-se por M(a), isto é:

M(A) = {x Z | a | x } = {aq | q Z}

Assim, por exemplo:

M(-1) = M(1) = Z

M(5) = {5q | q Z} = {0, +5, +10, +15, +20, ...}

É imediato que, para todo inteiro a 0, se tem M(a) = M(-a).

Definição 6.1 Sejam a e b dois inteiros diferentes de zero (a 0 e b 0). Chama-se múltiplo

comum de a e b todo inteiro x tal que a | x e b | x.

Em outros termos, múltiplo comum de a e b é todo inteiro que pertence simultaneamente aos

conjuntos M(a) e M(b).

O conjunto de todos os múltiplos comuns de a e de b indica-se por M(a,b). Portanto,

simbolicamente:

M(a,b) = {x Z | a | x e b | x}

Ou seja:

M(a,b) = {x Z | x M(a) e x M(b)}

E, portanto:

M(a,b) = M(a) M(b)

mdc(ak,bk) = mdc(-ak,-bk) =

= mdc(a.| k |, b.| k |) = | k |.mdc(a,b)

mdc(12,30) = mdc(2.6,5.6) = 6.mdc(2,5) = 6.1 = 6

CAPÍTULO 6

ALGORITMO DE EUCLIDES – MÍNIMO MÚLTIPLO COMUM

75

A interseção ( ) é uma operação comutativa, de modo que M(a,b) não depende da ordem

dos inteiros dados a e b, isto é: M(a,b) = M(b,a).

Obviamente, 0 é um múltiplo comum de a e b: 0 M(a,b). E os produtos ab e –(ab) também

são múltiplos comuns de a e b.

Exemplo 6.4 Sejam os inteiros a = 12 e b = -18. Temos:

M(12) = {12q | q Z} = {0, +12, +24, +36, +48, +60, +72, ...}

M(-18) = {-18q | q Z} = {0, +18, +36, +54, +72, +90, ...}

Portanto:

M(12,-18) = M(12) M(-18) = {0, +36, +72, ...}

6.3. Mínimo Múltiplo comum de dois inteiros

Definição 6.2 Sejam a e b dois inteiros diferentes de zero (a 0 e b 0). Chama-se mínimo

múltiplo comum de a e b o inteiro positivo m (m 0) que satisfaz às condições:

Observe-se que, pela condição (1), m é um múltiplo comum de a e b, e pela condição (2), m é

o menor dentre todos os múltiplos comuns positivos de a e b.

O mínimo múltiplo comum de a e b indica-se pela notação mmc(a,b).

Pelo “Princípio da boa ordenação” , o conjunto dos múltiplos comuns positivos de a e b

possui o elemento mínimo e, portanto, o mmc(a,b) existe sempre e é único. Além disso, por

ser o prodtudo ab um múltiplo comum de a e b, segue-se que o mmc(a,b) |ab|.

Em particular, se a | b, então o mmc(a,b) = | b |.

Exemplo 6.5 Sejam os inteiros a = -12 e b = 30. Os múltiplos comuns positivos de –12 e 30

são 60, 120, 180, ..., e como o menor deles é 60, segue-se que o mmc(-12,30) = 60.

Relação entre mdc e o mmc

Teorema 6.2 Para todo par de inteiros positivos a e b subsiste a relação:

mdc(a,b).mmc(a,b) = |ab|

(1) a | m e b | m

(2) se a | c e se b | c, com c 0, então m c.

CAPÍTULO 6

ALGORITMO DE EUCLIDES – MÍNIMO MÚLTIPLO COMUM

76

Demonstração:

Seja mdc(a,b) = d e mmc(a,b) = m. Como a | a(b/d) e b | b(a/d), segue-se que ab/d é um

múltiplo comum de a e b. Portanto, existe um inteiro positivo k tal que

ab/d = mk, k N

o que implica:

a/d = (m/b)k e b/d = (m/a)k

isto é, k é um divisor comum dos inteiros a/d e b/d. Mas, a/d e b/d são primos entre si

(corolário 1 desta apostila), de modo que k = 1. Assim sendo, temos:

ab/d = m ou ab = dm

isto é:

ab = mdc(a,b).mmc(a,b)

Esta importante relação permite determinar o mmc de dois inteiros quando se conhece o seu

mdc, e vice-versa.

Exemplo 6.6 Determinar o mmc(963,657).

Pelo algoritmo de EUCLIDES, temos mdc(963,657) = 9. Portanto:

Corolário 6.2 Para todo par de inteiros positivos a e b, o mmc(a,b) = ab se e somente se o

mdc(a,b) = 1.

Demonstração:

( ) Se o mdc(a,b) = 1, então:

( ) Reciprocamente, se o mmc(a,b) = ab, então:

6.5. MMC de vários inteiros

O conceito de mínimo múltiplo comum, definido para dois inteiros a e b, estende-se de

maneira natural a mais de dois inteiros. No caso de três inteiros a, b e c, diferentes de zero, o

mmc(a,b,c) é o inteiro positivo m (m 0) que satisfaz às condições:

mdc(a,b).ab = ab mdc(a,b) = 1

ab = 1.mmc(a,b) = mmc(a,b)

mmc(963,657) = 9

657.963=70299

CAPÍTULO 6

ALGORITMO DE EUCLIDES – MÍNIMO MÚLTIPLO COMUM

77

(1) a | m, b | m e c | m

(2) se a | e, se b | e e se c | e, com e 0, então m e.

Assim, por exemplo: mmc (39,102,75) = 33150.

Exemplo 6.7

Achar inteiros x, y e z que verifiquem a seguinte igualdade 11x + 19y + 3z = 1.

Achando o mdc(11,19)

1 1 2 1 2

19 11 8 3 2 1

8 3 2 1 0

Usando o algoritmo da divisão, podemos escrever:

19 = 11 x 1 + 8

11 = 8 x 1 + 3

8 = 3 x 2 + 2

3 = 2 x 1 + 1

2 = 1 x 2

Desprezando a última igualdade, eliminemos os restos a partir da penúltima igualdade:

1 = 3 – 2

1 = 3 – (8 – 3 x 2)

1 = 3 x 3 – 8

1 = (11 – 8) x 3 – 8

1 = 11 x 3 – 8 x 4

1 = 11 x 3 – (19 – 11) x 4

Achemos agora o mdc(3,1) . Como mdc(3,1) = 1 , vamos escrever este mdc como

combinação de 3 e 1:

Agora substituamos o valor de 1, dado na igualdade ( I ) , na igualdade ( II ):

1 = 3 x 1 + (11 x 7 – 19 x 4) (–2)

1 = 3 x 1 + 11 (–14) + 19 x 8

ou

1 = 11 (–14) + 19(8) + 3(1) .

Logo x = –14, y = 8 e z = 1

1 = 3 x 1 + 1 x (-2) ( II )

1 = 11 x 7 – 19 x 4 ( I )

CAPÍTULO 6

ALGORITMO DE EUCLIDES – MÍNIMO MÚLTIPLO COMUM

78

EXERCÍCIOS

1. Usando o algoritmo de Euclides, determinar:

a) mdc(306, 657)

b) mdc(272, 1479)

c) mdc(884, 1292)

d) mdc(-816, 7209)

e) mdc(7469, 2387)

f) mdc(-5376,-3402)

2. Usando o algoritmo de Euclides, determinar:

a) mdc(285, 675, 405)

b) mdc(209, 299, 102)

c) mdc(69, 398, 253)

3. Usando o algoritmo de Euclides, achar os

inteiros x e y que verifiquem cada uma das

seguintes igualdades:

a) mdc(56, 72) = 56x + 72y

b) mdc(24, 138) = 24x + 138y

c) mdc(119, 272)

d) mdc(1769, 2378) = 1769x + 2378y

4. Achar os inteiros x e y que verifiquem cada

uma das seguintes igualdades:

a) 78x + 32y = 2.

b) 104x + 91y = 13

c) 31x + 19y = 7

d) 42x + 26y = 16.

e) 288x + 51x = 3.

f) 52x + 13y = 1

g) 145x + 58y = 87

h) 17x + 5y = -2

5. Achar os inteiros x, y e z que verifiquem cada

uma das seguintes igualdades.

a) 11x + 19y +5z = 1.

b) 56x + 6y + 32z = 2.

c) 6x + 3y + 15z = 9.

d) 14x + 7y + 21z = 4.

6. Achar inteiros x, y e z que verifiquem a

igualdade 198x + 288y + 512z = mdc(198,

288, 512)

7. Calcular

As soluções de todos os itens podem ser

obtidas a partir da propriedade mdc(a,

b).mmc(a, b) = a . b.

Calcula-se o mdc pelo algoritmo de Euclides e

a seguir divide o produto ab pelo mdc(a, b)

a) mmc( 45, 21).

b) mmc(83, 68)

c) mmc( 120, 110)

d) mmc(86, 71)

e) mmc(224, 192)

f) mmc(1287, 507)

g) mmc(143, 227)

h) mmc(306, 657)

8. O mdc de dois inteiros positivos a e b é 8 e na

sua determinação pelo algoritmo de Euclides os

quocientes sucessivamente obtidos foram 2, 1,

1 e 4. Calcular a e b.

9. Determinar os inteiros positivos a e b sabendo:

a) ab = 4032 e mmc(a, b) = 336

b) mdc(a, b) = 8 e mmc(a, b) = 560

c) a + b = 589 e mmc(a, b)/mdc(a,b) = 84

10. Demonstrar que se a e b são inteiros positivos

tais que o mdc(a, b) = mmc(a, b) então a = b.

11. Sabendo que o mdc(a,b) = 1 , demonstrar:

a) mdc(2 a + b, a + 2 b) = 1 ou 3

b) mdc(a + b, a2 + b

2) = 1 ou 2

c) mdc(a + b, a2 – a

b + b

2) = 1 ou 3

12. Sendo a e b inteiros positivos, demonstrar que

o mdc(a, b) sempre divide o mmc(a, b).

13. Quantos pares de inteiros positivos A e B

existem cujo mínimo múltiplo comum é

126000?

Considere o par (A,B) como sendo o mesmo

que (B,A)

14. Calcule a soma dos números entre 200 e 500

que são múltiplos de 6 ou de 14, mas não

simultaneamente múltiplos de ambos."

15. O máximo divisor comum, o menor divisor

comum e o mínimo múltiplo comum dos

números 4, 8 e 12, são, respectivamente.

79

Capítulo 7

NÚMEROS PRIMOS

7.1. Introdução

A noção de número primo foi, muito provavelmente, introduzida por Pitágoras, 530 AC,

sendo que a mesma desempenhou um papel central tanto na matemática como no misticismo

pitagórico.

A escola pitagórica dava grande importância ao número um, que era chamada de unidade (em

grego: Monad). Os demais números inteiros naturais – o 2, 3, 4, etc – tinham caráter

subalterno, sendo vistos como meras multiplicidades geradas pela unidade e por isso recebiam

a denominação de número (em grego: Arithmós).

Entre os pitagóricos a preocupação com a geração dos números não parava por aí. Já o próprio

Pitágoras teria atinado que existem dois tipos de arithmós:

• Os protoi arithmós (números primários ou primos), que são aqueles que não podem ser

gerados – através da multiplicação – por outros arithmós, como é o caso de 2, 3, 5, 7...

• Os deuterói arithmós (números secundários), podem ser gerados por outros arithmós, por

exemplo, 4 = 2.2, 6 = 3.2, etc.

Ainda por influência dos Pitagóricos , por muitos séculos houve polemica a respeito da

primalidade do número dois. Os primeiros pitagóricos chamavam-lhe Dyad, atribuíam-lhe

caráter especial – embora menos importante que a unidade Monad – e alguns deles não o

incluíam entre os arithmós. Consequentemente, muitos pitagóricos não consideravam o dois

como primo. É só pela época de Aristóteles, 350 AC, que passou a ser considerado como

primo, sendo que este costume foi consagrado pelo livro Elementos de Euclides em cerca de

300 AC. Cabe mencionar que entre os gregos, principalmente os pitagóricos de

várias gerações após Pitágoras, surgiram outras denominações para os númerosprimos,

como: retilíneos, lineares e eutimétricos. Contudo, esta nomenclatura teve uso muito restrito e

caíram em desuso.

CAPÍTULO 7

NÚMEROS PRIMOS

80

Registros de documentos gregos

Foi supracitado que a noção de primo fora, muito provavelmente, introduzida por Pitágoras.

Com efeito, é impossível ter completa segurança nessa atribuição, pois Pitágoras não deixou

nenhum registro escrito de seus trabalhos e os documentos mais antigos que temos falando de

suas idéias resumem-se a pequenos fragmentos de textos escritos várias gerações após ele.

Entretanto, esses fragmentos, apesar de conterem informações muito escassas, são unânimes

em afirmar que Pitágoras iniciou o estudo de números primos.

O mais antigo livro de matemática que chegou completo aos nossos dias e que desenvolve

sistematicamente o estudo de números primos é Os Elementos de Euclides. Como é sabido,

Euclides seguiu muito de perto as orientações matemáticas dos pitagóricos. Assim não é

surpreendente que, no capítulo em que trata da teoria dos números, ele defina número primo

de um modo absolutamente compatível com as idéias pitagóricas expostas acima. Elementos,

Vol. VII, def 11, temos:

“protós arithmós estin monadi mone metroymenos”.

Ou seja: Número primo é todo aquele que só pode ser medido através da unidade.

Surgimento da denominação latina

A arithmetiké do grego Nikomachos, 100 dC, é o mais antigo livro de Teoria dos Números,

posterior a Elementos de Euclides, que chegou aos nossos dias. Trata-se de uma visão de

filósofo e letrado em Elementos, sendo que não há uma única demonstração entre os poucos

tópicos abordados. Apesar disso, teve grande repercussão na época e foi a base do primeiro

livro em latim que se escreveu sobre Teoria do Números: o De Institutione Arithmetica, do

romano Boethius 500 dC.

No livro de Boethius é onde aparece, pela primeira vez, a nomenclatura numerus primus

como tradução do tradicional protós arithmós preservada de Euclides por Nikomachos. Além

disto, Boethius, sempre seguindo Nikomachos, usa a velha classificação pitagórica dos

números naturais: primos incompostos versus secundários ou compostos.

O livro de Boethius foi, durante cerca de seiscentos anos, a única fonte de estudos de Teoria

dos Números disponível na Idade Média. Em torno de 1200 dC iniciou o renascimento

científico e matemático pela Europa, com o afluxo das obras árabes e a tradução das obras

gregas preservadas no Mundo Islamita. É dessa época um dos mais influentes livros de todos

os tempos: o Liber Abacci, de Fibonacci. Esse grande matemático, que havia estudado entre

os muçulmanos do Norte da África, diz que acha melhor dizer primus em vez do incomposto

preferido pelos árabes. Ficou assim, definitivamente, consagrada a denominação número

primo na Europa. (http://www.mat.ufrgs.br/~portosil/pqprimo.html)

CAPÍTULO 7

NÚMEROS PRIMOS

81

7.2. Números Primos (do lat. primus, principal. Prime em inglês)

Definição 7.1: Diz-se que um número positivo p > 1 é um número primo ou apenas um primo

se, e somente se, 1 e p são seus únicos divisores positivos. Um inteiro maior que 1 e que não é

primo diz-se composto.

Teorema 7.1: Se um número primo p não divide um inteiro a, então a e p são relativamente

primos (primos entre si).

Demonstração:

Seja d o mdc de a e p. Então d | a e d | p. Da relação d | p, resulta que d = 1 ou d = p, porque p

é primos, e como a segunda igualdade é impossível, porque p não divide a, segue-se que d =

1, isto é, o mdc ( a , p ) = 1. Logo, a e p são relativamente primos.

Corolário 7.1: Propriedade Fundamental dos Números Primos.

Se p é um primo tal que p | ab, então p | a ou p | b (podendo ser fator de ambos, a e b).

Demonstração:

Se p | a, nada há que demonstrar, e se, ao invés, p não divide a, então, pelo teorema anterior, o

mdc (p, a) = 1. logo, pelo teorema 5.4, p | b.

Nota: Observemos que esta propriedade necessária dos números primos é

também suficiente para que um inteiro positivo n seja primo: Pois, se n = k. s

é composto (1< s k < n) , temos n| k.s porém tanto n | k e n | s .

Corolário 7.2: Se p é um primo tal que p | a1a2a3 ... an, então existe um índice k, com 1 k

n tal que p | ak.

Demonstração:

Usando Indução, a proposição é verdadeira para n = 1(imediato) e para n = 2 (pelo corolário

5.1). Supondo, pois, n > 2 e que, se p divide um produto com menos de n fatores, então p

divide pelo menos um dos fatores (hipótese de indução).

Pelo corolário 7.1, se p|a1 a2 . . . an-1, então p|an ou p|a1 a2 ... an-1.

Se p|an, a proposição está demonstrada, e se, ao invés, p|a1 a2 ... an-1, então a hipótese de

indução assegura que p|ak, com 1 k n - 1. Em qualquer dos casos, p divide um dos inteiros

a1, a2, a3, ..., an.

Corolário7.3: Se os inteiros p, q1,q2 ,..., qn são todos primos e se p | q1q2 ... qn, então existe

um índice k, com 1 k n tal que p = qk.

CAPÍTULO 7

NÚMEROS PRIMOS

82

Demonstração:

De fato, pelo corolário 7.2, existe um índice k, com 1 k n , tal que p|qk, como os únicos

divisores positivos de qk são 1 e qk, porque qk, segue-se que p = 1 ou p = qk. Mas, p > 1,

porque p é primo. Logo, p = qk.

Teorema 7.2: Todo inteiro composto possui um divisor primo.

Demonstração:

Seja a um inteiro composto. Consideremos o conjunto A de todos os divisores positivos de a,

exceto os divisores 1 e a, isto é:

Pelo “Princípio da Boa Ordenação” existe o elemento mínimo p de A. que vamos mostrar ser

primo. De fato, se p fosse composto admitiria pelo menos um divisor d tal que 1 < d < p, e

então d|p e d|a, o que implica d|a, isto é, p não seria o elemento mínimo de A, se fosse

composto. Logo, p é primo.

7. 3. Teorema Fundamental da Aritmética.

Todo inteiro positivo n > 1 é igual a um produto de fatores primos.

Demonstração:

Mostraremos a existência da fatoração por indução. Se n é primo não há o que provar

(escrevemos m = 1, p1 = n). Se n é composto podemos escrever n = ab, a, b N , 1 < a < n, 1

< b < n. Por hipótese de indução, a e b se decompõem como produto de primos. Juntando as

fatorações de a e b (e reordenando os fatores) obtemos uma fatoração de n.

Nota: Este teorema (como qualquer outro teorema chamado de

"fundamental") não deveria ser aplicado sem a devida precaução. Existem

inúmeros sistemas numéricos nos quais a fatoração não é única. Por

exemplo, imagine um sistema que tenha apenas inteiros pares com a adição

e multiplicação usual e denominemos um número de "e-primo" se ele não for

o produto de dois outros números pares. Neste caso, alguns "e-primos" são

2, 6, 10, 14, 18, ... Observe que neste sistema, 36 possui duas fatorações

diferentes, 6x 6 e 2 x 18. (http://primes.utm.edu/)

Corolário 7.4: A decomposição de um inteiro positivo n > 1 como produto de fatores primos

é única, a menos da ordem dos fatores.

A = { x | a; 1 < x < a }

CAPÍTULO 7

NÚMEROS PRIMOS

83

Demonstração:

Suponha que

com, p1 ... pm, q1 ... qr . Como p1| q1... qr temos p1 | qi para algum valor de i, donde,

como qi é primo, p1 = qi e p1 q1 . Analogamente temos q1 p1, donde p1 = q1. Mas por

hipótese de indução

admite uma única fatoração, donde m = r e pi = qi para todo i.

Corolário 7.5: Todo inteiro positivo n > 1 admite uma única decomposição da forma

onde, para i =1,2,... , r cada ki é um inteiro positivo e cada pi é um primo, com

denominada decomposição canônica do inteiro positivo n > 1.

Demonstração:

Pelo teorema Fundamental da Aritmética, n é um produto de fatores primos q1 . q2 ... qm, com

q1 q2 ... qm (m 1). Agrupando-se os fatores primos repetidos na forma de potências de

primos, temos a representação enunciada neste corolário e, pelo Teorema Fundamental da

aritmética, tal representação é única.

Nota: Conhecidas as decomposições canônicas de dois inteiros positivos a

> 1 e b > 1, o mdc (a, b) é o produto dos fatores primos comuns às duas

decomposições canônicas tomados cada um com o menor expoente, e o mmc

(a, b) é o produto dos fatores primos comuns e não comuns às duas

decomposições canônicas tomados cada um com o maior expoente.

Corolário 7.6: Se p1.p2 ... pn divide ar, então p1 . p2 ... pn divide a, onde p1 p2 ... pn é o

produto de n primos e n e r são inteiros positivos.

p1 < p2 < ... < pr,

n = p1 ... pm = q1 ... qr

CAPÍTULO 7

NÚMEROS PRIMOS

84

Demonstração:

Se p1 p2 ... pn não divide a, então a não é nenhum dos primos p1p2 ... pn Seja pi, 1 i n , um

desses primos. Então, pi também não é fator primo de ar e, desta forma, não existe pi, 1 i

n, que divida ar, o que implica que p1 . p2 ... pn não divide a

r. Por contraposição, temos a

demonstração pedida.

Observação: O fato de que os expoentes dos primos pi sejam 1 é essencial. Por exemplo 4 =

22 divide 6

2 = 36 , mas 4 não divide 6.

7.4. A Seqüência dos Números Primos

Teorema 7.4: (de Euclides): Há um número infinito de primos.

Demonstração:

Suponha por absurdo que p1, p2, ..., pm fossem todos os primos. O número P = p1 . p2 ... pm +

1 > 1 não seria divisível por nenhum primo, o que contradiz o teorema fundamental da

aritmética.

Proposição 7.1. Para o n-ésimo número primo pn vale a estimativa .

Demostração:

Para n = 1 é verdade que . Suponhamos já provadas as

desigualdades

Se q é primo tal que q | p1 . p2 ... pn + 1, então q > pn, particularmente q pn+1.

Então,

Esta estimativa é exageradamente fraca, no geral pn é significativamente menor que por

exemplo = 256 e p4 é apenas 7. Uma estimativa melhor para pn, postulada por Bertrand

e, demonstrada por Chebychev, é dada pelo teorema seguinte:

CAPÍTULO 7

NÚMEROS PRIMOS

85

Teorema 7.5 (de Chebychev): Para todo inteiro m 2 existe um primo p com m < p< 2m. A

demonstração deste teorema está fora de nosso contexto. Um outro fato provado é que entre

dois cubos consecutivos existe sempre um primo.

Com esse Teorema podemos afirmar que pn+1 < 2pn (para n 1)

Corolário 7.6. Para o n-ésimo número primo pn vale a estimativa pn 2n.

Demonstração:

Temos 2 = p1 21 e pelo teorema de Chebychev: Para todo inteiro positivo n, tem-se pn < pn+1

< 2 . pn. De pn 2n segue que pn + 1 2. 2

n = 2

n+1.

Ao estudarmos a sequência de números primos percebemos que existem infinitos primos em

subconjuntos particulares dos inteiros, como, por exemplo, na sucessão aritmética: {4q + 3; q

inteiro e q 0} = {3, 7, 11, 15, 19, ...}.

Esse resultado foi generalizado pelo matemático alemão Peter Gustav Le-jeune Dirichlet

(1805-1859).

Teorema 7.6 (de Dirichlet). Sejam a e b inteiros primos entre si, isto é, mdc (a, b) = 1.

Existem infinitos primos da forma an + b, onde n é inteiro positivo.

A demonstração deste Teorema exige avançados conhecimentos de Análise Matemática.

Exemplos:

Na sequência 3n + 1, temos os primos 7, 13, 19, 31, 37, 43, 61, 67, 73, 97, 103, ...

Na sequência 9n + 4 temos os primos 13, 31, 67, 103, 139, 157, 193, 211, ...

O resultado de Dirichlet diz não só que o número de primos é infinito, mas também que, se

considerarmos subconjuntos particulares de inteiros, como as sucessões aritméticas acima,

teremos já nesses subconjuntos uma infinidade de primos.

Uma aplicação do Teorema de Dirichlet leva-nos a um resultado obtido pelo matemático

polonês W. Sierpinski , que nos mostra, mais uma vez, a forma surpreendente como os primos

se distribuem nos inteiros.

Teorema 7.7 (de Sierpinski). Dado um inteiro m maior que 1, existe um primo p tal que |p

1|, |p 2|, ..., |p m| são compostos.

Exemplo: Seja m = 10 e p = 19. Temos:

19+1, 19+2, 19+3, 19-4, 19+5, 19+6, 19+7, 19+8, 19+9 e 19-10. Os resultados são todos

números compostos: 20, 21, 22, 15, 24, 25, 26, 27, 28 e 9.

Observe que se tivéssemos escolhido o primo 17, não seria possível construir uma sequência

de compostos com m = 10, pois 17 + 6 = 23 e 17 – 6 = 11, ambos primos.

Demonstração:

Vejamos, em primeiro lugar, que existe um primo p tal que p + 1, p + 2, ..., p + m sejam

compostos. Para cada m dado, o Teorema 1 garante, em particular, que existe um inteiro

primo q maior do que m. Seja a = (q + 1) • (q + 2) • (q + 3) ... (q + m).

Se q divide a, então q divide q + i, e, portanto, q divide i, o que é impossível para 0 < i m <

q. Então a e q são primos entre si. Pelo teorema de Dirichlet, existe um primo p na sequência

CAPÍTULO 7

NÚMEROS PRIMOS

86

an + q. Seja p = (q +1) • (q + 2) • ... • (q + m) • n + q este primo. Então os números p + 1, p +

2, ..., p + m são m números compostos. Para ampliar este resultado, observemos que, por

motivos análogos aos de cima,

a' = (q - m).[q - (m- 1)]...(q - 1)...(q + m)

é primo com q e se p' for um primo da sequência a'n + q, isto é,

p' = (q - m) ... (q - 1) . (q + 1) ... (q + m) . n + q

os números p' – m, ... p' – 1, p' + 1, ... p' + m serão compostos. Assim o número primo p' se

encontra na sucessão dos inteiros, "isolado" por m compostos de cada lado. (RPM 11)

7.5. O Crivo de Eratóstenes.

Eratóstenes, matemático, astrônomo, historiador, geógrafo e filósofo grego, nasceu em Cirene

por volta de 276 a.C. e passou grande parte de sua juventude em Atenas. Com

aproximadamente 40 anos, foi convidado pelo rei Ptolomeu III do Egito para ser bibliotecário

da Universidade de Alexandria.

Ficou conhecido como Beta, e a respeito dessa alcunha existem algumas hipóteses. Alguns

acreditam que, por causa de seu saber, foi elevado à condição de um segundo Platão. Outros,

dizem que tal apelido lhe fora dado por ter sido o segundo bibliotecário da Universidade de

Alexandria. Uma terceira explicação sugere que, apesar de ser talentoso, Eratóstenes não

conseguiu ser o primeiro de seu tempo em nenhum ramo de estudo, em outras palavras, foi

sempre o segundo. Por fim, o historiador James Gow sugeriu que talvez Beta indicasse

simplesmente o número (grego) 2 referente a um gabinete ou a uma sala de leitura da

universidade.

Escreveu diversas obras, mas muitas se perderam, inclusive o tratado Sobre a medida da

Terra. Eratóstenes morreu em Alexandria, em 194 a.C.

(http://www.moderna.com.br/moderna/didaticos/ef2/matematica/erato/bio_eratostenes.htm)

Teorema 7.8: Se um inteiro positivo a > 1 é composto, então a possui um divisor primo

p .

Demonstração:

Com efeito, se o inteiro positivo a > 1 é composto, então:

Portanto, supondo b c, teremos:

b

2 bc = a b

a = bc, com 1 < b < a e 1 < c < a

CAPÍTULO 7

NÚMEROS PRIMOS

87

O teorema 7.8 fornece um processo que permite reconhecer se um dado inteiro a >1 é primo

ou é composto, para o que basta dividir a sucessivamente pelos primos que não excedem o

valor . Tal resultado é a base do chamado Crivo de Eratóstenes que veremos em seguida.

Uma questão natural sobre os números primos é a de determinar, dentre os inteiros positivos,

todos os números primos até certo número dado. Esta questão também foi resolvida na

antiguidade por Eratóstenes. A ele devemos o chamado Crivo de Eratóstenes. Com o crivo de

Eratóstenes podem-se determinar, sem auxílio de máquinas, todos os números primos até 200,

400 ou 500, por exemplo. Com o auxílio de computadores, o crivo de Eratóstenes,

convenientemente adaptado, permite determinar os números primos até limites bem altos.

Mesmo antes dos computadores, já haviam sido determinados os números primos até

10.000.000. Isto ocorreu por volta de 1914, por obra do matemático americano D. N. Lehmer.

Dois outros matemáticos (Bays e Hudson) calcularam, em 1976, (usando computadores,

evidentemente!), a tabela dos números primos até 12 x 1011

. Além disso, há tabelas de

números primos em determinados intervalos de inteiros e conhecem-se também números

primos bem grandes, como o número 244497

– 1, que possui 13395 algarismos! (RPM 19)

A construção de uma tabela de números primos que não excedam um dado inteiro n usando o

Crivo de Eratóstenes consiste no seguinte: escrevem-se na ordem natural todos os inteiros a

partir de 2 até n e, em seguida, eliminam-se todos os inteiros compostos que são múltiplos dos

primos p tais que p isto é, 2p, 3p, 4p, ...

Exemplo: Construir a tabela de números primos menores que 200.

Solução: Como , basta eliminar sucessivamente da tabela os números que são

múltiplos dos primos p menores que 14, ou seja, 2, 3, 5, 7, 11 e 13.

. 2 3 4 5 6 7 8 9 10

11 12 13 14 15 16 17 18 19 20

21 22 23 24 25 26 27 28 29 30

31 32 33 34 35 36 37 38 39 40

41 42 43 44 45 46 47 48 49 50

51 52 53 54 55 56 57 58 59 60

61 62 63 64 65 66 67 68 69 70

71 72 73 74 75 76 77 78 79 80

81 82 83 84 85 86 87 88 89 90

91 92 93 94 95 96 97 98 99 100

101 102 103 104 105 106 107 108 109 110

111 112 113 114 115 116 117 118 119 120

121 122 123 124 125 126 127 128 129 130

131 132 133 134 135 136 137 138 139 140

CAPÍTULO 7

NÚMEROS PRIMOS

88

141 142 143 144 145 146 147 148 149 150

151 152 153 154 155 156 157 158 159 160

161 162 163 164 165 166 167 168 169 170

171 172 173 174 175 176 177 178 179 180

181 182 183 184 185 186 187 188 189 190

191 192 193 194 195 196 197 198 199 200

Os valores em vermelho são os números primos que não foram “riscados” da tabela.

Listamos a seguir a os 199 primeiros números primos:

2, 3, 5, 7, 11, 13, 17, 19, 23, 29, 31, 37, 41, 43, 47, 53, 59, 61, 67, 71, 73, 79, 83, 89, 97, 101,

103, 107, 109, 113, 127, 131, 137, 139, 149, 151, 157, 163, 167, 173, 179, 181, 191, 193, 197,

199, 211, 223, 227, 229, 233, 239, 241, 251, 257, 263, 269, 271, 277, 281, 283, 293, 307, 311,

313, 317, 331, 337, 347, 349, 353, 359, 367, 373, 379, 383, 389, 397, 401, 409, 419, 421, 431,

433, 439, 443, 449, 457, 461, 463, 467, 479, 487, 491, 499, 503, 509, 521, 523, 541, 547, 557,

563, 569, 571, 577, 587, 593, 599, 601, 607, 613, 617, 619, 631, 641, 643, 647, 653, 659, 661,

673, 677, 683, 691, 701, 709, 719, 727, 733, 739, 743, 751, 757, 761, 769, 773, 787, 797, 809,

811, 821, 823, 827, 829, 839, 853, 857, 859, 863, 877, 881, 883, 887, 907, 911, 919, 929, 937,

941, 947, 953, 967, 971, 977, 983, 991, 997, 1009, 1013, 1019, 1021, 1031, 1033, 1039, 1049,

1051, 1061, 1063, 1069, 1087, 1091, 1093, 1097, 1103, 1109, 1117, 1123, 1129, 1151, 1153,

1163, 1171, 1181, 1187, 1193, 1201, 1213 ,1217.

Nota: Podemos fazer um crivo mais econômico, já que não é possível evitar

completamente o fato de que alguns números são riscados várias vezes.

Podemos proceder da seguinte maneira: Primeiro escrevemos uma lista com

os ímpares de 3 até n. Como queremos riscar os números de p em p é claro

que os múltiplos de p que são múltiplos de primos menores que p já foram

riscados da lista. Então, nesta etapa, podemos começar a riscar de p em p a

partir do menor múltiplo de p, que não é múltiplo de um primo menor que p;

isto é, a partir de p2. Isto evita muitas duplicações.[Coutinho]

3, 5, 7, 32, 11, 13, 15, 17, 19, 21 23, 52, 27, 29, 31, 33, 35, 37, 39, 41,

43, 45, 47, 72, 51, 53, 55, 57, 59, 61, 63, 65, 67, 69, 71, 73, 75, 77, 79,

81, 83, 85, 87, 89, 91, 93, 95, 97, 99.

CAPÍTULO 7

NÚMEROS PRIMOS

89

Definição 7.2: Para qualquer número real x > 0, seja (x) o número de primos p x , isto é,

(x) é quantidade dos números primos menores que ou iguais a x.

Tabela dos 20 primeiros valores inteiros da Função p (x)

x (x)

1 0

2 1

3 2

4 2

5 3

6 3

7 4

8 4

9 4

10 4

11 5

12 5

13 6

14 6

15 6

16 6

17 7

18 7

19 8

20 8

De acordo com o Teorema de Chebychev podemos afirmar que

(2n) - (n) 1 (para n 2)

Nota: Um problema prático, onde as propriedades dos números primos têm

reflexos importantes, é o problema do reconhecimento da fala por

computadores que exige o desenvolvimento de algoritmos tão rápidos quanto

possível para a decomposição de sons nas suas frequências fundamentais,

uma técnica conhecida como Análise de Fourier. A velocidade teórica

máxima desses algoritmos esta diretamente relacionada com a função (x)

que fornece o numero de primos menores que ou iguais a x.

Fórmula de Minác: Dado um inteiro m 2 , J. Minác estabeleceu uma fórmula para a

contagem dos números primos (m) :

Demonstração: Será vista após estudarmos o Teorema De Wilson.

CAPÍTULO 7

NÚMEROS PRIMOS

90

Exemplo:

(6) = 1 + 1 + 0 + 1 + 0 = 3. Resultado que nos diz que existem três primos antes do número

seis.

Fórmula Para o n-ésimo Número Primo

Devido ao resultado acima podemos escrever uma fórmula que nos retorna o n-ésimo número

primo estabelecida por C. P. Willans em 1964:

Exemplo:

Definição 7.3: Para todo número primo p, seja p# o produto de todos os números primos q

p . p# é chamado o primorial de p.

Tabela dos 17 primeiros Primoriais

P

2

3

5

7

11

13

17

23

29

p#

2

6

30

210

2310

30030

510510

9699690

223092870

CAPÍTULO 7

NÚMEROS PRIMOS

91

Teorema 7.8: p# +1 não possui nenhum fator primo menor do que ou igual a p.

Demonstração: Suponhamos, por contradição, que p# + 1 seja divisível por um primo q p .

Ou seja, existe um inteiro positivo s tal que tal que p# + 1 = q.s, isto é q.s – p# =1. Como q

p , então q é necessariamente um fator de p#. Logo q divide ambas as parcelas da diferença

q.s – p#. Portanto q divide 1, o que é um absurdo uma vez que q é primo.

Nota: Veja que resultado interessante:

Leitura: A Distribuição dos Números Primos

Ao contemplar uma tabela de números primos, a primeira impressão que se tem é a de que

não há nenhuma ordem entre os números primos: às vezes eles aparecem próximos uns dos

outros, às vezes afastados, ora menos, ora mais afastados; enfim, analisando-os

individualmente ou em pequenos grupos, não divisamos qualquer regularidade em sua

distribuição. Entretanto, a sagacidade de inteligências privilegiadas consegue ver mais fundo,

e foi precisamente isso o que aconteceu por obra do matemático francês Adrien - Marie

Legendre (1752-1833). Ele se ocupou dessa questão e por volta de 1800 formulou uma

conjectura que revela certa ordem no que parecia ser um caos completo. Para explicarmos a

conjectura de Legendre, introduzimos o símbolo (x) como sendo o número de números

primos até certo valor x. Assim, (8) = 4, ou seja, o número de números primos até 8 é 4;

(11) = 5, pois há cinco números primos até 11, precisamente, 2, 3, 5, 7, 11; e assim por

diante. Pois bem, o que Legendre conjecturou, empiricamente, analisando tabelas de números

primos (em 1797 uma dessas tabelas foi publicada, contendo todos os números primos até

400031), é que (x) podia ser aproximado pela função (o logaritmo que aqui aparece é o

logaritmo natural, isto é, na base e 2,718281...), e que essa aproximação seria tanto melhor

quanto maior fosse x. Mas isto deve ser entendido em termos relativos, isto é, o erro que se

comete tomando em lugar de (x) torna-se tanto menor quanto maior for x, relativamente

a Em outras palavras, seja

31

37

41

43

47

53

59

61

6469693230

200560490130

7420738134810

304250263527210

13082761331670030

614889782588491410

32589158477190044730

1922760350154212639070

CAPÍTULO 7

NÚMEROS PRIMOS

92

o erro que se comete ao tomar em lugar. de (x). Pois bem, o que se torna pequeno com o

crescer de x é o erro relativo

Este erro pode ser feito, em valor absoluto, tão pequeno quanto quisermos, desde que façamos

x suficientemente grande.

Carl Friedrich Gauss (1777-1855), que é considerado por muitos o maior matemático de todos

os tempos, conta, numa carta de 1849, publicada vários anos mais tarde, que quando ainda

bem jovem, com apenas 15 anos de idade, pensou muito sobre a distribuição dos números

primos, chegando a conjecturar algo equivalente ao que conjecturou Legendre.

Seja como for, essa conjectura logo impressionou os matemáticos como algo notável, pois

quem diria que a seqüência dos números primos pudesse ter algo a ver com a função

logaritmo!

A descoberta de Legendre e Gauss demorou a ser demonstrada. Embora ela tenha sido objeto

da atenção dos melhores matemáticos do século, desafiou a argúcia desses homens por cerca

de 100 anos. De fato, foi somente em 1896 que ela foi demonstrada pela primeira vez. E nesse

mesmo ano apareceram duas demonstrações, uma pelo matemático francês Jacques Hadamard

(1865-1963) e outra, pelo belga Charles de Ia Vallée Poussin (1866-1962). Essas

demonstrações, independentes uma da outra, baseavam-se nas idéias de um outro grande

matemático do século, Bernhard Riemann (1826-1866). Embora não tenha logrado

demonstrar a conjectura de Legendre e Gauss, Riemann, num memorável trabalho intitulado

Sobre o número de números primos menores que um certo número, deixou ideias notáveis

sobre teoria dos números, que vêm sendo exploradas pelos estudiosos do assunto até os dias

de hoje.

Antes mesmo das demonstrações de Hadamard e de la Vallée Poussin, o matemático russo

Pafnutii Chebyshev (1821-1894) provou, por volta de 1850, um resultado próximo à

conjectura de Legendre e Gauss. Segundo Chebyshev, existem constantes positivas c e C (c

0,92, C 1, 106) tais que

Para bem entendermos o significado da aproximação

vamos comparar os gráficos das funções y = x e y = log x. Eles nos revelam que ambas as

funções crescem com o crescer de x, tendendo a infinito.

(4)

(3)

(2)

CAPÍTULO 7

NÚMEROS PRIMOS

93

No entanto, como podemos ver, claramente, a primeira dessas funções cresce mais depressa

que a segunda, distanciando-se mais e mais desta última, à medida que x cresce acima de

qualquer número dado. Isto fica mais claro ainda quando levamos em conta que o gráfico do

logaritmo tem a concavidade voltada para baixo, significando que, embora esta função esteja

crescendo sempre com o crescer de x, trata-se de um crescimento cada vez mais lento, quanto

maior for x. Isto quer dizer que o quociente no segundo membro de (4) também cresce,

tendendo a infinito com o crescer de x, o que está de acordo com o fato de que existem

infinitos números primos, isto é, (x) cresce acima de qualquer número, desde que façamos x

suficientemente grande. Não obstante tudo isso, o erro absoluto expresso em (1) pode tornar-

se muito grande, mas não o erro relativo expresso em (2); este tende a zero, isto é, pode ser

feito menor do que qualquer número positivo dado, desde que façamos x suficientemente

grande.

Uma conclusão simples que podemos tirar de (4) é que, em certo sentido, os números primos

vão ficando cada vez mais raros, à medida que avançamos na seqüência dos números naturais.

Para bem entender o que estamos dizendo, observe que significa

de sorte que é a densidade média dos números primos no intervalo que vai de 1 até x. O

fato de que essa densidade decresce com o crescer de x significa precisamente o que dissemos

acima: os números primos vão ficando cada vez mais raros, à medida que avançamos na

seqüência dos números naturais. ( RPM19)

Definição 7.3 Chamam-se primos gêmeos dois inteiros positivos ímpares e consecutivos que

são ambos primos. Em outras palavras, dizemos que dois primos ímpares são gêmos quando a

diferença entre eles é igual a 2.

Assim, por exemplo, são pares de primos gêmeos:

Não se sabe até hoje se há um número infinito de pares de primos gêmeos, mas são

conhecidos primos gêmeos muito grandes, tais como:

Um fato interessante é a existência de apenas um terno de inteiros positivos ímpares e

consecutivos que são todos primos: 3, 5 e 7.

140.737.488.353.507 e 140.737.488.353.509

140.737.488.353.699 e 140.737.488.353.701

3 e 5, 5 e 7, 11 e 13, 17 e 19, 29 e 31

CAPÍTULO 7

NÚMEROS PRIMOS

94

7.6. Seqüência de Inteiros Consecutivos Compostos

Existem, na sequência dos primos, primos consecutivos “tão afastados quanto se deseje”. Ou

seja, existem “saltos’ arbitrariamente grandes na seqüência dos primos.

Teorema 7.9: Dado um inteiro positivo n >1, é possível determinar n inteiros consecutivos

tais que nenhum deles seja primo.

Demonstração:

De fato, é evidente que na sequência:

os seus n termos são inteiros positivos consecutivos, e cada um deles é composto, porque (n

+1)! + j é divisível por j se 2 j n + 1.

Assim, por exemplo, supondo n = 4, obtemos a sequência:

5! + 2, 5! + 3, 5! + 4, 5! + 5

Cujos termos são 4 inteiros positivos consecutivos, cada um dos quais é composto, pois,

temos:

5! + 2 = 122 = 2 . 61, 5! + 3 = 123 = 3 . 41

5! + 4 = 124 = 4 . 31, 5! + 5 = 125 = 5 . 25

Outras sequências de 4 inteiros consecutivos e compostos existem, tais como

24, 25, 26, 27 e 32, 33, 34, 35

54, 55, 56, 57 e 74, 75, 76, 77

Nota: Em 1984 Samuel Yates iniciou uma lista dos "Maiores Primos

Conhecidos" e criou o nome primo titânico para designar qualquer número

primo com 1.000 ou mais dígitos decimais. Denominou também de titãs

aqueles que provaram a sua primalidade.

A maioria dos primos são titânicos e dezenas de milhares deles são

"conhecidos". Entretanto, na época em que Yates definiu os primos titânicos,

tinha-se conhecimento de apenas alguns poucos.

Cerca de dez anos mais tarde, Yates designou como primo gigante todo

número primo que possuísse 10.000 ou mais dígitos decimais. E os

Megaprimos são números primos que possuam no mínimo um milhão de

dígitos decimais.

http://www.numaboa.com.br/criptologia/matematica/primos.php

(n + 1)! + 2, (n + 1)! +3, (n + 1)! + 4, ..., (n + 1)! + (n + 1)

CAPÍTULO 7

NÚMEROS PRIMOS

95

Corolário 7.7: Dado um inteiro positivo n, existem dois primos consecutivos ph, ph+1 tais que

Demonstração:

Seja ph o maior dos primos que são menores que ( n +1 )! + 2.

Então, ph (n + 1)!+ 1. Do teorema anterior, temos ainda que

Fazendo a diferença entre ambas as desigualdades, temos

Exemplo: Seja n = 6, de acordo com a demonstração podemos considerar os primos p1 =

5039 e p2 = 5059. Assim, 5059 – 5039 > 6, isto é, 20 > 6.

Teorema 7.10: O produto de qualquer sequência de k inteiros consecutivos é divisível por k!.

Demonstração:

Vamos considerar n e k inteiros positivos com k n. Sabemos que o número de combinações

de n, tomadas k a k, é um inteiro dado por:

Sendo o numerador o produto de k inteiros consecutivos temos o resultado para uma

sequência de k inteiros positivos. No caso de zero ser um elemento na seqüência o resultado é

trivial, uma vez que zero é divisível por qualquer inteiro não nulo.

Se a sequência contiver só números negativos, a fração do lado direito da igualdade acima

sofrerá, no máximo, uma mudança de sinal continuando a ser um inteiro, o que conclui a

demonstração.

ph +1 > (n + 1)! + (n + 1)

ph +1 > (n + 1)! + (n + 1)

ph+1 – ph > n.

CAPÍTULO 7

NÚMEROS PRIMOS

96

7.7 . Conjecturas

● Conjectura de Goldbach.

Em 1742, numa carta a Leonhard Euler (1707-1783), Christian Goldbach (1690-1764)

expressou a seguinte conjectura:

Todo inteiro n > 5 é a soma de três números primos.

Em resposta, Leonhard Euler observou que essa conjectura era equivalente à seguinte:

Todo inteiro par maior que ou igual a 4 é a soma de dois primos.

Esta conjectura é conhecida como conjectura de Goldbach. Um romance interessantíssimo

sobre a dificuldade desse assunto é “Tio Petros e a Conjectura de Goldbach” escrito por

Apostolos Doxiadis e publicado pela Editora 34.

Exemplos:

4=2+2

6=3+3

8=3+5

10=3+7, 5+5

12=5+7

14=3+11, 7+7

16=3+13, 5+11

18=5+13, 7+11

20=3+17, 7+13

22=3+19, 5+17, 11+11

24=5+19, 7+17, 11+13

26=3+23, 7+19, 13+13

28=5+23, 11+17

30=7+23, 11+19, 13+17

32=3+29, 13+19

34=3+31, 5+29, 11+23, 17+17

36=5+31, 7+29, 13+23, 17+19

38=7+31, 19+19

40=3+37, 11+29, 17+23

Muitos matemáticos continuam tentando encontrar um contra-exemplo ou uma demonstração

para essa conjectura. Por exemplo:

• Georg Cantor (1845-1918), efetuou em 1894 todas as decomposições possíveis, como

soma de dois números primos, de todos os números pares inferiores a 1000.

• Aubry estendeu a lista de Cantor até 2000.

CAPÍTULO 7

NÚMEROS PRIMOS

97

• R. Haussner em 1897 estendeu essa tabela até 5000.

• Em 1937 o matemático soviético I.M.Vinogradov demonstrou, usando somas

trigonométricas adequadas, que qualquer número ímpar suficientemente grande é soma de

três números primos.

• Em 1966 o matemático chinês Jeng-Run Chen provou que a partir de algum número n,

todo par maior que 2 ou é soma de dois primos, ou a soma de um primo com o produto de

dois primos. O argumento de Chen não diz qual é esse n; apenas demonstra que ele existe.

Além da Conjectura de Goldbach, em Teoria dos Números, particularmente em Números

Primos, existem muitos problemas em aberto. Segue uma lista com algumas conjecturas que,

embora já tenham sido testadas para inúmeros casos, ainda não foram demonstradas. Eis

algumas:

• Todo número ímpar maior que cinco é a soma de três primos. Esse fato já foi provado,

por Vinogradov, para números suficientemente grandes. Em 1956, Borodzkin mostrou que

n > 314348907

é suficiente. Esse número foi diminuído, em 1989, para 1043000, por Chen e

Wang, mas ainda é muito grande para que os casos menores possam ser testados com o

uso de um computador.

Exemplos:

7 = 3 + 2 + 2; 21 = 11 +7 + 3 ; 41 = 11 + 13 + 17; 49 = 13 + 17 + 19

• Existem infinitos primos da forma k2 + 1.

Exemplos:

5 = 22 +1; 17 = 4

2 +1; 3

7= 6

2 + 1.

• Existem infinitos pares de primos consecutivos (Primos Gêmeos) .

Exemplos: (3 e 5), (5 e 7), (11 e 13), (17 e 19), (29.879 e 29.881), ...

Em 2000, foi apresentado um par de primos gêmeos cada um com 18075 dígitos. É o par

• Existe sempre um primo entre dois quadrados consecutivos. Exemplos: 3 entre 1 e 4; 5 e 7 entre 4 e 9; 11 e 13 entre 9 e 16, ....

• Primos de Sophie Germain. Um número primo p é um número primo de Sophie

Germain se 2p + 1 é também primo. São famosos porque Sophie Germain provou que o

Último Teorema de Fermat é verdadeiro para estes números. A existência de um número

infinito de tais números primos é uma uma afirmação ainda não provada. Os primeiros

primos de Sophie Germain são 2, 3, 5, 11, 23, 29, 41, 53, 83, 89, 113, 131, 173, 179, 191,

233 ...

4 648 619 711 505. 260000

± 1

CAPÍTULO 7

NÚMEROS PRIMOS

98

Nota: Primos em Progressão Aritmética.

Um problema famoso que permaneceu por muito tempo em aberto, era o de

provar se existiam progressões aritméticas arbitrariamente longas formadas

exclusivamente por primos. Van der Corput já havia provado em 1939 que há

uma infinidade de progressões aritméticas formadas por 3 primos. Ben Green

do Instituto de Matemática de Vancouver e Terence Tao da Universidade da

Califórnia, provaram em 2006, que tais sequências existem. Mas a prova não

especifica como encontrá-las ou entre quais primos tais sequências se

encontram.

A mais longa progressão aritmética de números primos conhecida até o

momento, tem 24 termos. Foi descoberta por Jaroslaw Wroblewski em

janeiro de 2007:

468395662504823 + 45872132836530.k, para k = 0, 1, ..., 23.

7.8. Fórmulas que geram alguns números primos

Muitas tentativas têm sido realizadas para encontrar fórmulas aritméticas simples que

forneçam somente primos. Nesta seção será apresentada algumas fórmulas famosas sobre

primos.

1) Fórmula de Fermat:

Fermat fez sua famosa conjectura de que os números da forma

são primos.

Para n = 1, 2, 3, 4 obtemos:

todos primos. Porém em 1732, Euler descobriu a fatoração

portanto, F(5) não é primo. Até este momento (05 /2005) o maior primo de Fermat

conhecido é F4

+ 1 = 4294967297 = (641).(6700417)

F1 = 22 + 1 = 5

F2 = + 1 = 24 + 1 = 17

F3 = + 1 = 28 + 1 = 257

F4 = + 1 = 216

+ 1 = 65.537

CAPÍTULO 7

NÚMEROS PRIMOS

99

2) Fórmula de Euler:

Em 1772 Leonhard Euler descobriu um polinômio tendo uma longa sucessão de valores

primos, dado por

que fornece primos para n = 1, 2, ..., 39. Entretanto, para n = 40 o valor é composto:

3) Fórmula de Mersenne:

Marin Mersenne em 1644 fez a seguinte afirmação: “Todo natural Mp = 2p – 1 é primo

para os primos p = 2, 3, 5, 7, 13, 17, 19, 31, 67, 127 e 257, e é composto para todos os

outros primos p < 257”.

Entretanto, esta afirmação é incorreta, pois, segundo o site http://www.mersenne.org/

prime.htm, até setembro de 2006 já eram conhecidos, 44 primos de Mersene, para os primos p

= 2, 3, 5, 7, 13, 17, 19, 31, 61, 89, 107, 127, 521, 607, 1279, 2203, 2281, 3217, 4253, 4423,

9689, 9941, 1213 ,19937, 21701, 23209, 44497, 86243, 110503, 132049, 216091, 756839,

859433, 1257787, 1398269, 2976221, 3021377, 6972593, 13466917, 20996011, 24036583,

30402457 e 32582657. Esse último

primo tem 9.808.358 dígitos.

Como se pode ver, Mersenne cometeu duas falhas: Incluiu p= 67, 257 na sua lista de primos e

excluiu dessa lista p= 61, 89, 107.

Somente em 1947 ( mais de 300 anos depois) a lista correta p = 2, 3, 5, 7, 13, 17, 19, 31, 61,

89, 107 e 127 onde p < 257, ficou pronta..

4) Outras fórmulas que geram alguns primos são:

Cabe agora a pergunta: Existe algum polinômio (não-constante), com coeficientes inteiros,

que forneça a sequência dos números primos ou apenas números primos? Infelizmente a

resposta é não!

Teorema 7.11: Não existe polinômio algum P(x) = anxn + an-1x

x-1+ ... + a0, a0 0 com

coeficientes ak, 0 k n , todos inteiros, cujos valores numéricos sejam sempre primos para

valores inteiros da variável x. ( VER RPM 45)

F(n) = n2 - n + 41para n = 1, 2, 3, 4, ..., 40

F(n) = n2 - 79n + 1601para n = 0, 1, 2, ..., 79

F(n) = n2 + n + 17 para n = 0, 1, 2, ..., 15

F(n) = 3n2 + 3n + 23para n = 0, 1, 2, ..., 21

F(n) = 6n2 + 6n + 31para n = 0, 1, 2, ..., 28

F(40) = 402 + 40 + 41 = 40. (40 + 1) + 41 = 40.41 + 41 = 41.(40 + 1) = 41.41.

F(n) = n2 + n + 41

CAPÍTULO 7

NÚMEROS PRIMOS

100

Demonstração: Suponhamos, por contradição, que o polinômio P(x), nas condições do

teorema, produz sempre primos para valores inteiros da variável x. Então, para x = j, sendo j

um inteiro fixo, P( j ) = p é um primo, e qualquer que seja o inteiro s, temos:

Desenvolvendo cada uma das potências pela fórmula do binômio e agrupando os primeiros

termos de cada desenvolvimento, temos:

onde g(s) indica um certo polinômio não constante em s com coeficientes inteiros, de grau

n, logo:

Então, p| P(j + ps). Se P(j + ps) é primo devemos ter P( j + ps) = ± p, donde 1+ g(s) = ± 1,

para todo s. Temos uma contradição, pois g(s) não é constante.

Nota: O teorema anterior refere-se a polinômios numa variável. Os trabalho

de Putnam, Davis, Robison e Matijasevic conduziram a uma surpreendente

conclusão: Existe um polinômio de coeficientes inteiros, tal que o conjunto

dos números primos coincide com o conjunto dos valores positivos

assumidos por esse polinômio, quando as variáveis percorrem o conjunto

dos inteiros positivos.

Jones, Sato, Wada e Wiens (1976) foram os primeiros a escrever,

explicitamente, um polinômio desse tipo, de grau 25 e com 26 varáveis. [

Ribombim ]

Leitura: Uma Fórmula que Fornece todos os Números Primos

Sejam x e y números naturais, y 0 e a = x( y + 1) - ( y!+ 1).

A fórmula que dá todos os números primos e somente esses é:

f(x, y) =

Por exemplo:

Se x = 1 e y = 1, então a = 0 e f(1,1) = 2;

Se x = 1 e y = 2, então a = 0 e f(1,2) = 3;

Se x = 1 e y = 3. então a = –3 e f(1,3) = 2;

e, atribuindo-se a x e a y mais alguns valores, percebe-se logo que a função f tem uma

predileção muito grande pelo número primo 2. Mas ela fornece todos os números primos:

P(j + ps) = p(1 + g(s))

P(j + ps) = (an jn + an-1 j

n-1 + ... + a2j

2 +a1j + a0) + pg(s) = P(j) + pg(s) = p + pg(s)

P(j + ps) = an (j + ps)n + na-1 (j + ps)

n-1 + ... + a2 (j +ps)

2 +a1 (j + ps) + a0

CAPÍTULO 7

NÚMEROS PRIMOS

101

Como foram achados os pares (x,y) acima? A resposta é simples: para obter o número primo

p, calcule f(x,y) para

Assim, para obter 13, fizemos

Como se vê, a fórmula existe, mas não é nada prática, uma vez que envolve cálculos com

números muito grandes(RPM 37).

A demonstração dessa fórmula será vista após estudarmos o Teorema de Wilson.

7.9. Decomposição do Fatorial em Fatores Primos

Mostraremos como achar a fatoração em números primos de n! onde n é um número natural

arbitrário.

Proposição 7.2: Sejam a 0 e b, c > 0 . Temos que

Demonstração: Sejam.

Logo,

e

portanto,

como

br2 + r1 b(c - 1) + b - 1 = bc - 1

a = bq1 + r1 = b(cq2 + r2) + r1 = bcq2 + br2 + r1

a = bq1 + r1 , com r1 b – 1

5 = f (5,4); 7 = f (103,6); 11= f (329891,10); 13 = f (36846377, 12); ....

CAPÍTULO 7

NÚMEROS PRIMOS

102

segue-se que é o quociente da divisão de a por bc, ou seja,

Dados um número primo p e um número natural m, vamos definir por Ep (m) o expoente da

maior potência de p que divide m, ou seja, é o expoente da potência de p que aparece na

fatoração de m em fatores primos.

Em particular, Ep (n!) representará a potência de p que aparece na fatoração de n! em fatores

primos.

Teorema de Legendre. Sejam m um número natural e p um número primo. Então

Demonstração: Note, inicialmente, que a soma acima é finita, pois existe um número natural

r tal que pi > n para todo i > r portanto , se i r

Vamos demonstrar o resultado por indução sobre n . A fórmula vale trivialmente para n = 0.

Suponha que o resultado vale para qualquer natural m com m < n Sabemos que os múltiplos

de p entre 1 e n são:

Portanto, pela hipótese de indução, temos que

O resultado, agora, decorre da preposição 7.2.

Para calcular Ep (n!) faz-se uso do seguinte algoritmo:

Como q1 > q2 > ..., seguem-se que, para alguns s, tem-se que. Portanto, seguem-se que.

E (n!) = q1 + q2 + ... + qs

n = pq1 + r1

q1 = pq2 + r2

.....

qs-1 = pqs + rs

p, 2 p, ...,

Ep (n!) =

q2 =

CAPÍTULO 7

NÚMEROS PRIMOS

103

Exemplo: Vamos determinar a decomposição de 10! Em fatores primos.

Para resolvermos o problema, devemos achar Ep (10!) para todo primo p 10. Sendo

Seguem-se que

10! = 283

45

27 .

Lema 7.1. Sejam a1,..., am ,b inteiros positivos. Tem–se que

Demonstração: Sejam qi e ri respectivamente o quociente e o resto da divisão de ai por b para

i = 1, ...., m. somando, membro a membro, as igualdade ai = bqi + ri temos que

Segue–se dai que o quociente da divisão de a1 + ... + am por b é maior ou igual do que q1 + ...

+ qm pois r1 + ... + rm poderia superar b – 1 . Isto é o que se queria provar.

Corolário 7.8. Se a1,..., am,b são números naturais com b 0 , então é natural o número

Demonstração: De fato, pelo Lema 7.1, para todo número primo P e todo número natural

i, temos que

Somando, membro a membro, as desigualdades acima, obtemos que

Ep ((a1 + ... + am)!) Ep (a1 !) + ... + Ep (am)

a1 + ... + am = (q1 + ... + qm) b + r1 + ... + rm

CAPÍTULO 7

NÚMEROS PRIMOS

104

O que prova o resultado.

O próximo resultado relacionará Ep ( n!) e a representação p-ádica de n (i.e., a representação

relativa à base p)

Teorema 7.12. Sejam p,n inteiros positivos, com p primo. Suponha que.

Seja a representação p – ádica de n . Então.

Demonstração: Sendo 0 ni p , temos que

Portanto,

Exemplo: Seja determinar a potência de 3 na decomposição de 53! em fatores primos.

Primeiramente escrevemos 53 na base 3, isto é:

53 = (1222)3

Aplicando o Teorema 7.12

n = nrpr + nr-1p

r-1 + ... + n1p + no

CAPÍTULO 7

NÚMEROS PRIMOS

105

Verificando esse resultado pelo Teorema de Lagrange:

7.10. Método da Fatoração de Fermat

Até o momento, um dos procedimentos matemáticos mais difíceis é o de fatorar um número

arbitrariamente grande e isso às vezes requer um tempo razoável. Para os casos mais simples

podemos usar os conhecidos testes de divisibilidade, mas fatorar números grandes é objeto de

intensas pesquisas matemáticas. Damos uma aqui um uma ideia desse difícil problema

matemático, utilizando o chamado método da Fatoração de Fermat. Em cursos mais

avançados outros métodos são apresentados.

Proposição 7.3: Seja n > 1 um inteiro ímpar. Há uma correspondência biunívoca entre a

fatoração de n e a representação de n como diferença de dois quadrados.

Demonstração:

Se n = a.b, e n ímpar, então a e b são ímpares. Logo a+b e a-b são pares, então e

são inteiros.

Então,

Expressa n como a diferença de dois quadrados.

Reciprocamente, suponha n escrito como a diferença de dois quadrados:

n = s2 – t

2, então n = (s-t) . (s+t) é a forma fatorada de n.

Você pode ver que esses dois procedimentos – da fatoração para a diferença e da diferença

para a fatoração – determinam uma relação biunívoca.

7. 11 – Algoritmo de Fermat

A proposição acima nos permite descrever um algoritmo, que é muito eficiente quando n tem

um fator primo que não é muito menor que .

Para começar vamos supor que n é ímpar, já que se n for par então 2 é um de seus fatores. A

idéia do algoritmo de Fermat é tentar achar números inteiros positivos x e y tais que n = x2 - y

2

. Supondo que encontramos estes números, temos que

n = x

2 - y

2 = (x - y) (x + y).

CAPÍTULO 7

NÚMEROS PRIMOS

106

Logo x - y e x + y são fatores de n.

O caso mais fácil do algoritmo de Fermat ocorre quando n é um quadrado perfeito; isto é,

quando existe algum inteiro r tal que n = r2. Neste caso temos que r é fator de n. Além disso,

na notação acima x = r e y = 0. Observe que se y > 0 então

Isto sugere a seguinte estratégia para encontrar x e y.

Entrada: inteiro positivo ímpar n.

Saída: um fator de n ou uma mensagem indicando que n é primo.

Etapa 1: Comece com ; se n = x2 então x é fator de n e podemos parar.

Etapa 2. Caso contrário incremente x de uma unidade e calcule .

Etapa 3. Repita a Etapa 2 até encontrar um valor inteiro para y, ou até que x seja igual a :

no primeiro caso n tem fatores x+y e x-y, no segundo n é primo.

Exemplo: Seja n = 1342127 o número obtido como produto de dois primos. A variável x é

inicializada com a menor parte inteira da raiz quadrada de n . Mas x2

= 11582 = 1340964 < 1342127 logo passamos a incrementar x de um em um. Fazemos isso

até que seja inteiro, ou x seja igual a , que neste caso valeria 671064. É mais

fácil resumir isto em uma tabela

x

1159 33,97

1160 58,93

1161 76,11

1162 90,09

1163 102,18

1164 113

Obtivemos assim um inteiro no sexto laço. Portanto x = 1164 e y = 113 são os valores

desejados. Os fatores correspondentes são x + y = 1277 e x – y = 1051. Logo, 1051 e 1277

são os dois números primos procurados.

CAPÍTULO 7

NÚMEROS PRIMOS

107

EXERCÍCIOS

1. Com uma calculadora, achar todos os primos

da forma n2 – 2, para 25 35n

2. Determine todos os primos que são iguais a

diferença de quadrado entre dois primos.

3. De quantos modos podem escrever 497 como a

soma de dois números primos?

4. Mostrar que a soma de dois inteiros positivos

ímpares e consecutivos nunca é um primo.

5. Em um quadro estão escritos alguns números

naturais. Dentre eles, há nove múltiplos de 4,

sete múltiplos de 6, cinco múltiplos de 12,

três números primos e nada mais. Qual a

quantidade mínima de números escritos?

6. Achar todos os primos p e q, tais que p – q =

3.

7. Achar todos os primos que são iguais a um

quadrado menos 1.

8. Achar todos os primos que são iguais a um

cubo menos 1.

9. Escreva os números 55, 83 e 211 como uma

soma de três primos.

10. Determinar todos os inteiros positivos n tais

que n, n + 2 e n + 4 são todos primos.

11. Determinar todos os primos p tais que 3p + 1 é

um quadrado.

12. Com uma calculadora, determinar se são

primos os números

a) 1699

b) 7429

c) 21793

d) 1189

13. Encontre todos os primos p, tais que 17p + 1 é

um quadrado.

14. Usando a decomposição em fatores primos dos

inteiros 507 e 1287, achar o mdc (507, 1287) e

o mmc (507, 1287).

15. Achar o mdc(a, b) e mmc(a, b) sabendo a =

230 . 521 . 19 . 233 e b = 26 . 3 . 74 . 112 .

195 . 237

16. Achar o menor inteiro positivo pelo qual se

deve dividir 15! para se obter um quadrado.

Qual o menor valor do número natural n

que torna n! divisível por 1000?

17. Achar todos os primos que são divisores de

50!.

18. Verifique com uma calculadora, se são primos

gêmeos:

a) 1949 e 1951

b) 1997 e 1999

19. Achar uma sequência de quatro inteiros

positivos consecutivos e compostos.

20. Achar um sequência de 100 inteiros positivos

consecutivos e compostos.

21. Mostre que nenhum número inteiro da forma

1 4n é divisível pelo número primo 3.

22. Com uma calculadora, verificar a conjectura de

Goldbach para n par, 42 n 100 .

23. Determinar o menor valor positivo do inteiro n

tal que 2n2 + p, seja um número inteiro

composto e p um primo terminado em 7.

24. Demonstrar que todo primo, p 5 é da forma

6k – 1 ou 6k + 1, onde k é um inteiro positivo.

25. Demonstrar que todo primo p 3, é da forma

4k + 1 ou 4k – 1, onde k é um inteiro positivo.

26. Determine todos os primos p 5tais que 8p4 -

3003 também seja primo.

27. Mostrar que todo inteiro da forma n4 + 4, com

n > 1 não é primo.

28. Mostrar que todo inteiro da forma 8n + 1, com

n > 1, não é primo.

29. Mostrar que se n2 + 2 é primo então 3 | n, para

todo n > 1.

CAPÍTULO 7

NÚMEROS PRIMOS

108

30. Se p > 5 é um primo, então p2 + 2 é composto.

31. Demonstrar as seguintes propriedades:

a) Todo primo da forma 3n + 1 é também da

forma 6m + 1.

b) Todo inteiro n > 11 pode ser expresso

como a soma de dois inteiros não-primos.

c) Se p 5 é um primo ímpar, então p2 – 1 ou

p2 + 1 é divisível por 10.

d) Se p > q > 5 e se p e q são ambos primos,

então 24 | p2 – q

2.

e) Todo inteiro da forma 3n + 2 tem um fator

primo desta forma.

f) Se p é um primo e se p | an , então p

n | a

n.

32. Demonstrar que o inteiro positivo a > 1 é um

quadrado se e somente se todos os expoentes

dos fatores primos da sua decomposição são

inteiros pares.

33. Demonstrar que, se o inteiro k 2, não é

primo, então 2k – 1 nunca será primo.

34. Demonstrar que se 2k – 1, (k 2) é primo,

então k também é primo.

35. Seja p o maior fator primo do número 314

+ 313

– 12, então p é igual a:

36. Sejam p, q inteiros positivos. Mostre que 2p + 1

= q2 implica p e q primos e p = q = 3.

37. Mostrar que um inteiro da forma 42n+1

+ 1,

onde n 1, nunca é primo.

38. Sendo n um inteiro positivo, mostre que 24(n+1)

– 1 nunca será primo.

39. Mostrar que se n > 4, não é primo, então n

divide (n – 1)!.

40. Verificar que todo inteiro pode escrever-se sob

a forma 2k m, onde o inteiro k > 0 e m é um

inteiro ímpar.

41. Demonstrar que, se o inteiro n > 2, então existe

um primo p tal que n < p < n!.

42. Qual é o menor número primo que um fator da

soma 19992002

+ 20012002

?

43. Prove que um triângulo retângulo não pode

apresentar as medidas de seus lados sendo

números primos.

44. Se p e 8p2 + 1 são números primos, prove que

p = 3.

45. Mostre que se n 1 é natural então, o número 2 122

n

não é primo.

46. Sendo n > 1 um inteiro, prove que 4n + n

4 não

é primo.

47. Mostrar, mediante um exemplo, que a seguinte

conjectura é falsa:

“Todo inteiro positivo maior que 1, pode-se

escrever sob a forma a2 + p, com a > 0 e p é um

inteiro primo ou 1”.

48. Determine todos os números primos p e q, para

os quais os q números p, p + (q + 1), p + 2 (q

+ 1), p + 3 (q + 1), . . . , p + (q − 1) (q + 1),

também são primos.

49. Demonstrar que existem infinitos primos da

forma 4n + 3, com n inteiro positivo.

50. Seja m um intero positivo. Demonstre que não

existem números primos da forma 25m

+ 2m +1.

51. Determinar o número inteiro positivo n que

que é produto dos primos p, q e r, sabendo que

r - q = 2p e rq + p2 = 676.

52. Mostre que existem infinitos valores primos p

para os quais 8.p2 + 5 é divisível por 77.

53. Seja p > 2 um primo. Determine todos os

valores inteiros positivos de m e n, tal que

(p – 1) (pn + 1) = 4m (m + 1).

Nos problemas que se seguem faça uso de uma

calculadora para verificar os resultados e

explicite bem os passos utilizados na resolução.

54. Segundo o Teorema de Chebychev, para um

inteiro m 2, existe um primo p tal que m < p

< 2 m. Determine todos os primos entre 600 e

1200.

55. Segundo o Teorema de Dirichlet, se o mdc (a,

b) =1, então existem infinitos primos da forma

an+b com n um inteiro positivo. Determine

todos os primos p da forma 4n+9, com

88 < 4n + 9 < 388.

56. Usando o Teorema de Sierpinski, determine

um primo p>19 e escreva 20 inteiros

compostos.

57. Usando a Fórmula de Minàc, determine

(12).

CAPÍTULO 7

NÚMEROS PRIMOS

109

58. Usando a Fórmula do n-ésimo número primo,

determine o quarto número primo.

59. Calcule:

a) 3#.5# 7 #

11#

b) 5#.7# 11#

13#

60. Verifique se existem primos gêmeos entre 600

e 700.

61. Determine dois números primos consecutivos

tais que a diferença entre eles seja maior que 7.

62. Decomponha 98! Em fatores primos.

63. Determine a potência de 5 na decomposição de

75! em fatores primos, fazendo a

decomposição p-ádica de 75.

64. Com quantos zeros termina o número 1000! ?

Qual é a potência de 3 que aparece na

decomposição de 1000! em fatores primos?

65. Justifique se o número 93.94. ... .112.113

21!

é inteiro. Em caso afirmativo, calcule o seu

valor.

66. Encontrar o maior valor do inteiro 0n tal

que 10200!

504n seja inteiro.

67. Utilizando o Teorema do Número Primo:

a) Faça uma estimativa (sem muito rigor) de

quantos primos de 200 dígitos existem.

b) Mostre que entre os números de k-dígitos, um

em cada 2, 3k é primo.

68. Qual o menor valor do número natural n que

torna n! divisível por 1000?

110

Capítulo 8:

EQUAÇÕES DIOFANTINAS

LINEARES

UM POUCO DE HISTÓRIA SOBRE DIOFANTO

Diofanto tem o seu nome ligado à cidade que foi o maior centro de atividade matemática na

Grécia antiga. Pouco se sabe acerca da sua vida, o desconhecimento impede-nos mesmo de

fixar com segurança em que século viveu. Têm sido sugeridas datas distanciadas de um

século, antes ou depois do ano 250 d. C. Por uns versos encontrados no seu túmulo, escritos

em forma de um enigmático problema, deduz-se que viveu 84 anos. Positivamente, tal

problema não deve ser tomado como o paradigma dos problemas sobre os quais se interessou

Diofanto, pois ele pouca atenção deu a equações do 1º grau. Alexandria foi sempre um centro

muito cosmopolita e a matemática que se originou nela não era toda do mesmo tipo. Os

resultados de Heron eram bem diferentes dos de Euclides ou dos de Apolonios ou dos de

Arquimedes, e na obra de Diofanto há novamente uma quebra abrupta da tradição clássica

grega. Sabido é que os gregos, na época clássica, dividiram a aritmética em dois ramos: a

aritmética propriamente dita como "teoria dos números naturais". Frequentemente, tinha mais

em comum com a filosofia platônica e pitagórica do que com o que habitualmente se

considera como matemática, e logística ou cálculo prático que estabelecida as regras práticas

de cálculo que eram úteis à Astronomia, à Mecânica, etc. O principal tratado de Diofanto

conhecido, e que. ao que parece, só em parte chegou até nós, é a "Aritmética". Apenas seis

dos livros originais em grego sobreviveram, o número total (13) não passa de uma conjectura.

Era um tratado caracterizado por um alto grau de habilidade matemática e de engenho, pelo

que pode ser comparado aos grandes clássicos da "Primeira idade Alexandrina", ou seja, da

"época de ouro" da matemática grega, no entanto, quase nada têm em comum com esses ou,

na verdade, com qualquer matemática grega tradicional. Representa essencialmente um novo

ramo e usa um método diferente, dai a época em que possivelmente Diofanto viveu se chamar

"segunda idade Alexandrina", conhecida por sua vez por "época de prata" da matemática

grega. Diofanto, mais que um cultor da aritmética, e, sobretudo da geometria, como o foram

os matemáticos gregos anteriores, deve considerar-se um precursor da álgebra, e, em certo

sentido, mais vinculado com a matemática dos povos orientais (Babilônia, Índia,...) que com a

dos gregos. A sua "Aritmética” assemelha-se à álgebra babilônica em muitos aspectos, mas

enquanto os matemáticos babilônicos se ocupavam principalmente com soluções

“aproximadas" de equações "determinadas" e, sobretudo de equações "indeterminadas" do 2º

e do 3º graus das formas canônicas, em notação atual, Ax2+Bx+C = y2 e

Ax3+Bx2+Cx+D=y2, ou conjuntos (sistemas) destas equações. É exatamente, por esta razão

em homenagem a Diofanto -que a esta "Análise indeterminada" se chama “Análise

diofantina” ou “Análise diofântica". No desenvolvimento histórico da álgebra considera-se,

em geral, que podem ser reconhecidos três estádios: o primitivo ou retórico, em que tudo era

completamente escrito em palavras, um intermédio ou sincopado, em que foram adaptadas

algumas abreviaturas e convenções, e um final ou simbólico, em que são usados somente

símbolos. A "Aritmética" de Diofanto deve ser colocada no segundo estádio; nos seus seis

livros há um uso sistemático de abreviaturas para potências de números e para relações e

operações.

CAPÍTULO 8

EQUAÇÕES DIOFANTINAS LINEARES

111

3.1. Generalidades

A teoria das Equações Diofantinas é o ramo da Teoria dos Números que investiga as soluções

inteiras de equações polinomiais, como por exemplo:

x2 + y

2 = z

2, possui infinitas soluções representadas pelas ternas ordenadas (x,y,z)

conhecidas como ternos pitagóricos.

xn + y

n = z

n, que não possui soluções não nulas para para n > 2, e é conhecida como o

Último Teorema de Fermat.

y2 = x

3 + 17, que é válida, por exemplo, para os seguintes valores positivos: (2,5) ;

(4,9); (8,23); (43, 282); (52, 375); ...

Equação de Pell : x dy m2 2 , onde d um inteiro positivo que não seja um

quadrado e m é um inteiro qualquer.

Etc...

O tipo mais simples de equação diofantina é a equação diofantina linear com duas incógnitas

x e y:

onde a, b e c são inteiros dados, sendo ab 0 .

Todo par de inteiros 0x ,

0y tais que a0x + b

0y = c diz-se uma solução inteira ou apenas uma

solução da equação ax + by = c.

Consideremos, por exemplo, a equação diofantina linear com duas incógnitas:

Temos:

Logo, os pares de inteiros:

são soluções da equação 3x + 6y = 18

Existem equações diofantinas lineares com duas incógnitas que não têm solução. Assim, por

exemplo, a equação diofantina linear:

2x + 4y = 7

4 e 1, -6 e 6, 10 e -2

3.4 + 6.1 = 18

3(-6) + 6.6 = 18

3.10 + 6(-2) = 18

3x + 6y = 18

ax + by = c

CAPÍTULO 8

EQUAÇÕES DIOFANTINAS LINEARES

112

não tem solução, porque 2x + 4y é um inteiro par quaisquer que sejam os valores inteiros de x

e y, enquanto que 7 é um inteiro ímpar (observe-se que 2 = mdc (2, 4) não divide 7).

De modo geral, a equação diofantina linear ax + by = c não tem solução todas as vezes que d

= mdc (a, b) não divide c, como é óbvio.

3.2. Condição de Existência de Solução

Teorema 3.1: A equação diofantina linear ax + by = c tem solução se e somente se d divide c,

sendo d = mdc (a, b).

Demonstração:

Suponhamos que a equação ax + by = c tem uma solução, isto é, que existe um par de

inteiros 0x ,

0y tais que a0x + b

0y = c.

Por ser o mdc (a, b) = d, existem inteiros r e s tais que a = dr e b = ds, e temos:

e como r0x + s

0y é um inteiro, segue-se que d divide c ( d | c ).

Reciprocamente, suponhamos que d divide c ( d | c ), isto é, que c = dt, onde t é um inteiro.

Por ser o mdc (a, b) = d, existem inteiros 0x e

0y tais que

o que implica:

isto é, o par de inteiros:

é uma solução da equação ax + by = c.

x = t0x = (c/d)

0x , y = t0y = (c/d)

0y

c = dt = (a0x + b

0y )t = a(t0x ) + b(t

0y )

d = a0x + b

0y

c = a0x + b

0y = dr0x + ds

0y = d(r0x + s

0y )

CAPÍTULO 8

EQUAÇÕES DIOFANTINAS LINEARES

113

3.3. Soluções da equação ax + by = c.

Teorema 3.2: Se d divide c ( d | c ), sendo d = mdc (a, b), e se o par de inteiros 0x ,

0y é uma

solução particular da equação diofantina linear ax + by = c, então todas as outras soluções

desta equação são dadas pelas fórmulas:

onde t é um inteiro arbitrário

Demonstração: Suponhamos que o par de inteiros 0x ,

0y é uma solução particular da

equação ax + by = c, e seja 1x ,

1y uma outra solução qualquer desta equação. Então, temos:

e, portanto:

Por ser o mdc (a, b) = d, existem inteiros r e s tais que a = dr e b = ds, com r e s primos entre

si. Substituindo estes valores de a e b na igualdade anterior e cancelando o fator com d,

obtemos:

Assim sendo, 0 1r | s(y y ) , e como o mdc (r, s) = 1, segue-se que

0 1r | (y y ) , isto é:

onde t é um inteiro. Portanto, temos as fórmulas:

Estes valores de 1x e

1y satisfazem realmente a equação ax + by = c, qualquer que seja o

inteiro t, pois, temos:

a

1x + b1y = a[

0x (b / d)t ] + b[0y (a / d)t ] = a

0x + b0y + (ab/d – ab/d)t = c + 0.t = c

1 0 0x x st x (b / d)t

1 0 0y y rt y (a / d)t

0 1y y = rt e 1 0x x = st

r(1x -

0x ) = s(0y -

1y )

a(1x -

0x ) = b(1y -

0y )

a0x + b

0y = c = a1x + b

1y

0

bx x t

d,

0

ay y t

d

CAPÍTULO 8

EQUAÇÕES DIOFANTINAS LINEARES

114

Como se vê, se d = mdc (a, b) divide c ( d | c ), então a equação diofantina linear ax + by = c

admite um número infinito de soluções, uma para cada valor do inteiro arbitrário t.

Corolário 3.1: Se o mdc (a, b) = 1 e se 0x ,

0y é uma solução particular da equação diofantina

linear ax + by = c, então todas as outras soluções desta equação são dadas pelas fórmulas:

onde t é um inteiro arbitrário.

Nota: Uma solução particular da equação diofantina linear se obtém por tentativas ou pelo

algoritmo de Euclides. E em ambos os casos a solução geral se pode obter usando o teorema

3.2, conforme se vai ver nos exemplos a seguir.

Exemplo 3.1: Determinar todas as soluções inteiras e positivas da equação diofantina linear

Determinemos o mdc (18, 5) pelo algoritmo de Euclides:

18 = 5.3 + 3

5 = 3.1 + 2

3 = 2.1 + 1

2 = 1.2

Portanto, o mdc (18, 5) = 1 e a equação dada tem solução. e para exprimir 1 como

combinação linear de 18 e 5 basta eliminar os restos 2 e 3 entre as três primeiras igualdades

anteriores do seguinte modo:

isto é:

e

Logo, o par de inteiros 0x = 96,

0y = -335 é uma solução particular da equação proposta, e

todas as demais soluções são dadas pelas fórmulas:

3 1 1 2

18 5 3 2 1

3 2 1 0

x = 96 + 5t, y = -336 – 18t

48 = 18.96 + 5(-336)

1 = 18.2 + 5(-7)

1 = 3 – 2 = 3 – (5 – 3) = 2.3 – 5 = 2(18 – 5.3) – 5 = 18.2 + 5(-7)

18x + 5y = 48

0x x bt , 0y y at

CAPÍTULO 8

EQUAÇÕES DIOFANTINAS LINEARES

115

onde t é um inteiro arbitrário.

As soluções inteiras e positivas se acham escolhendo t de modo que sejam satisfeitas as

desigualdade:

Isto é:

o que implica t = -19 e, portanto:

Assim, o par de inteiros x = 1, y = 6 é a única solução inteira e positiva da equação 18x + 5y =

48.

Exemplo 3.2: Resolver a equação diofantina linear

O mdc (a, b) = 13 e como 13 não divide 105, segue-se que a equação dada não tem solução.

EXERCÍCIOS

1) Determinar todas as soluções inteiras das

seguintes equações diofantinas lineares:

a) 56x + 72y = 40

b) 24x + 138y = 18

c) 221x + 91y = 117

d) 84x – 438y = 156

e) 48x + 7y = 5

f) 57x – 99y = 77

g) 11x + 30y = 31

h) 27x – 18y = 54

i) 13x – 7y = 21

j) 44x + 66y = 11

k) 21x – 12y = 72

l) 17x + 54y = 8

2) Determinar todas as soluções inteiras e

positivas das seguintes equações diofantinas

lineares:

a) 5x – 11y = 29.

b) 32x + 55y = 771

c) 58x – 87y = 290

d) 62x + 11y = 788

e) 30x + 17y = 300

f) 54x + 21y = 906

g) 123x + 360y = 99

h) 158x – 57y = 7

3) Determinar o menor inteiro positivo que

dividido por 8 e por 15 deixa os restos 6 e 13,

respectivamente.

39x + 26y = 105

x = 96 + 5(-19) = 1, y = -336 -18(-19) = 6

t > 19,2 e t < 18,6

96 + 5t > 0, -336 – 18t > 0

CAPÍTULO 8

EQUAÇÕES DIOFANTINAS LINEARES

116

4) Exprimir 100 como soma de dois inteiros

positivos de modo que o primeiro seja

divisível por 7 e o segundo seja divisível por

11.

5) Determinar as duas menores frações positivas

que tenham 13 e 17 para denominadores e

cuja soma seja igual a 305

221.

6) Determine todas as soluções inteiras do

sistema de equações 2 3 5 201

3 5 7 315

x y z

x y z

7) Encontre todas as soluções da equação

Diofantina (6x+15y)(8x+7y) = 129.

8) Uma pessoa foi ao banco para descontar um

cheque no valor de x reais e y centavos.

O caixa do banco errou na leitura do valor do

cheque e pagou y reais e x centavos. A pessoa

guardou o dinheiro no bolso sem verificar a

quantia. No caminho de casa, ela gastou cinco

centavos e quando chegou em casa verificou

que tinha exatamente o dobro do valor do

cheque. Sabendo-se que essa pessoa não levou

dinheiro nenhum consigo quando foi ao

banco, pergunta-se qual era o valor do cheque.

9) Um grupo de pessoas gastou 1000 dólares

num hotel. Sabendo-se que apenas alguns dos

homens estavam acompanhados pelas esposas

e que cada homem gastou 19 dólares e cada

mulher gastou 13 dólares, pede-se determinar

quantas mulheres e quantos homens estavam

no hotel.

10) Um grupo de pessoas gastou 690 dólares num

hotel. Sabendo-se que apenas alguns dos

homens estavam acompanhados pelas esposas

e que cada homem gastou 18 dólares e cada

mulher gastou 15 dólares, pede-se determinar

quantas mulheres e quantos homens estavam

no hotel.

11) Ao entrar num bosque, alguns viajantes

avistam 37 montes de maçãs. Após serem

retiradas 17 frutas, o restante foi dividido

igualmente entre 79 pessoas. Qual pode ter

sido a parte de cada pessoa?

12) Sabendo que um time de basquete é composto

de 5 jogadores e um time de vôlei é formado

por 6 jogadores. Quantas quadras de basquete

e quantas de vôlei são necessárias para que 80

alunos joguem simultaneamente qualquer um

dos esportes? E se forem 77 alunos?

13) O laboratório Sangue Bom, dispõe de 2

máquinas para examinar amostras de sangue.

Uma delas examina 15 amostras de cada vez,

enquanto a outra examina 25. Quantas vezes

essas máquinas devem ser acionadas para

examinar exatamente 2 mil amostras?

14) Num determinado lugar a moeda é o mirrél.

Suponhamos que só existam moedas de 15 e

7 mirréis e que se queira pagar uma

determinada quantia em mirréis. Será que é

sempre possível? E se existirem moedas de

12 e 30 mirréis?

15) Para agrupar 13 aviões em filas de 3 ou de 5,

exatamente quantas filas serão formadas de

cada tipo?

16) Para participar de um evento comemorativo

em um clube, não sócios pagavam R$ 12,00 e

sócios R$ 8,00. Sabendo-se que foram

arrecadados R$ 908,00 na portaria, quantos

sócios estiveram no evento?

17) Um galo custa 5 mirréis, uma galinha 3

mirréis, e três pintinhos 1 mirrél. Com 100

mirréis um fazendeiro comprou 100 dessas

aves. Quantos galos, galinhas e pintinhos

foram comprados?

18) Demonstrar que se a e b são inteiros positivos

primos entre si, então a equação diofantina ax

– by = c têm um número infinito de soluções

inteiras e positivas.

117

Capítulo 9

CONGRUÊNCIAS

9.1. Congruências

Definição 9.1 Sejam a e b inteiros quaisquer e seja m >1 um inteiro positivo fixo. Diz-se que

a é congruente a b módulo m se, e somente se, m divide a diferença a – b. Em outros termos a

é congruente a b módulo m se, e somente se, existe um inteiro k tal que a – b = km

Simbolicamente:

Exemplos 9.1 3 24 (mod 7) ; –31 11 (mod 6) ; –15 –63 (mod 8)

Definição 9.2 Se m não divide a diferença a – b, então diz-se que a é incongruente a b

módulo m .

Notação: a b (mod m)

Observações:

1) Dois inteiros quaisquer são congruentes módulo 1

2) Dois inteiros são congruentes módulo 2, se ambos são pares ou ambos são

ímpares

3) a 0 (mod m) se, e somente se, m | a.

9.2. Caracterização de Inteiros Congruentes

Teorema 9.1 Dois inteiros a e b são congruentes módulo m se, e somente se, a e b deixam o

mesmo resto quando divididos por m.

Demonstração:

( ) Suponhamos que a b ( mod m). Então, pela definição:

a – b = km, k Z

a b (mod m) m | ( a – b ) a - b = km a = km + b

CAPÍTULO 9

CONGRUÊNCIAS

118

Seja r o resto da divisão de b por m; então pelo algoritmo da divisão:

Portanto:

e isto significa que r também é o resto da divisão de a por m, isto é, os inteiros a e b divididos

por m deixam o mesmo resto r.

( ) Reciprocamente, suponhamos que a e b divididos por m deixam o mesmo resto r. Então,

podemos escrever:

e, portanto:

9.3. Propriedades das Congruências

Teorema 9.2 Seja m um inteiro positivo fixo (m > 1) e sejam a, b e c inteiros quaisquer.

Valem as propriedades:

1) a a (mod m) (Reflexiva)

2) Se a b (mod m), então b a (mod m) (Simétrica)

3) Se a b (mod m) e se b c(mod m), então a c (mod m) (Transitiva)

Demonstração:

(1) a 0 ou seja, a (a –a), o que implica: a a (mod m)

(2) Se a b (mod m), então a – b = km, k z.

Portanto:

(3) Se a b (mod m) e se b c (mod m), então existem inteiros h e k tais que

Portanto:

a – c = (a - b) + (b - c) = hm + km = (h + k)m

a– b = hm e b – c = km

b – a = -(km) = (-k)m b a (mod m)

a – b = ( q1 – q2) m m (a – b) a b ( mod m)

a = mq1 + r e b = mq2 + r , 0 r < m

a = km + b = km + mq + r = (k + q)m + r

b = mq + r, 0 r < m

CAPÍTULO 9

CONGRUÊNCIAS

119

e isto significa que a c (mod m).

Nota: Consoante este teorema, as relação binária R no conjunto Z dos

inteiros definidas por

aRb a b (mod m) é reflexiva, simétrica e transitiva, ou seja, R é uma

relação de equivalências em Z. Esta relação de equivalência R em z é

denominada “congruência módulo m” .

Teorema 9.3 Seja m um inteiro positivo fixo (m > 1) e sejam a, b dois inteiros quaisquer.

Valem as seguintes propriedades:

1) Se a b (mod m) e se n | m, com n > 0, então a b (mod n)

Demonstração: Com efeito:

onde k e q >0 são inteiros.

Portanto:

2) Se a b (mod m) e se c > 0 , então ac bc (mod mc)

Demonstração: Com efeito, se a b (mod m), então:

3) Se a b (mod m) e se a, b, m são todos divisíveis pelo inteiro d > 1, então

d

b

d

a (mod

d

m)

Demonstração:

Com efeito, se a b (mod m), então:

a b m a b ma - b = km - = k( ) (mod )

d d d d d d

a – b = km ac – bc = k(mc) ac bc (mod mc)

a –b = (kq)n a b (mod n)

a b (mod m) a – b =km e n m m = nq

CAPÍTULO 9

CONGRUÊNCIAS

120

Teorema 9.4 Seja m um inteiro positivo fixo (m > 1) e sejam a, b, c, d inteiros quaisquer.

Valem as seguintes propriedades:

1) Se a b (mod m) e se c d (mod m), então a + c b + d (mod m) e ac bd

(mod m).

Demonstração:

Se a b (mod m) e se c d (mod m), então existem inteiros h e k tais que

a – b = hm e c – d = km.Portanto:

(a + c) – (b + d) = (a - b)+ (c- d) = (h + k)m

e

ac – bd = (b + hm) (d + km) - bd = (bk + dh + hkm)m

o que implica:

2) Se a b (mod m) e c um inteiro qualquer, então a + c b + c (mod m) e ac bc

(mod m).

Demonstração: Temos:

Logo, pela propriedade anterior:

a + c b + c (mod m) e ac bc (mod m)

Em particular, se c = -1, então:

a (-1) b(-1) (mod m) ou -a -b (mod m)

3) Se a b (mod m), então an b

n (mod m) para todo inteiro positivo n.

Demonstração:

Usando o “Teorema da indução Matemática”, a proposição é verdadeira para n = 1, e

suposta verdadeira para o inteiro positivo k temos:

Portanto, pela propriedade 1 acima

a k b k (mod m) e a b (mod m)

a b (mod m) e c c (mod m)

a + c b + d (mod m) e ac bd (mod m)

CAPÍTULO 9

CONGRUÊNCIAS

121

isto é, as proposição é verdadeira para o inteiro positivo k + 1. logo, a preposição é verdadeira

para todo inteiro positivo n.

Teorema 9.5: Se ac bc (mod m) e se o mdc(c,m) = d, então a b (mod d

m)

Demonstração: Com efeito, se ac bc (mod m), então:

Como o mdc (c,m) = d, existem inteiro r e s tais que c = dr e m = ds, onde r e s são primos

entre si. Portanto:

o que implica que s (a – b)r, com o mdc (r,s) = 1. Logo, pelo Teorema 5.4 ( de Euclides): s

(a – b) e a b (mod s) ou, por ser s =d

m, a b (mod

d

m ).

Corolário 9.1 Se ac bc ( mod m) e se o mdc (c,m) = 1, então a b (mod m).

Esta propriedade mostra que é permitido cancelar fatores de ambos os membros de uma

congruência que são primos com o módulo.

Corolário 9.2 Se ac bc (mod p), com p primo, e se p não divide c, então

a b (mod p)

Demonstração: Com efeito, as condições: p não divide c e p é primo, implicam que o

mdc(c, p) = 1.

9.4. Sistemas Completos de Restos

Definição 9.3 Chama-se sistema completo de restos módulo m todo conjunto

S = {r1 , r2 , ... , rm} de m inteiros tal que um inteiro qualquer a é congruente módulo m a um

único elemento de S.

Exemplo 9.2: Cada um dos conjuntos:

é um sistema completo de restos módulo 3.

{1, 2, 3} , {0, 1, 2} , { –1, 0, 1} , {1, 5, 9}

(a – b) dr = kds ou (a - b)r = ks

ac – bc = (a – b)c = km , com k Z.

a k .a b k .b (mod m) ou ak+1

bk+1

(mod m)

CAPÍTULO 9

CONGRUÊNCIAS

122

Teorema 9.6 O conjunto S = {0,1, 2, ..., m – 1} é um sistema completo de restos módulo m.

Demonstração: Com efeito, o conjunto S tem m elementos e, além disso, qualquer que seja o

inteiro a temos, pelo algoritmo da divisão:

o que implica a ≡ r (mod. m), e como o resto r só pode assumir os m valores 0, 1, 2, ..., m-1,

segue-se que o inteiro a é congruente módulo m a um único desses m inteiros.

Corolário 9.3 Se S = {r1 , r2 , ... , rm} é um sistema completo de restos módulo m, então os

elementos de S são congruentes módulo m aos inteiros 0, 1, 2, ... , m – 1, tomados numa certa

ordem.

Demonstração: Com efeito, se a é um inteiro qualquer, então:

o que implica: r1 ≡ k (mod. m).

9.5 – Aritmética Módulo m

Definição 9.4.: Seja a um inteiro. Chama-se classe de congruência de a módulo m (m > 1)

o conjunto formado por todos os inteiros que são congruentes a a módulo m. Denotamos esse

conjunto por a . Temos, então:

Como x a (mod m), se e somente se, x é da forma x = a + k.m, para algum

k Z, também podemos escrever:

Mostraremos a seguir que a relação de congruência entre números se traduz em igualdade no

sentido estrito entre classes.

Proposição 9.1: Sejam a e b inteiros. Então a b (mod m), se e somente se, a b .

Demonstração: Suponhamos que a b (mod m), queremos provar que a = b , isto é, uma

igualdade entre conjuntos. Dado x a , temos por definição que x a (mod m). Da

propriedade transitiva de congruência e da hipótese, segue imediatamente que x b . Logo, a

b . A inclusão b a em sentido contrário segue de forma análoga.

a = { a + k.m | k Z }

a = { x Z ; x a (mod m) }

a ≡ r1 (mod. m), com r1 Є S

a ≡ k (mod. m), com 0 ≤ k ≤ m-1

a = mq + r, com 0 ≤ r ≤ m

CAPÍTULO 9

CONGRUÊNCIAS

123

Reciprocamente, se a = b , como a a , temos também que a b , logo, a b (mod m).

Corolário 9.4: Sejam a e b inteiros. Se a b , então a b = .

Demonstração: Se a b = , consideremos um inteiro c que pertença a ambas as classes.

Como c a , temos que c a (mod m) e, de forma análoga, c b (mod m). Portanto, a b

(mod m) e, da proposição acima, a = b .

Note que, por exemplo, para as classes módulo 7, temos que

...71470 ou ...3114 etc.

Mais precisamente, dada uma classe a , para qualquer inteiro x tal que x a ,

temos que x=a . Por causa disto, cada inteiro pertencente a uma dada classe

diz-se um representante daquela classe. Por exemplo, 11 e – 3 são

representantes da classe 4 módulo 7.

Consideremos um sistema completo de classes ou resíduos módulo m, por exemplo, os

inteiros 0, 1, ..., m – 1 e suas respectivas classes:

Conforme já foi considerado, cada inteiro pertence a uma e apenas uma das m classes.

Por exemplo, se m = 7, todas as classes possíveis, módulo 7, são as seguintes:

Denotaremos pelo símbolo m o conjunto das classes de congruências módulo m e o

chamaremos de Conjunto dos Inteiros Módulo m.

Assim, Z7 = { 0 , 1, 2 , 3 , 4 , 5 , 6 }.

0 = { 0, 7, 14, 21, ... }

1 = { 1, 1 7, 1 14, 1 21, ... }

2 = { 2, 2 7, 2 14, 2 21, ... }

3 = { 3, 3 7, 3 14, 3 21, ... }

4 = { 4, 4 7, 4 14, 4 21, ... }

5 = { 5, 5 7, 5 14, 5 21, ... }

6 = { 6, 6 7, 6 14, 6 21, ... }

0 = { 0, m, 2.m, 3.m, ... }

1 = { 1, 1 m, 1 2.m, 1 3.m, ... }

...

1m = { m – 1, m – 1 m, m – 1 2.m, m – 1 3.m, ... }

CAPÍTULO 9

CONGRUÊNCIAS

124

Note que, por exemplo,

0 = 7 , 1 = 15 , 2 = 9 , 3 = 11, 4 = 25 , 5 = 16 ,

6 = 8 e também podemos escrever:

Z7 = { 7 , 15 , 9 , 11, 25 , 16 , 8 }.

Em geral, se { a1, a2, ..., am } é um sistema completo de restos módulo m, temos que:

Tomando o sistema de restos mais simples, podemos escrever:

Note que, conforme as observações acima, o conjunto m tem precisamente

m elementos.

9.6. Adição e Multiplicação em m

Agora gostaríamos de introduzir operações de soma e produto em m e estudar suas

propriedades. Existe uma forma natural de fazê-lo. Por exemplo, para somar e multiplicar 3 e

6 em Z7, faríamos:

3 + 6 = 9 = 2

3 . 6 = 18 = 4

m = { 0 , 1, 2 , ..., 1m }

m = { 1a ,

2a , ..., ma }

CAPÍTULO 9

CONGRUÊNCIAS

125

Exemplificando a adição e a multiplicação em uma “tabuada” módulo 7:

+ 0 1 2 3 4 5 6 * 0 1 2 3 4 5 6

0 0 1 2 3 4 5 6 0 0 0 0 0 0 0 0

1 1 2 3 4 5 6 0 1 0 1 2 3 4 5 6

2 2 3 4 5 6 0 1 2 0 2 4 6 1 3 5

3 3 4 5 6 0 1 2 3 0 3 6 2 5 1 4

4 4 5 6 0 1 2 3 4 0 4 1 5 2 6 3

5 5 6 0 1 2 3 4 5 0 5 3 1 6 4 2

6 6 0 1 2 3 4 5 6 0 6 5 4 3 2 1

Observações:

Observe, na tabela de adição, o conceito de inverso aditivo módulo m. Dizemos

que dois elementos de m são inversos aditivos, se e somente se, 0(mod )a b m

. Assim, por exemplo, 4 e 3 são inversos aditivos módulo 7, uma vez que

4 3 0(mod7) .

Mais explicitamente, definimos soma e produto em Zm por:

Quer dizer, para efetuar a soma de duas classes módulo m, tomamos representantes

(quaisquer) a e b dessas classes, efetuamos a soma a + b em Z e consideramos como resultado

da soma a classe de a + b módulo m . A operação de produto se faz de forma análoga.

Surge agora uma pergunta natural: será que o resultado das operações não depende dos

representantes escolhidos? Voltando ao exemplo de Z7 , para somar 3 + 6 , poderíamos tomar

38 como um representante de 3 e 27 como representante de 6 . Será que 38 + 27 = 65 é o

mesmo resultado que aquele obtido acima, 3 + 6 = 2? A resposta é afirmativa. Como 65 2

(mod 7), felizmente o resultado é o mesmo. O lema abaixo mostra que isso não é uma mera

coincidência.

Lema 9.1: Sejam a, a’, b e b’ inteiros tais que a = 'a e b = 'b . Então,

ba = 'b'a e b .  a = 'b' .  a .

Demonstração: É uma conseqüência imediata das propriedades.

Proposição 9.2: Em m valem as seguintes propriedades:

3 + 6 = 9 = 2

3 . 6 = 18 = 4

CAPÍTULO 9

CONGRUÊNCIAS

126

(P1) Propriedade Associativa: Para toda terna a , b , c de inteiros módulo m, tem-se que: a + (

b + c ) = ( a + b ) + c

(P2) Existência do Elemento Neutro: Existe um único elemento em m que é precisamente 0

a classe do elemento 0, tal que: a + 0 = a , para todo a m

(P3) Existência do Elemento Oposto: Para cada inteiro módulo m, a , existe um único

elemento em Zm que chamaremos oposto de a e indicaremos por a , tal que: a + ( a ) = 0

(P4) Propriedade Comutativa: Para todo par a , b de elementos de Zm tem-se que: a + b = b

+ a .

Demonstração: As demonstrações são feitas apoiando-se nos axiomas para as operações com

números inteiros. A título de ilustração, provaremos P1 e P3.

(P1)Utilizando repetidamente a definição de soma em m , temos:

Agora, como vale a associativa da soma entre números inteiros, ou seja:

temos que:  )cb(  a = c )ba(  

logo, a + ( b + c ) =  )cb(  a = c )ba(   = ( a + b ) + c

Na última sequência de igualdade usamos, novamente, apenas a definição de soma em m .

( P3) Dado a m , basta tomar a classe a e verificar que:

Para provar a unicidade, suponhamos que b m também verifica a + b = 0 ou, usando da

comutatividade, b + a = 0 . Temos então:

A verificação de P2 é imediata. Note, porém, que a classe do elemento neutro é formada

também pelos múltiplos de m. Temos, assim, que 0 = m .

Da demonstração de P3 vem que o oposto de a em m é a classe a = – a . É claro que, se

explicitamos m na forma

m = { 0 , 1, 2 , ..., 1m } e a é um dos representantes utilizados,

então – a não é um deles. Para obter o menor representante positivo da classe de – a, fazemos:

a = 0 + ( a ) = m + ( a ) = m – a = a    m

b = b + 0 = b + ( a + ( a ) ) = ( b + a ) + ( a ) = 0 + ( a ) = a .

a + ( a ) = )a(a = 0

a + ( b + c ) = ( a + b ) + c

a + ( b + c ) = a + ( cb ) =  )cb(  a

CAPÍTULO 9

CONGRUÊNCIAS

127

Por exemplo, em Z7 temos que 2 = 0 – 2 = 7 – 2 = 2    7 = 5 (de fato, 2+5=7=0 ).

Listamos na próxima proposição as propriedades do produto e a propriedade distributiva, que

relaciona ambas as operações.

Proposição 9.3: Em m valem as seguintes propriedades:

(P5) Propriedade Associativa: Para toda terna a , b , c de inteiros módulo m, tem-se que:a . (b

. c ) = ( a . b ) . c

(P6) Existência do Elemento Neutro: Existe o único elemento em m que é precisamente1, tal

que: a . 1 = a .

(P7) Propriedade do Elemento Oposto: Comentário mais adiante.

(P8) Propriedade Comutativa: Para todo par a , b de elementos de m tem-se que: a . b = b .

a .

(P9) Propriedade Distributiva: Para toda terna a , b , c de elementos de Zm tem-se que:

a . (b + c ) = a .b + a .c .

Demonstração: Tal como na proposição anterior, deixaremos as demonstrações como

exercício. São feitas reduzindo-as ao caso dos inteiros.

Você deve ter notado que não listamos uma propriedade P7 que, no paralelismo que

estávamos fazendo com as propriedades das operações nos inteiros, corresponderia à

propriedade cancelativa. Isso ocorreu porque ela não é válida em geral. Com efeito, por

exemplo, em Z6 temos que 3 .2 = 6 = 0 , 3 .4 = 12 = 0 ; logo, 3 .2 = 3 .4 , porém 2 4 .

No contra-exemplo acima, temos dois elementos não-nulos de Z6 cujo produto é zero,

situação que não acontece em . Para melhor estudar a propriedade cancelativa,

começaremos formalizando esse conceito.

Definição 9.5: Um elemento não-nulo a m diz-se um divisor de zero se existe b

m ,

também não-nulo, tal que a .b = 0 . Agora, determinaremos quais são os divisores de zero em

m .

Lema 9.2: Um elemento não-nulo a m é divisor de zero, se e somente se,

m.d.c.(a, m) 1.

Demonstração: Seja a um divisor de zero e b 0 um elemento de m tal que a .b = 0 . Como

a .b = b .a = 0 , temos que a.b 0 (mod m), isto é, m divide a.b (representamos por “m | a.b”).

Supondo por absurdo que m.d.c.(a, m) = 1, o Teorema de Euclides diz que para a e b inteiros

tais que a | b.c, se m.d.c.(a, b) = 1, então a | c , assim, vem que m | b, logo, b = 0 , uma

contradição. Reciprocamente, suponhamos que m.d.c.(a, m) = d > 1. Vamos determinar um

elemento b 0 em Zm tal que a .b = 0 .

Podemos escrever a = a1 . d , e m = m1 . d, em que 0 < m1 < m (já que d > 1).

Logo, m1 0 . Agora, temos que:

CAPÍTULO 9

CONGRUÊNCIAS

128

Logo, em Zm temos:

Assim, basta tomar b = m1.

Como consequência imediata deste Lema temos o Corolário abaixo.

Corolário 9.5: Seja p > 1 um inteiro primo. Então, p não contém divisores de zero.

Vale também a recíproca.

Lema 9.3: Se m não contém divisores de zero, então m é primo.

Demonstração: Suponhamos, por absurdo, que m seja composto, isto é da forma m = r . s

com 1 < r < m, 1 < s < m. Temos, então, que: 0 = m = r . s em que r 0 e s 0, uma

contradição.

Proposição 9.4: A propriedade cancelativa do produto vale em m , se e somente se, m é

primo.

Demonstração: Suponhamos inicialmente que m seja primo, e sejam a , b , c elementos de

m , com a 0, tais que a .b = a .c . Então, a .(b – c ) = 0.

Como a 0 e m não tem divisores de zero, deve ser b – c = 0, donde b = c .

Suponhamos que a propriedade cancelativa seja válida, mostraremos que nesse caso que m

não contém divisores de zero. A tese seguirá então do Lema anterior.

Sejam a , b Zm tais que a .b = 0 . Se a 0 , escrevemos a .b = a .0 e, como podemos

cancelar, temos que b = 0.

Para continuar nosso estudo comparativo de com m , introduzimos ainda outro conceito.

Definição 9.6: Um elemento a m diz-se inversível se existe 'a

m tal que a . 'a = 1. Um

elemento 'a nessas condições diz-se um inverso de a .

O conjunto dos elementos de m que têm inversos é muito importante. Vamos denotá-lo por

(m). Em outras palavras,

1m .  a = m .  a1 = 0

a . m1 = ( a1 . d ) . m1 = a1 . ( d . m1 ) = a1 . m

CAPÍTULO 9

CONGRUÊNCIAS

129

No caso de m ser primo, todas as classes diferentes de 0 possuem inverso.

Os únicos elementos inversíveis de são 1 e –1.

Obviamente, 1 e 1 são sempre inversíveis em m . Porém há outros exemplos.

Em Z5 temos que 2 .3 = 6 = 1 e 4 .4 = 16 = 1, logo 2 , 3 e 4 são também inversíveis de Z5, 2

é o inverso de 3 e, reciprocamente, 4 é o seu próprio inverso.

Em Z6 temos que 5 .5 = 25 = 1; logo, 5é um inversível de Z6.

Por outro lado, é claro que 0 não é inversível em m , para nenhum valor de m. De fato, para

qualquer a m temos que 0 .a = 0 1.

Observe as tabelas de multiplicação em Z6 e em Z7.

* 0 1 2 3 4 5 6 * 0 1 2 3 4 5

0 0 0 0 0 0 0 0 0 0 0 0 0 0 0

1 0 1 2 3 4 5 6 1 0 1 2 3 4 5

2 0 2 4 6 1 3 5 2 0 2 4 0 2 4

3 0 3 6 2 5 1 4 3 0 3 0 3 0 3

4 0 4 1 5 2 6 3 4 0 4 2 0 4 2

5 0 5 3 1 6 4 2 5 0 5 4 3 2 1

6 0 6 5 4 3 2 1

Com exceção de 0, todos os elementos em Z7 (7 é primo) possuem inverso, enquanto que em

Z6 (6 não é primo), apenas 1 e 5 possuem inverso. Isso está de acordo com a proposição

abaixo.

Proposição 9.5: Seja a um elemento não-nulo de m . Então, a é inversível, se e somente se,

m.d.c.(a,m) = 1.

Demonstração: Suponhamos que m.d.c.(a, m) = 1. O Teorema de Bézout afirma que sendo a

e b inteiros, d = m.d.c.(a, b). Então existem inteiros r e s tais que d = r.a + s.b . Também,

existem inteiros r e s tais que a.r + m.s = 1.

Tomando classes temos que:

Logo, r é o inverso de a .

Reciprocamente, se m.d.c.(a, m) 1, então a é divisor de zero e existe b 0 tal que a .b = 0 .

Mostraremos que, nesse caso, a não pode ser inversível. Com efeito, suponhamos que existe

'a tal que 'a.a = 1. Teríamos, então:

b = b .1 = b .( 'a.a ) = ( b .a ). 'a = (a .b ). 'a = 0 . 'a = 0 , uma contradição.

1 = s.m  r.a = r.a + s.m = a .r + m .s = a .r + 0 .s = a .r

(m) = { a m ; m.d.c.(a, m) = 1 }

CAPÍTULO 9

CONGRUÊNCIAS

130

Uma consequência imediata da proposição anterior é a seguinte:

Corolário 9.6: Seja p > 0 um inteiro primo. Então, todo elemento não-nulo de p é

inversível.

Exemplo 9.3: Veja na “tabuada” de Z11 que todo elemento não nulo, possui um inverso:

* 1 2 3 4 5 6 7 8 9 10

1 1 2 3 4 5 6 7 8 9 10

2 2 4 6 8 10 1 3 5 7 9

3 3 6 9 1 4 7 10 2 5 8

4 4 8 1 5 9 2 6 10 3 7

5 5 10 4 9 3 8 2 7 1 6

6 6 1 7 2 8 3 9 4 10 5

7 7 3 10 6 2 9 5 1 8 4

8 8 5 2 10 7 4 1 9 6 3

9 9 7 5 3 1 10 8 6 4 2

10 10 9 8 7 6 5 4 3 2 1

9.7. Subtração em m

A operação de subtração no sistema aritmético módulo m, tal como na aritmética comum, é

definida em termos de adição.

Na aritmética comum, podemos achar a resposta para 13 – 4, procurando o número que

somado a quatro nos dê 13. Como 4 + 9 = 13, o número 9 é a resposta do problema

13 – 4 = 9.

Você deve ter usado muito este processo ao conferir resultados de subtrações. Na verificação

adicionamos a diferença ao subtraendo para ver se a soma se iguala ao minuendo. Dizemos

que a subtração é a operação inversa da adição.

Usando os elementos mód. 7, resolva o problema de subtração 6 – 5 = ? Qual é o elemento

que somado a 5 resulta em 6?

Da tabela da adição mód. 7, 5 + 1 = 6. Portanto, podemos substituir 1 no lugar do ponto de

interrogação, e 6 – 5 = 1.

CAPÍTULO 9

CONGRUÊNCIAS

131

Na aritmética comum, não podemos subtrair 5 – 6 sem termos um resultado negativo. Os

números negativos não são necessários na subtração mód. m. No problema 5 – 6 = ?,

procuramos um elemento que somado a 6 nos dê 5. A tabela da adição nos revela que a soma

de 6 + 6 é 5. Portanto, 6 pode substituir o? e 5 – 6 = 6.

Você encontrará outros problemas de subtração nesse sistema que são bem diferentes e

interessantes. Considere o problema 4 – 5 = n, onde n é a resposta. Pela nossa definição de

subtração n + 5 deve ser 4. Qual é o número que somado a 5 em mód. 7 é 4? Da tabela da

adição, vemos que n = 6 e, portanto, 4 – 5 = 6.

Neste sistema finito, 1 - 4 = 4 – 1? No primeiro membro, qual é o elemento que somado a 4

dá 1? Como 4 + 4 = 1, 1 – 4 = 4. Do mesmo modo, temos que achar um número que somado a

1 resulta em 4. Como 1 + 3 = 4, 4 – 1 deve ser 3. Você vê, portanto, que 1 – 4 ≠ 4 – 1 e a

propriedade comutativa não é válida para a operação subtração.

A tabela abaixo resume o que foi dito.

- 0 1 2 3 4 5 6

0 0 1 2 3 4 5 6

1 6 0 1 2 3 4 5

2 5 6 0 1 2 3 4

3 4 5 6 0 1 2 3

4 3 4 5 6 0 1 2

5 2 3 4 5 6 0 1

6 1 2 3 4 5 6 0

9.8. Divisão em m

Não é necessário usar frações se você trabalhar no sistema aritmético de módulos. Como é

isto possível? Vamos considerar a operação de divisão. Na divisão de números naturais,

verificamos as respostas, multiplicando o quociente pelo divisor para chegarmos ao

dividendo. Assim, 125 ÷25 = 5 pois 5 . 25 = 125.

A operação divisão, na aritmética módulo m, também pode ser definida como operação

inversa da multiplicação. Por exemplo, usando o mód.7, o problema 5 ÷ 4 é resolvido

encontrando um número n, tal que n . 4 = 5. Já que completamos a tabela de multiplicação

mód. 7 podemos usá-la para achar a resposta. Temos que, 3 . 4 = 5 e, portanto, 5 ÷ 4 = 3. Qual

é a resposta de 6 ÷ 3? Como 2 . 3 = 6, a resposta ao problema 6 ÷ 3 é 2. Neste sistema a

divisão, quando existe, é sempre exata.

7

CAPÍTULO 9

CONGRUÊNCIAS

132

Se, consideramos a expressão 3 ÷ 2 como sendo o mesmo que 3

2 e 1 ÷ 3 o mesmo que

1

3,

teremos uma interessante vantagem na aritmética de módulos.

3

3÷2=52

e também 1

1÷ 3 = 53

As frações são rapidamente substituídas por algum elemento do sistema mód. 7. Uma

expressão tal

como

12

41

33

é simplificada de modo análogo.

A fração 1

4 é equivalente a 2, pois em mód. 7, 4 . 2 = 1.

Portanto, 1

24

é equivalente a 2 + 2, ou 4. O denominador, 1

33

, é igual a 1, pois

15

3 e 3 + 5 = 1 em mód. 7. Finalmente

44

1, pois 1 . 4 = 4.

Considere o problema 2 ÷ 6 em aritmética mód. 7. Neste caso, temos que achar a resposta n,

tal que n . 6 = 2. Como 5 . 6 = 2, então 2 ÷ 6 = 5. Entretanto, se mudarmos o problema para 6

÷ 2 teremos uma resposta totalmente diferente. No caso de 6 ÷ 2 a resposta é 3, pois 3 . 2 = 6.

É evidente que 6 ÷ 2 não é igual a 2 ÷ 6, portanto, a propriedade comutativa não se verifica

na divisão módulo m.

A tabela abaixo resume o que foi dito.

7

÷ 1 2 3 4 5 6

1 1 2 3 4 5 6

2 4 1 5 2 6 3

3 5 3 1 6 4 2

4 2 4 6 1 3 5

5 3 6 2 5 1 4

6 6 5 4 3 2 1

Podemos resolver facilmente o problema 3.x = 0, usando a multiplicação de números naturais

da aritmética comum. Sabemos que o produto é 0; portanto, o numero que devemos colocar

no lugar de x deve ser o zero. Você já deve ter visto muitas vezes a afirmação de que se um

produto é zero, pelo menos um dos fatores deve ser igual a zero. Isto não é sempre verdade!

Pelo menos não é verdade em alguns sistemas aritméticos finitos.

É possível um problema de divisão ter mais de uma resposta? Você deve estar imaginando

que isto nunca é possível. Entretanto, no sistema aritmético módulo m isso pode ocorrer.

CAPÍTULO 9

CONGRUÊNCIAS

133

Consideremos sistema módulo 6. Neste sistema, 4 ÷ 4 = 1. Isto não deve ser estranho para

você, mas acontece que 4 ÷ 4 também é igual a 4 na aritmética mód. 6. Vamos explorar este

sistema finito mais detalhadamente.

Já vimos que os sistemas aritméticos módulo m, podem ser divididos em duas categorias: o de

módulo primos e não- primos.

Consideremos os sistemas aritméticos onde m é não-primo.

Vamos usar como exemplo o módulo 6. Na tabela completa de multiplicação abaixo, notamos

alguns padrões um pouco estranhos.

Quando a tabela é simétrica com relação a diagonal, dizemos que a operação é comutativa.

Entretanto, repare nos produtos resultantes da multiplicação por 2. Além do estranho

resultado de 2 . 3 ser igual a 0, apesar de nenhum fator ser 0. E nesta multiplicação só temos

como respostas o 0, 2 e 4.

* 1 2 3 4 5

1 1 2 3 4 5

2 2 4 0 2 4

3 3 0 3 0 3

4 4 2 0 4 2

5 5 4 3 2 1

Do mesmo modo, somente teremos respostas 0 ou 3 na multiplicação por 3. Estes fatos tornar-

se-ão mais significativos quando estudarmos a divisão neste sistema.

Alguma vez você trabalhou num problema de matemática durante tanto tempo, sem conseguir

resolvê-lo? Você deve ter começado a desconfiar que talvez o problema não tivesse solução.

Isto pode parecer estranho, mas alguns problemas em matemática não tem solução para as

condições dadas.

Isto acontece com a divisão no sistema da aritmética de módulos não-primos.

Seja calcular 5 ÷ 1(mod 6), quer dizer, um número tal que, quando multiplicado por 1 dá o

produto 5. Em termos gerais, a ÷ b = c ,tal que c . b = a . Podemos ver na tabela de

multiplicação que 1 . 5 = 5, portanto, 5 ÷ 1 = 5.

Deparamos com uma situação diferente no problema 4 ÷ 2. A tabela de multiplicação mostra

claramente que 2 . 2 = 4 e também 2 . 5 = 4. Portanto, ambos os elementos 2 e 5 satisfazem

aos requisitos para o quociente 4 ÷ 2.

Mas há mais! No problema 5 ÷ 3, procuramos um quociente n, tal que n . 3 = 5. Olhe a tabela

de multiplicação para módulo 6. Não há um número n tal que n . 3 = 5. O problema 5 ÷ 3 não

tem resposta neste sistema.

Verifique as respostas na tabela da divisão módulo 6 completa absixo. Relacione estas

respostas com a tabela de multiplicação desenvolvida anteriormente. A divisão por zero é

CAPÍTULO 9

CONGRUÊNCIAS

134

excluída. Na tabela abaixo, e nas demais, estaremos considerando cada um dos elementos da

primeira linha como o dividendo e cada um dos elementos da primeira coluna como o divisor.

Dividendo

Divisor

Exemplo 9.4: Construa as tabelas de multiplicação e divisão módulo 8.

* 1 2 3 4 5 6 7

1 1 2 3 4 5 6 7

2 2 4 6 0 2 4 6

3 3 6 1 4 7 2 5

4 4 0 4 0 4 0 4

5 5 2 7 4 1 6 3

6 6 4 2 0 6 4 2

7 7 6 5 4 3 2 1

÷ 1 2 3 4 5 6 7

1 1 2 3 4 5 6 7

2 - 1; 5 - 2; 6 - 3; 7 -

3 3 6 1 4 7 2 5

4 - - - 1; 3; 5;7 - - -

5 5 2 7 4 1 6 3

6 - 3; 7 - 2; 6 - 1; 5 -

7 7 6 5 4 3 2 1

÷ 1 2 3 4 5

1 1 2 3 4 5

2 - 1; 4 - 2; 5 -

3 - - 1; 3; 5 - -

4 - 2; 5 - 1; 4 -

5 5 4 3 2 1

CAPÍTULO 9

CONGRUÊNCIAS

135

9.9. Potenciação em m

Exponenciação Rápida

Sejam a e n inteiros positivos. Temos que

é a expansão binária de n. Os coeficientes in são 0 ou 1. Portanto

Com isto, desenvolvemos a seguinte ideia:

1. Calculamos os quadrados sucessivos de i2a , 0 i k .

2. Determinamos na como o produto daqueles i2a para os quais

in =1 .

Observe que i+1 i2 2 2a (a )

Portanto, i+12a pode ser calculado de

i2a por uma elevação ao quadrado.

Vejamos um exemplo: Seja calcular 823144 (mod1037) .

Primeiro façamos a expansão binária de 823:

Escrevamos essa expansão na base dez:

Logo,

Agora, calculemos cada uma das potências por uma elevação sucessiva á potência 2.

144 144 (mod 1037)

1442 1033 (mod 1037)

1444 16 (mod 1037)

144823

= 144512

. 144256

. 14432

. 14416

. 1444 . 144

2 . 144

1

(1100110111)2 = 29 + 2

8 + 2

5 + 2

4 + 2

2 + 2

1 + 2

0

(1100110111)2 = 512 + 256 + 32 + 16 + 4 + 2 + 1

823 = (1100110111)2

ki

i i ii=0 i

i i

kn 2nn 2 2

i = 0 0 i k n 1

a a = (a ) a

ki

i

i=0

n = n 2

CAPÍTULO 9

CONGRUÊNCIAS

136

1448 256(mod 1037)

14416

205(mod 1037)

14432

545(mod 1037)

14464

443(mod 1037)

144128

256(mod 1037)

144256

205(mod 1037)

144512

545

Assim,

823144 545.205.545.205.16.1033.144 mod1037

= 766.766.117 mod1037

= 851.117 mod1037

= 99567 mod1037

= 15 mod1037

Conclusão: 144823

15 (mod 1037).

Voltaremos a falar deste assunto, depois de estudarmos o Algoritmo Chinês do Resto e o

Pequeno Teorema de Fermat.

Leitura: Teste de Primalidade Circular

O Teste de Primalidade Circular é, em termos gerais, semelhante à verificação tradicional de

primalidade de dividir um inteiro N > 1 pelos primos menores ou iguais ao piso da raiz

quadrada de N. A principal diferença entre o Teste Circular e o método tradicional é que,

neste caso, os divisores não são todos primos. Podemos também perceber uma diferença no

que diz respeito ao desempenho. O Teste Circular, também pode ser considerada um tipo

especial de crivo.

CAPÍTULO 9

CONGRUÊNCIAS

137

Procedimento

O primeiro passo consiste em escolher uma pequena quantidade dos primeiros primos.

Por exemplo, escolhe-se os primos 2, 3 e 5.

As classes de congruências que serão consideradas, terão módulo m igual ao produto desses

primos: m = 2 . 3 . 5 = 30

Em seguida, toma-se todos os inteiros positivos menores que 30 que não sejam múltiplos de

2, 3, e 5 ( relativamente primo com 30). Elaborando uma pequena tabela, e removendo os

múltiplos, obtém-se:

1 2 3 4 5 6

7 8 9 10 11 12

13 14 15 16 17 18

19 20 21 22 23 24

25 26 27 28 29 30

Os números que não foram marcados formam a base do crivo: 1, 7, 11, 13, 17, 19, 23, 29

(módulo 30). Cada um desses números indicará as classes de congruências módulo 30 que

serão utilizadas, ou seja, 1,7,11,13,17,19,23 e 29 .

Finalmente, semelhante ao método tradicional, para se verificar que um dado número N é

primo, divide-se N pelos primos iniciais, usados para gerar m, e pelos elementos das classes

de congruências (com exceção do 1) que sejam maiores que, ou iguais a N . Se N não for

divisível por nenhum desses termos, então N é primo.

Um exemplo prático

Para verificar se 3331 é primo, define-se o limite superior 3331 = 57. Em seguida,

divide-se 3331 pelos primos iniciais 2, 3, 5 e por todos os elementos das classes

1,7,11,13,17,19,23 e 29 (mod 30), que não ultrapassam o valor de 57, até se obter um

resto zero ou a divisão chegar ao último elemento das classes (mod 30) sem se obter uma

divisão exata, o que signficará que 3331 é primo.

Sejam as classes com seus respectivos elementos menores que 57:

1 {1,31,...}

7 {7,37,...}

11 {11,41,...}

13 {11,41,...}

17 {17,47,...}

19 {19,49,...}

23 {23,53,...}

29 {29,...} .

CAPÍTULO 9

CONGRUÊNCIAS

138

Assim, os elementos das classes, que não ultrapassam o valor 57, com exceção do 1, serão

usados para o teste de primalidade de 3331 e mais os primos 2, 3 e 5, ou seja, dividiremos

3331 pelos seguintes números :

2, 3, 5, 7, 11, 13, 17, 19, 23, 29, 31, 37, 41, 43, 47, 49, 53. Observe que nesta lista, diferente

do método tradicional, temos o número 49 que não é primo!

Como 3331 não divisível por nenhum desses números, então ele é primo.

Nota: É claro que um inteiro maior que 5, que é “candidato” a primo não

pode ser par e nem terminar em 5. Logo, a divisão por 2 e 5 devem ser

descartadas, bem como por 1. Qualquer candidato a primo,obrigatoriamente

deverá ser um ímpar que termine em 1 ou 3 ou 7 ou 9.

Uma forma indireta de crivo

É claro que o aparecimento de números compostos na sequências dos prováveis divisores de

N é uma mera consequência teórica. Antes que um N seja divisível por um composto, já terá

sido divisível por algum primo que seja um divisor deste composto. Uma vantagem deste

método é não ser preciso saber se um número é ou não primo para usá-lo como provável

divisor, como acontece no método tradicional.

Pelo Teorema de Dirichlet, existem infinitos primos em cada classe de congruência a

módulo m. O que podemos afirmar é que, na sequência dos números a serem usados, se

houver divisores de N, o menor deles é necessariamente um primo de uma das classes de

congruências. Isso nos dá um método indireto para saber se um número é primo, ou seja, uma

forma indireta de crivo.

Vamos exemplificar: Queremos saber se N = 3127 é composto. Se existirem números que

dividam N em alguma das classes a serem usadas, então o menor desses divisores é primo.

De fato, seja 3127 55 . Consideremos m = 2.3 = 6, logo as classes módulo 6 são:

Da classe 5 , temos que 53 é o menor divisor de 3127, logo 53 é primo.

1={7,13,19,25,31,37,43,49,55}

5={11,17,23,29,35,41,47,53}

CAPÍTULO 9

CONGRUÊNCIAS

139

EXERCÍCIOS

1) Verdadeiro (V) ou falso (F)

a) 91 0 (mod.7).

b) 3 + 5 + 7 5 (mod.10).

c) –2 2 (mod.8).

d) 112 1 (mod.3).

e) 17 9 (mod.2).

f) 42 -8 (mod.10).

2) Verdadeiro (V) ou falso (F)

a) x 3 (mod.5) x { .... –7, -2, 3, 8,

13 .... }.

b) 5 -1 (mod.6) e -1 -7 (mod.6)

5 -7 (mod.6).

3) Achar o menor inteiro positivo que represente a

soma:

a) 5 + 3 + 2 + 1 + 8 (mod. 7)

b) 2 + 3 – 1 + 7 – 2 (mod.4)

4) Sabendo-se que 1766 1066 (mod. m), achar

todos os possíveis valores do módulo m.

5) Exprimir que “n é ímpar” de três outras

maneiras.

6) Achar todos os inteiros x tais que 0 < x < 15 e

3x 6 (mod. 15)

7) Achar todos os inteiros x tais que 1 < x < 100 e

x 7 (mod. 17)

8) Sabendo-se que k 1 (mod. 4), mostrar que

6k + 5 3 (mod. 4)

9) Mostrar, mediante um exemplo, que a2 b

2

(mod.m) não implica a b (mod.m).

10) Mostrar que todo primo ímpar é congruente

módulo 4 a 1 ou 3.

11) Mostrar que todo primo maior que 3 é

congruente módulo 6 a 1 ou 5.

12) Mostrar que

a) 1110

1 (mod 100)

b) 31000

+ 3 é divisível por 28.

13) Mostrar que 41 divide 220

– 1.

14) Achar os restos das divisões de:

a) 250

por 7

b) 4165

por 7

c) 25100

+ 11500

por 3.

d) 35555

por 80.

e) 51000

por 126.

f) 3399300

+ 29 por 13

15) Mostrar que

a) 89 | 244

– 1.

b) 97 | (248

– 1)

c) 13| 270

+ 370

d) n|1 2 ... ( 1)n n nn ,, n ímpar.

16) Demonstrar que, se a b (mod. m) então

mdc(a, m) = mdc(b, m).

17) Mostrar, mediante um exemplo, que ak bk

(mod. m) e k j não implica aj bj.

18) Demonstrar as seguintes proposições:

a) Se a é um inteiro ímpar então a2 1 (mod.

8)

b) Se a é um inteiro qualquer, então a3 0, 1

ou 8 (mod. 9).

c) Se a é um inteiro qualquer, então a3 a

(mod. 6).

19) Mostre que 7 divide 22225555

+55552222

.

20) Determine o resto da divisão de 61987

por 37.

21) Prove que 7 divide 32n+1

+2n+2

para todo

natural n.

22) Determine o algarismo das unidades de

777 .

23) Determine os infinitos valores positivos de n

tal que 2n +27 seja divisível por 7.

24) Prove que 2k −5, k 4 nunca deixa resto 1

quando dividido por 7.

25) Prove que 4 divide 12233 . 455679+876533

26) Prove que 11. 31. 61 | 2015

- 1.

27) Calcular o resto da divisão do inteiro

n = (116 + 1717

)21

por 8.

28) Calcular o resto da divisão do inteiro

7100

+ 11100

por 13

CAPÍTULO 9

CONGRUÊNCIAS

140

29) Mostrar que o inteiro n = 1316

– 243

. 517

é

divisível por 3.

30) Mostrar que a expressão 3.5²n+1

+ 2³n+1

é

divisível por 17, sendo n um inteiro positivo

qualquer.

31) Mostrar que o número de 2³² + 1 é divisível

por 641.

32) Demonstrar que, se o inteiro positivo n não é

divisível por 4, então a soma: S = 1n + 2

n + 3

n +

4n é divisível por 5.

33) Determinar todos os inteiros positivos n para os

quais 2n + 1 é divisível por 3.

34) Mostrar que 63! 61! (mod 7)

35) Mostrar que não existe inteiro algum n que

verifique as condições: n 5 (mod 12) e n 4

(mod 15)

36) Na divisão do inteiro n por 10 o quociente é q e

o resto é r, tais que 2r – q 0 (mod 7).

Mostrar que 7 n.

37) Determinar quais dos seguintes conjuntos são

sistemas completos de restos módulo 4.

a) { -2, -1, 0, 1}

b) {0, 4, 8, 12}

c) { -13, 4, 17, 18 }

d) {–5, 0, 6, 22 }

38) Determinar quais dos seguintes conjuntos são

sistemas completos de restos módulo 6.

a) { 1, 2, 3, 4, 5}

b) {0, 5, 10, 15, 20, 25}

c) {-4, -3, -2, -1, 0, 1}

d) {17, -4, 6, 7, 10, 3}

39) Achar um sistema completo de restos {p1, p2,

...pi, ..., p7} módulo 7, tal que todo pi é primo.

40) Achar um sistema completo de restos módulo 7

formado só de múltiplos não negativos de 4.

41) Se n é um múltiplo positivo de 4, qual o resto

da divisão de 1n + 2

n + ... + 8

n + 9

n por 10?

42) Mostre que 121n – 25

n + 1900

n – (–4)

n é

divisível por 2000, para todo natural n..

43) Determinar o algarismo das unidades do

número 3100

.

44) Que valores de a e b tornam o número

30a0b03 divisível por 13?

45) Determine para que inteiros positivos n, 2n –1 é

divisível por 7.

46) Prove que 2n + 1 nunca é divisível por 7, para

qualquer inteiro positivo n.

47) Demonstre que para todo inteiro 0n , 17 é

um divisor de 34n + 2

+ 26n + 3

.

48) Determine o algarismo das dezenas do número

7999999

.

49) Determine o resto da divisão por 7 do número 2 3 10010 10 10 1010 10 10 ... 10 .

50) Mostre que, para todo n natural, temos

102n + 1

+ 1 0 (mod 11).

CAPÍTULO 10

FERMAT, WILSON E EULER

141

Capítulo 10

TEOREMAS DE FERMAT,

WILSON e EULER

Fermat foi um dos poucos matemáticos amadores famosos. Filho de um rico comerciante de

couro, pôde se dedicar completamente aos estudos. Por influência de sua mãe, descendente de

uma família de juristas, estudou leis na Universidade de Orleans e formou-se em advocacia.

Trabalhou durante toda sua vida na corte de justiça de Toulouse. Foi nomeado juiz e ocupava

os seus momentos de folga em diversos lazeres, entre os quais a poesia e a Matemática.

Seu interesse pela matemática iniciou-se em 1629 com o estudo dos trabalhos de Apolônio

(matemático grego, 260 A.D.) sobre curvas planas. Trocava correspondência com os maiores

matemáticos da época, como Torricelli, Roberval, Huyghens e Pascal, e, dessa forma relatava

suas descobertas. Jamais publicou seus trabalhos de nenhuma outra forma, mas o conteúdo

das cartas de Fermat é atualmente incluído em todos os textos usuais de teoria dos números.

Seu interesse na teoria dos números surgiu após ler o livro Aritmética de Diofanto

(matemático grego, 200 A.C.) e alguns dos problemas propostos por Fermat, nesta área, eram

tão difíceis que somente muitos anos mais tarde foram provados. Um desses problemas

afirmava que "todo número inteiro pode ser escrito como a soma de no máximo quatro

quadrados" e foi provado em 1770, pelo matemático francês Lagrange. Entretanto, seu

resultado mais famoso resistiu por mais de 350 anos e inspirou a publicação, em 1996, do

bestseller O Último Teorema de Fermat. Este teorema diz que “se n é um natural maior que 2,

então não existem números inteiros x, y e z que satisfaçam a equação xn + y

n = z

n”. Isto foi

provado definitivamente, em 1994, pelo matemático inglês Andrew Wiles (repare que no caso

n = 2 o teorema é satisfeito por todos os ternos pitagóricos, isto é, por inteiros que satisfaçam

o Teorema de Pitágoras).

10.1.

Teorema 10.1 (“Pequeno Teorema de Fermat” – PTF): se p é primo e se o MDC(p, a) = 1,

então:

Demonstração: consideremos os (p – 1) primeiros positivos de s, isto é, os inteiros

ap–1

1 (mod p)

CAPÍTULO 10

FERMAT, WILSON E EULER

142

Obviamente, nenhum desses (p – 1) inteiros é divisível por p e, além disso, dois quaisquer

deles são incongruentes módulo p, pois, se

fosse:

então, o fator comum a poderia ser cancelado, visto que o MDC(a, p) = 1, e teríamos:

o que é impossível, porque 0 < s – r < p.

Assim sendo, dois quaisquer dos (p – 1) inteiros a, 2.a, 3.a, ..., (p – 1).a divididos por p

deixam restos distintos, e por conseguinte cada um desses p – 1 inteiros é congruente módulo

p a um único dos inteiros 1, 2, 3, ..., p – 1, naturalmente numa certa ordem, multiplicando

ordenadamente essas p – 1 congruências, teremos:

ou seja,

Como o MDC (p, (p – 1)!) = 1, porque p é primo e p não divide (p – 1)!, podemos cancelar o

fator (p – 1)!, o que dá a congruência de Fermat:

Exemplo 10.1: seja o primo p = 7 e o inteiro a = 3 tais que 7 não divide 3, temos os p – 1 = 6

primeiros múltiplos positivos de 3: 3, 6, 9, 12, 15, 18. Nenhum desses 6 inteiros é divisível

por 7, todos são incongruentes módulo 7, e cada um deles é congruente módulo 7 a um único

dos inteiros 1, 2, 3, 4, 5, 6: 3 3 (mod 7), 6 6 (mod 7), 9 2 (mod 7), 12 5 (mod 7), 15

1 (mod 7), 18 4 (mod 7). Multiplicando ordenadamente essa 6 congruências, temos:

ou seja,

Como o MDC(7, 6!) = 1, podemos cancelar o fator comum 6!, que resulta em:

36 . 6! 6! (mod 7)

3 . 6 . 9 . 12 . 15 . 18 3 . 6 . 2 . 5 . 1 . 4 (mod p)

ap–1

1 (mod p)

ap–1

(p – 1)! (p – 1)! (mod p)

a . 2a . 3a . ... . (p – 1)a 1 . 2 . 3 . ... . (p – 1) (mod p)

r s (mod p), isto é p | (a – x)

r.a s.a (mod p), 1 r < s p – 1

a, 2 . a, 3 . a, ..., (p – 1) . a

CAPÍTULO 10

FERMAT, WILSON E EULER

143

Corolário 1: se p é um primo, então ap a (mod p), qualquer que seja o inteiro a.

Demonstração: se p divide a, então a 0 (mod p) e ap 0 (mod p), que implica em:

Se, ao invés disto, p não dividisse a, então pelo PTF:

Teorema 10.2: se p e q são primos distintos tais que ap a (mod q) e a

q a (mod p), então:

Demonstração: pelo corolário 1 e da hipótese, temos:

Portanto, p | (apq

– a), e q | (apq

– a), que implica em:

Pelo corolário 1, se p é primo, então 2p 2 (mod p), isto é, p | (2

p – 2).

Entretanto, a recíproca “se n | (2n – 2), então n é primo” não é uma proposição verdadeira, e os

inteiros positivos ímpares compostos que satisfazem essa condição são chamados de

pseudoprimos.

Por exemplo, 2341

2 (mod 341), isto é, 341 | (2341

– 2), onde o inteiro 341 = 11.31 é

composto. Com efeito, temos:

que implica em

Portanto,

2

11 2 (mod 31), e 2

31 2.(2

10)3 (mod 11) 2.(1)

3 (mod 11) 2 (mod 11)

210

1 (mod 11), e 210

1 (mod 31)

210

= 1024 = 3.11.31 + 1

(p.q) | (apq

– a), isto é, apq

a (mod p.q)

(aq)p a

q (mod p) a

q a (mod p) a

pq a (mod p)

(ap)q a

p (mod q) a

q a (mod q) a

pq a (mod q)

apq

a (mod p.q)

ap–1

1 (mod p), e ap a (mod p)

ap a (mod p)

36 1 (mod 7)

CAPÍTULO 10

FERMAT, WILSON E EULER

144

Como os inteiros 11 e 31 são primos distintos, temos, de acordo com o Teorema 2:

Congruência que mostra ser falsa a recíproca do PTF.

Logo, 341 é um pseudoprimo na base 2.

Definição 1 (Pseudoprimos): todo inteiro positivo composto n tal que 2n 2 (mod n) se

chama um pseudoprimo para a base 2.

Há um número infinito de pseudoprimos para a base 2, mas somente sete são menores que

2000, e todos ímpares:

Nota: O primeiro exemplo de um pseudoprimo par para a base 2 foi dado

pelo matemático americano LEHMER em 1950. O menor deles é 161038 = 2

. 73 .1103 e, em 1951, BEEGER mostrou a existência da infinidade de

pseudoprimos pares para a base 2.

Teorema 10.3: seja um inteiro a > 1 e um primo p > 2.

é um pseudoprimo na base a.

Demonstração: pelo PTF,

e se verifica facilmente que estes números são ímpares, de onde n 1 (mod 2.p), ou

n = 2.k.p + 1, para um k inteiro. Assim, como a2p

1 (mod n), temos:

Existe uma infinidade de pseudoprimos de base a > 1.

Façamos a demonstração no caso de a = 2.

an a

2kp+1 (mod n) (a

2p)k . a (mod n) a (mod n)

pa 1

a 1

pa 1

a 1 (mod p) 1 (mod p)

2p p p

2

a 1 a 1 a 1n

a 1 a 1 a 1

341 = 11 . 31 645 = 3 . 5 . 43 1387 = 16 . 73

561 = 3 . 11 . 17 1105 = 5 . 13 . 17 1729 = 7 . 13 . 19

211.31

2 (mod 11.31), isto é, 2341

2 (mod 341), ou que 2340

1 (mod 341)

CAPÍTULO 10

FERMAT, WILSON E EULER

145

Proposição 1: se n é um pseudoprimo de base 2, então 2n – 1 também é um pseudoprimo de

base 2.

Demonstração: dada a igualdade (xt – 1) = (x – 1).(x

(t–1) + x

(t–2) + ... + x

2 + x + 1), sejam d, k

tais que n = d.k, d > 1, k < n, e seja m = 2n – 1. Então m é composto.Para isto, basta

considerar na igualdade acima x = 2d e t = k. Por hipótese 2

n–1 1 (mod n) e, portanto, existe

k > 0 tal que 2n–1

– 1 = k.n. Assim,

Voltando à igualdade acima e fazendo x = 2n e t = 2.k, concluímos que 2

m–1 – 1 é um múltiplo

de m = 2n – 1, ou seja 2

m–1 1 (mod m) 2

m 2 (mod m). Concluímos assim que m é um

pseudoprimo de base 2.

Corolário 2: existe uma infinidade de pseudoprimos de base 2.

Demonstração: basta notar que 341 é um pseudoprimo de base 2 e aplicar sucessivamente a

proposição anterior.

Nota: O fato de existir uma infinidade de pseudoprimos em qualquer base

não implica a existência de uma infinidade de pseudoprimos.

EXERCÍCIOS

1) Verificar o PTF com a = 2 e p = 13.

2) Verificar o PTF com a = 3 e p = 17.

3) Verificar utilizando o PTF:

a) 1850

2 (mod 7)

b) (b) 19933

8 (mod 31)

4) Mostrar que 538

4 (mod 11) pelo PTF.

5) Mostrar que o inteiro 117 é composto usando

o PTF.

6) Achar o algarismo das unidades do inteiro 3400

com o auxílio do PTF.

7) Sendo a um inteiro, mostrar que n5 e n têm o

mesmo algarismo das unidades.

8) Demonstrar que 13 | (270

+ 370

) através do PTF.

9) Demonstra que 22225555

+ 55552222

é divisível

por 7 .

10) Achar o resto da divisão de 21137

por 17 com a

ajuda do PTF.

11) Mostrar que, se o MDC(a, 35) = 1, então

a12

1 (mod 35).

12) Demonstrar que, para todo inteiro a, se tem:

a) a13

a (mod 7)

b) a37

a (mod 13)

n n 1m 1 (2 2) 2.(2 1) 2km1 1 2 1 2 12 2

CAPÍTULO 10

FERMAT, WILSON E EULER

146

c) a21

a (mod 15)

d) a7 a (mod 42)

13) Demonstrar que, para todo inteiro positivo n, se

tem:

a) 22n

1 (mod 3)

b) 23n

1 (mod 7)

14) Achar todos os primos p tais que p | (2p + 1).

15) Demonstrar que 1105 | (31105

– 3) [sugestão:

PTF].

16) Demonstrar que 161038 é um pseudoprimo

para a base 2, i.e., que 161038 | (2161038

– 2).

17) Mostrar que 2047 é um pseudoprimo para a

base 2.

18) Mostre que 341 não é um pseudoprimo para a

base 3.

19) Mostrar que 561 é um pseudoprimo para a base

2, 3 e 5.

20) Demonstrar que se 7 não divide n, então

n6 1996 (mod 7).

21) Demonstrar que se MDC(n, 7) = 1 então,

7 | (n12

– 1).

22) Demonstrar que se MDC(a, 240) = 1, então

240 é um divisor de a4 – 1.

23) Demonstrar que todo primo maior que 5 divide

um inteiro formado só de algarismos 1.

24) Demonstrar que para todo inteiro positivo n,

temos que (n3 – n).(5

8n+4 + 3

4n+2) é um múltiplo

de 3804.

25) Prove que 15 divide 5 33 5 7n n n .

26) Prove que 91 divide 12 12a b , onde a e b são

relativamente primo com 91.

10.2. TEOREMA DE WILSON

Teorema 10.4 (Teorema de Wilson): se p é um primo, então (p – 1)! –1 (mod p).

Demonstração: o teorema é verdadeiro para p = 2 e para p = 3, pois:

de modo que vamos supor p 5. Consideremos a congruência linear a.x 1 (mod p), onde a é

um dos (p – 1) primeiros inteiros positivos 1, 2, 3,..., p – 1 de modo que o MDC(a, p) = 1.

Nestas condições, existe um único inteiro positivo a’, com 1 a’ p – 1, tal que

Como p é primo, tem-se que a = a’ se e somente se a = 1, ou a = p – 1, visto que

a2 1 (mod p) implica em (a – 1).(a + 1) 0 (mod p) e, portanto,

a – 1 0 (mod p), ou (a + 1) 0 (mod p)

a.a’ 1 (mod p)

(2 – 1)! = 1! = 1 –1 (mod 2)

(3 – 1)! = 2! = 2 –1 (mod 3)

CAPÍTULO 10

FERMAT, WILSON E EULER

147

isto é, a = 1 ou a = p – 1.

Nota: Este teorema foi descoberto primeiramente por John Wilson (1741 - 1793), estudante

do matemático inglês Edward Waring. Waring anunciou o teorema em 1770, embora nenhum

deles tenha conseguindo prová-lo. Lagrange deu a primeira prova em 1773.

Há uma evidência que Leibniz estava ciente do resultado um século antes, mas nunca o

publicou.

Exemplo 10.2: Com p = 13, existe um único inteiro positivo a’, com 1 a’ p – 1, tal que

a 1 2 3 4 5 6 7 8 9 10 11 12

a' 1 7 9 10 8 11 2 5 3 4 6 12

Omitindo os inteiros 1 e p – 1, com os p – 3 restantes: 2, 3, ..., p – 2 podemos formar p 3

2

pares (a, a’), com a a’ tais que a.a’ 1 (mod p). Então, multiplicando ordenadamente todas

essas p 3

2 congruências, obtemos a congruência:

ou, multiplicando por (p – 1):

que é a própria congruência de Wilson.

Com p = 13, por exemplo, os 10 inteiros 2, 3,..., 11 dão lugar a 5 pares tais que o produto dos

inteiros de cada par é congruente a 1 módulo 13:

multiplicando ordenadamente essas cinco congruências, obtemos:

e, portanto,

(2.7) . (3.9) . (4.10) . (5.8) . (6.11) 11! (mod 13) 1 (mod 13)

2.7 1 (mod 13)

4.10 1 (mod 13)

6.11 1 (mod 13)

3.9 1 (mod 13)

5.8 1 (mod 13)

(p – 1)! (p – 1) (mod p) –1 (mod p)

2 . 3 . ... . (p – 2) 1 (mod p), ou (p – 2)! 1 (mod p)

a.a’ 1 (mod p)

CAPÍTULO 10

FERMAT, WILSON E EULER

148

isto é,

Recíproca do Teorema de Wilson: “se (n – 1)! –1 (mod p), então n é primo”.

Demonstração: a recíproca é verdadeira. Suponhamos, por absurdo, que o inteiro n é

composto. Então, n tem um divisor d tal que 1 < d < n, de modo que d é um dos fatores do

produto:

e, portanto, d | (n – 1)!. Mas, por hipótese n | (n – 1)! + 1, e como d | n, segue-se que:

o que é absurdo, visto que d > 1. Logo, n não possui divisores menores que ele mesmo e

diferentes de 1, ou seja, n é primo.

Nota: Este teorema recíproco dá um critério para se reconhecer se um inteiro dado é primo.

Mas, do ponto de vista prático, no momento ainda é de interesse apenas teórico, pois o cálculo

de fatoriais exigiria muito tempo.

Teorema 10.5 (Teorema de Leibniz): um inteiro n > 1 é primo, se e somente se,

Demonstração:

( ) suponhamos que o inteiro n > 1 é primo. Então, pelo teorema de Wilson:

obviamente,

portanto,

( ) reciprocamente, se (n – 2)! 1 (mod n), então:

Logo, pelo recíproco do Teorema de Wilson, o inteiro n é primo.

(n – 1)! – (n – 2)! –1 (mod n)

(n – 2)! 1 (mod n)

(n – 1)! = (n – 1). (n – 2)! – (n – 2)! (mod n)

(n – 1)! –1 (mod n)

(n – 2)! 1 (mod n)

d | (n – 1)! + 1, e d | 1

1 . 2 . 3 . ... . (n – 1) = (n – 1)!

(p – 1)! –1 (mod p), com p = 13

12! 12 (mod 13) –1 (mod 13)

CAPÍTULO 10

FERMAT, WILSON E EULER

149

EXERCÍCIOS

1) Verificar o Teorema de Wilson para p = 5 e para p = 7.

2) Mostrar que 11, 13, 17 e 19 são primos usando o Teorema de Wilson.

3) Mostrar que 8 é composto usando o Teorema de Wilson.

4) Achar o resto da divisão de 15! por 17.

5) Mostrar que 18! + 1 0 (mod 437).

6) Sendo p um primo ímpar, demonstrar que 2.(p – 3)! –1 (mod p).

7) Verificar o Teorema de Leibniz com o primo p = 13.

8) Usando o Teorema de Leibniz, mostrar que 17 é primo.

9) Formar com os inteiros 2, 3, 4, ... , 21, todos os pares de a e b tais que a.b 1 (mod 23).

CAPÍTULO 10

FERMAT, WILSON E EULER

150

10.3. TEOREMA DE EULER

Leonhard Euler (15/04/1707 – 18/07/1783) foi um matemático e físico de origem suíça.

Nasceu na Basiléia, filho do pastor calvinista Paul Euler (lê-se “óiler”) que, desprezando seu

prodigioso talento matemático, determinou que ele estudasse Teologia e seguiria a carreira

religiosa. Daniel e Nikolaus Bernoulli convenceram o pai de Euler a permitir que seu filho

trocasse o hábito pelos números.

Euler, logo após, deixou a Suíça, indo para os palácios de Berlim e São Petersburgo, onde

passou os mais criativos anos de sua vida. Os governos da Europa estavam interessados em

usar a Matemática para resolver problemas práticos e competiam entre si para empregar os

melhores cérebros.

Durante sua vida resolveu enorme quantidade de problemas, da navegação às finanças, da

acústica à irrigação. A solução de tais problemas, que atendiam aos reclamos do mundo

prático, não o entediava, principalmente porque cada novo trabalho inspirava-o para criar uma

Matemática nova e engenhosa. Era capaz de escrever vários trabalhos em um único dia com

os cálculos completos e prontos para serem publicados.

Conseguiu provar o uma conjectura de Fermat relativa aos números primos. Fermat afirmava

que o primeiro tipo de número primo sempre era representado pela soma de números ao

quadrado enquanto que o segundo jamais o seria. Esta propriedade dos números primos é

extremamente simples, porém, ao tentar provar que isto é uma verdade para qualquer número

primo, torna-se extremamente difícil. Em 1749, depois de sete anos de trabalho e quase cem

anos após a morte de Fermat, conseguiu apresentar esta prova.

Ao final de vida estava completamente cego. Aliás, já era cego de um olho desde os vinte

anos, o que não o perturbara em nada. Certa vez disse: “agora eu terei menos distrações”. Aos

sessenta anos foi acometido por catarata que o cegou completamente. Apesar disto, continuou

a produzir Matemática por mais sete anos. Seu imenso conhecimento permitia-lhe criar

conceitos sem ter que colocá-los no papel e sua memória fenomenal permitia-lhe usar seu

cérebro como uma biblioteca mental.

Passou os anos finais de sua vida na Rússia, então sob a proteção de Catarina, a Grande.

Definição 2: chama-se função aritmética toda função f definida no conjunto dos naturais e

com valores no conjunto dos inteiros, i.e., toda função f de em (f : ) .

Definição 3: uma função aritmética f se diz multiplicativa se f(r.s) = f(r) . f(s), para todo par

de inteiros positivos r e s, tais que o MDC(r, s) = 1.

CAPÍTULO 10

FERMAT, WILSON E EULER

151

10.4. FUNÇÃO TOTIENT (n)

Definição 4: Chama-se Função Totient a função aritmética assim definida para todo inteiro

positivo n:

(n) = quantidade de inteiros positivos menores que n relativamente primos a n.

Em outros termos, (n) é igual ao número de elementos do conjunto

Observação: (1) = 1, pois MDC(1, 1) = 1.

Exemplo 10.3: (30) = 8 e (12) = 4.

n 1 2 3 4 5 6 7 8 9 10

(n) 1 1 2 2 4 2 6 4 6 4

Tabela de (n) para os dez primeiros inteiros positivos.

Teorema 10.6: A Função Totient é uma função aritmética multiplicativa.

Demonstração: sejam r e s dois inteiros positivos tais que o MDC(r, s) = 1.

Cumpre demonstrar que (r.s) = (r) . (s).

A proposição é verdadeira se r ou s é igual a 1, pois, temos:

Suponhamos, pois, que r > 1 e s >1. Neste caso, todos os inteiros de 1 a r.s podem ser

dispostos em r colunas com s inteiros em cada uma delas, do seguinte modo:

1 2 ... h ... r

r + 1 r + 2 ... r + h ... 2.r

2.r + 1 2.r + 2 ... 2.r + h ... 3.r

... ... ... ... ... ...

(s – 1).r + 1 (s – 1).r + 2 ... (s – 1).r + h ... s.r

Como o MDC(q.r + h, r) = MDC(h, r), os inteiros da h-ésima coluna são primos com r, se e

somente se, h é primo com r. Além disto, como na primeira linha o número de inteiros que são

primos com r é igual a (r), segue que há somente (r) colunas formadas com inteiros que são

todos os primos com r. Por outro lado, em cada uma destas (r) colunas existe precisamente

(s) inteiros que são primos com s, porque na progressão aritmética:

h, r + h, 2.r + h, ..., (s – 1).r + h

Se r = 1, (r.s) = (1.s) = (s) = 1 . (s) = (1) . (s) = (r) . (s)

Se s = 1, (r.s) = (r.1) = (r) = (r) . 1 = (r) . (1) = (r) . (s)

#{ x | 1 x < n, MDC(x, n) = 1 }

CAPÍTULO 10

FERMAT, WILSON E EULER

152

onde o MDC(r, h) = 1, o número de termos que são primos com s é igual a (s).

Assim sendo, o número total de inteiros que são primos com r e s, isto é, que são primos a r.s,

é igual a (r). (s), e isto significa que (r.s) = (r) . (s).

10.5 – CÁLCULO DE (n)

Teorema 10.7: seja p um primo, então (p) = p – 1.

Demonstração: ( ) Se n > 1 é primo, então cada um dos inteiros positivos menores que n é

primo com n e, portanto, (n) = n – 1. ( ) Reciprocamente, se (n) = n – 1, com n > 1, então

n é primo, pois, se n fosse composto, teria pelo menos um divisor d tal que 1 < d < n, de modo

que pelo menos dois dos inteiros 1, 2, 3,..., n não seriam primos com n, d e n, isto é, (n) n –

2. Logo, n é primo.

Teorema 10.8: se p é primo e se k é um inteiro positivo, então

Demonstração: obviamente, o MDC(pk, n) = 1, se e somente se, p não divide n, e entre 1 e p

k

existem pk–1

inteiros que não são primos com pk, que são todos os múltiplos de p:

segue-se que o conjunto {p, 2p, 3p, ..., p2, ..., p

3, ..., p

k} contém exatamente p

k – p

k–1 inteiros

que são relativamente primos a pk, de modo que pela definição da função (n) de Euler,

temos:

Teorema 10.9: se n = 31 2 rkk k k

1 2 3 rp . p . p . ... . p é a decomposição canônica do inteiro n > 1,

então:

ou seja,

(n) = 1

1 1

1( ) . (1 )i i

r rk k

i ii i

i

p p np

(n) = 1 1 2 2 r rk k 1 k k 1 k k 1

1 1 2 2 r r

1 2 r

1 1 1p p . p p . ... . p p n. 1 . 1 . ... . 1

p p p

(pk) = p

k – p

k–1

p, 2.p, 3.p, ..., p2, ..., p

3, ..., p

k

(pk) = p

k – p

k–1 = p

k .

11

p

CAPÍTULO 10

FERMAT, WILSON E EULER

153

Demonstração: usaremos o Teorema da Indução Matemática sobre r, número de fatores

primos distintos de n. A proposição é verdadeira para r = 1.

Suponhamos, então, a proposição verdadeira para r = i.

Como o 1 2 r r+1k k k k

1 2 r r+1MDC p . p . ... . p , p 1, e (n) é uma função aritmética multiplicativa,

temos:

ou seja,

ou, à vista da hipótese de indução:

e isto significa que a proposição é verdadeira para r = i + 1. Logo, a proposição é verdadeira

para todo inteiro positivo r.

Teorema 10.10: Para todo inteiro positivo n > 2 , (n) é um inteiro par

Demonstração:

Se n é uma potência de 2, isto é, 2kn , com 2k , então:

que é um inteiro par.

Se, ao invés, n não é uma potência de 2, então n é divisível por um primo impar p, isto é: kn p m , onde 1k e o ( , ) 1kmdc p m

E como ( )n é uma função aritmética multiplicativa temos:

que é também um inteiro par, por que 2 | ( 1)p .

Assim, ( )n é um inteiro ímpar somente para n = 1 e n = 2:

(1) (2) 1

1( ) ( ) ( ) ( 1) ( )k kn p m p p m

11( ) (2 ) 2 (1 ) 2

2

k k kn

1 2 r r+1 1 1 2 2 r r r+1 r+1k k k k k k 1 k k 1 k k 1 k k 1

1 2 r r+1 1 1 2 2 r r r+1 r+1p . p . ... . p . p p p . p p . ... . p p . p p

1 2 r r+1k k k k

1 2 r r+1p . p . ... . p . p 1 2 rk k k

1 2 rp . p . ... . p . r+1 r+1k k 1

r+1 r+1p p

1 2 r r+1k k k k

1 2 r r+1p . p . ... . p . p 1 2 rk k k

1 2 rp . p . ... . p . r+1k

r+1p

CAPÍTULO 10

FERMAT, WILSON E EULER

154

Teorema 10.11 (Teorema de Euler): Se n é um inteiro positivo e se MDC(a, n) = 1, então

Provaremos, primeiramente, o seguinte lema:

“Sejam a e n > 1 inteiros tais que o MDC(a, n) = 1. Se a1, a2, ..., a (n) são inteiros positivos

menores que n e que são relativamente primos com n, então cada um dos inteiros a.a1, a.a2, ...,

a.a (n) é congruente módulo n a um dos inteiros a1, a2, ..., a (n) (não necessariamente nesta

ordem em que aparecem).”

Demonstração: os inteiros a.a1, a.a2, ..., a.a (n) são mutuamente incongruentes módulo n, pois,

se a.ai a.aj (mod n), com 1 i j (n), como o MDC(a, n) = 1, podemos cancelar o fator

comum a, o que dá ai aj (mod n) n | (aj – ai). Isto é impossível, visto que (aj – ai) < n.

Por outro lado, como o MDC(ai, n) = i, i = 1, 2, ..., (n) e o MDC(a, n) = 1, segue que o

MDC(a.ai, n) = 1. Mas, pelo algoritmo da divisão, a.ai = n.qi + ri, 0 ri < n, que implica em

portanto, MDC(ri, n) = MDC(a.ai, n) = 1, de modo que ri é um dos inteiros a1, a2, ..., a (n), isto

é, cada um dos inteiros a.a1, a.a2, ..., a.a (n) é congruente módulo n a um único dos inteiros a1,

a2, ..., a (n), em uma certa ordem.

Provemos, agora, o Teorema de Euler:

A proposição é verdadeira para n = 1, pois a(1)

1 (mod 1). Suponhamos, pois, n > 1, e sejam

a1, a2, ..., a (n) os inteiros positivos menores que n e relativamente primos a n.

Como o MDC(a, n) = 1, então, pelo Lema acima, os inteiros a.a1, a.a2, ..., a.a (n) são

congruentes módulo n aos inteiros a1, a2, ..., a (n), em uma certa ordem:

a.a1 a1*, a.a2 a2

*, ..., a.a (n) a (n)

*

onde a1*, a2

*, ..., a (n)

* denotam os inteiros a1, a2, ..., a (n) em uma certa ordem.

Multiplicando ordenadamente todas essas (n) congruências, obtemos:

(a.a1).(a.a2). ... .(a. a (n)) a1* . a2

* . ... . a (n)

* (mod n)

ou seja,

a(n)

. (a1 . a2 . ... . a (n)) a1 . a2 . ... . a (n) (mod n)

Cada um dos inteiros a1, a2, ..., a (n) é relativamente primo a n, de modo que podem ser

sucessivamente cancelados, o que dá a congruência de Euler:

a(n)

1 (mod n)

Nota: se p é um primo, (p) = p – 1, e se o MDC(a, p) = 1, então:

a(p)

ap–1

(mod p) 1 (mod p)

a.ai ri (mod n), com 0 ri < n

a(n)

1 (mod n)

CAPÍTULO 10

FERMAT, WILSON E EULER

155

que é a congruência de Fermat. Assim, o Teorema de Euler é uma generalização do teorema

de Fermat.

Corolário 3: Se m > 1, k 0, n 0 e a um inteiro qualquer são tais que, MDC(a, m) = 1 e

k n (mod (m)) então, ak a

n (mod m).

Demonstração: basta considerar o caso em que k > n. Como k n (mod (m)) existe q 1

tal que k – n = q . (m) e, portanto,

ak = a

k–n . a

n = a

q. (n) . a

n = (a

(n))q . a

n a

n (mod m)

Exemplo 10.4: sejam a = 5, m = 6, k = 8 e n = 2. Temos (6) = 2, e 8 2 (mod 2).

Como 52 1(mod 6), então 5

8 1 (mod 6) e desta forma, 5

8 5

2 (mod 6).

10.6. RESOLUÇÃO DE CONGRUÊNCIAS LINEARES PELO TEOREMA DE EULER

A congruência linear a.x b (mod m) no caso em que o MDC(a, m) = 1, admite uma única

solução módulo m, que se pode facilmente obter usando o Teorema de Euler. Realmente,

temos:

portanto,

Como o MDC(a,m) = 1, podemos cancelar o fator comum a, que resulta em:

Exemplo 10.5: no caso da congruência linear 3.x 5 (mod 8), onde o MDC(3, 8) = 1, temos:

x 5. 3(8)–1

(mod 8) 5.34–1

(mod 8) 5.27 (mod 8) 135 (mod 8) 7 (mod 8)

Em particular,

a.x 1 (mod n) x a(n)

(mod n)

determina um inverso de a módulo n.

Exemplo 10.6: queremos determinar o inverso de 7 módulo 11, ou seja, queremos resolver a

congruência linear 7.x 1 (mod 11).

Queremos determinar o menor inteiro positivo em 11 que satisfaça a equação

x 7(11) – 1

(mod 11) 710–1

(mod 11) 79 (mod 11) 8 (mod 11)

assim, temos que x = 8 é a menor solução positiva em 11 para o problema.

x b.a(m) – 1

(mod m)

a.x b.a(m)

(mod m)

ak = a

k–n . a

n = a

q. (n) . a

n = (a

(n))q . a

n a

n (mod m)

CAPÍTULO 10

FERMAT, WILSON E EULER

156

10. 7. RESOLUÇÃO DA EQUAÇÃO (n) = m.

Vejamos agora como resolver a equação (n) = m, quando m é um inteiro positivo dado.

Sabemos que não existe solução se é m é um ímpar maior que 1. Em geral não se conhece

uma fórmula para se resolver essa equação, quando m é qualquer inteiro positivo par. Mesmo

assim, existe um método com o qual podemos determinar todas as soluções dessa equação.

Seja i

rk

i

i = 1

pn um inteiro qualquer, decomposto em fatores primos, que satisfaça a equação

Façamos

Assim,

Todas essas equações estabelecem três condições para os id , que nos permitem determinar

os valores de n que satisfazem a equação (n) = m:

1) cada 1id é um primo;

2) cada id é um divisor positivo de m;

1

1

i

rk

i ii

p d m ,

1

i

rk ii

ii

dp m

p

,

. i

i

dn m

p

,

1

1

.r

ir i

ii

mn p

d

1i id p , 1,2,...,i r

(n) = m. Como (n) = 1

1( )i i

rk k

i ii

p p , então

(n) = 1 1

1 1( ) ( 1)i i i

r rk k k

i i i ii i

p p p p m .

CAPÍTULO 10

FERMAT, WILSON E EULER

157

3)

1

r

ii

m

d é um inteiro positivo que contém somente fatores primos incluídos em

1

r

ii

p .

Exemplo 10.7: Resolver a equação 324 2 .3x

Os divisores positivos de 24 são 1,2,3,4,6,8,12 e 24. Sem dúvida, os possíveis id são os

inteiros 1,2,4,6 e 12 já que eles são os únicos tais que 1id é um primo. Como todos os

produtos de quatro ou cinco dos id são maiores que 24,podemos eliminar esses produtos

imediatamente; quer dizer,

1

24k

i

i

d

não e um inteiro quando quatro o mais id são

considerados. As possíveis restantes expressões para 1

k

i

i

d são os seguintes produtos abaixo:

1, 2 , 4, 6 , 12 , 1.2 , 1.4, 2.4

2.6 , 2. 12 , 4.6 , 4.12 , 6.12 ,1.2.4, 1.2.6,

1.2.12, 1.4.6 , 1.4.12 ,1.6.12 , 2.4.6 ,

2.4.12, 2.6.12 , e 4.6.12.

Os oito últimos produtos destacados, podem ser eliminados, já que em cada caso

1

24k

i

i

d

não é um inteiro ; os sete primeiros produtos destacados, podem ser eliminados ,já que em

cada caso

1

24k

i

i

d

, contém fatores primos não contidos em 1

( 1)k

i

i

d , isto é, 1

k

i

i

p .

CAPÍTULO 10

FERMAT, WILSON E EULER

158

Agora, os produtos restantes para 1

k

i

i

d conduzem as soluções:

1

k

i

i

d

1

24k

i

i

d

1

k

i

i

p

1

1

24.

k

iki

i

i

p

d

1.2

1.6

1.12

2.4

2.12

4.6

1.2.4

1.2.6

1.2.12

1.4.6

22. 3

22

2

3

1

1

3

2

1

1

2.3

2.7

2.13

3.5

3.13

5.7

2.3.5

2.3.7

2.3.13

25.7

23.3

2

23.7

22.13

32.5

3.13

5.7

2.32.5

22.3.7

2.3.13

2.5.7

72

56

52

45

39

35

90

84

78

70

Em conseqüência, as soluções da equação ( ) 24x são 35,39,45,52,56,70,72,78,84 e 90.

Exemplo 10.8: Usando o fato de que a função Totient é multiplicativa, encontrar as 10

soluções da equação ( ) 24x .

Consideremos 1 2 3( ) ( ). ( ). ( )... 24x n n n ,onde

1 2 3, ,n n n são relativamente primos entre

si dois a dois. Investigando uma lista de valores de ( )n , podemos determinar conjuntos de

fatores 1 2 3( ), ( ), ( )n n n ,... onde

1 2 3, ,n n n ,... são relativamente primos dois a dois cujos

produtos são iguais a 24. Os produtos 1 2 3. . ...n n n representam soluções da equação

Como

Produto mdc Solução

(3) . (13) = 2.12 (3,13) = 1, x = 39;

(4) . (13) = 2.12 (4,13) = 1, x = 52;

(5) . (7) = 4.6 (5,7) = 1, x = 35;

(5) . (9) = 4.6 (5,9) = 1, x = 45;

(6) . (13) = 2.12 (6,13) = 1, x = 78;

(7) . (8) = 6.4 (7,8) = 1, x = 56;

(7) . (10) = 6.4 (7,10) = 1, x = 70;

(7) . (12) = 6.4 (7,12) = 1, x = 84;

(8) . (9) = 4.6 (8,9) = 1, x = 72;

(9) . (10) = 6.4 (9,10) = 1, x = 90;

x

24.x

CAPÍTULO 10

FERMAT, WILSON E EULER

159

Assim, as dez soluções da equação são 35, 39, 45, 52, 56, 70, 72, 78, 84 e 90.

Este resultado esta de acordo com exemplo anterior.

Note que para usar este método e necessário conhecer primeiramente o número de soluções

desejadas ou limitar os valores das soluções desejadas com o fim de decidir quando termina a

investigação.

Kevin Ford provou em 1999 que para todo número inteiro k ≥ 2 há um número m para o qual

a equação ( )x m tem exatamente k soluções; este resultado já haviam sido conjecturado

por Sierpiński. No entanto, nenhum desses m é conhecido para k=1, e de acordo com a

conjectura da função totiente de Carmichael acredita-se que neste caso esse m não existe.

10.8 – VALÊNCIA DA FUNÇÃO TOTIENTE: ( )N m .

A função ( )N m , chamada de Valência da Função Totiente, é definida como o número de

inteiros positivos k, tal que (k) = m, também chamada de multiplicidade de m.

Exemplo 1: Temos que (8) 5N , porque existem apenas 5 inteiros, k = 15, 16, 20, 24 e 30,

tal que (k) = 8. A tabela abaixo mostra os valores de ( )N m para n 12.

m ( )N m k, tal que (k) = m

1 2 1, 2

2 3 3, 4, 6

4 4 5, 8, 10, 12

6 4 7, 9, 14, 18

8 5 15, 16, 20, 24, 30

10 2 11, 22

12 6 13, 21, 26, 28, 36, 42

Pode-se provar que existem inteiros pares m > 1 que não são valores assumidos pela função

Totiente, como por exemplo (14) 0N . Andrzej Schinzel provou em 1956, que para todo

1t , o valor 2. 7t não é valor da função Totiente. Veja que (26) 0N e, 26 não é da

forma 2. 7t . Mas, em 1976, Nathan Mendelsohn provou a existência de uma infinidade de

números primos p, tais que para todo 1t , o valor 2 .t p não é assumido pela função Totiente

e, 26 é um número dessa forma.

Observe os seguintes valores da função Totiente:

24x

CAPÍTULO 10

FERMAT, WILSON E EULER

160

m (m)

2 1!

4 2!

9 3!

35 4!

231 5!

Himanshu Gupta provou em 1950, que para todo 1t existe m tal que (m) = t!

EXERCÍCIOS

1. Calcular (420), (1001), (5040) e (8316).

2. Verificar que (n + 2) = (n) + 2, para n = 12,

14, 20.

3. Verificar que (n) = (n + 1) = (n + 2), para n

= 5186.

4. Verificar que (3k . 568) = (3

k . 638).

5. Verificar que (n) é uma função aritmética

multiplicativa para n = 144.

6. Verificar o Teorema de Euler com a = 3 e n =

10; a = 7 e n = 12.

7. Verifique que:

a) 35711 59(mod96)

b) 313046 84(mod101)

c) 420523 67(mod77)

d) 1210120 20(mod57)

e) 85017 100(mod143)

8. Achar os dois últimos algarismos da direita do

inteiro 3256

.

9. Calcular o menor inteiro positivo n tal que n

71015

(mod 31).

10. Achar o algarismo das unidades do inteiro 3145

.

11. Usando o Teorema de Euler, resolver as

seguintes congruências lineares:

(a) 5x 7 (mod 12)

(b) 2x 3 (mod 9)

(c) 7x 1 (mod 10)

(d) 8x 4 (mod 5)

(e) 2x 1 (mod 17)

(f) 5x – 3 (mod 8)

12. Demonstrar que se n é um inteiro positivo

ímpar, então:

a) (2.n) = (n)

b) (4n) = 2. (n)

13. Resolva em as seguintes equações:

a) (n) = 12

b) (n) = 18

c) (n) = 20

d) (n) = 30

e) (n) = 4!

CAPÍTULO 10

FERMAT, WILSON E EULER

161

10.9. TEOREMA CHINÊS DO RESTO (TCR)

O livro “Manual Aritmético do Mestre Sol” foi escrito por Sun Zi Suanjing (ou Sun Tzu Suan

Ching), provavelmente entre 280 d.C. a 483 d.C. O livro está dividido em 3 capítulos.

O 1º capítulo, contém apenas dois problemas que dizem respeito sobretudo a métodos para

fazer multiplicações e divisões, utilizando “palitinhos chineses”. O segundo capítulo, contém

28 problemas, apresenta métodos para o cálculo com frações, extração da raiz quadrada,

determinação de áreas e volumes, proporções e regra de três simples. O terceiro capítulo

contém 36 problemas aritméticos.

No problema 26 (também conhecido como “problema do Mestre Sun”) do 3° capítulo, Sun

Tzu utiliza pela primeira vez o chamado Teorema Chinês do Resto (TCR). O problema está

enunciado abaixo:

“Temos coisas, mas não sabemos quantas; se as contarmos de três em três, o resto é 2; se as

contarmos de cinco em cinco, o resto é 3; se as contarmos de sete em sete, o resto é 2.

Quantas coisas temos?”

Em notação moderna, este problema equivale a procurar as soluções do seguinte sistema de

congruências:

x 2 (mod 3)

x 3 (mod 5)

x 2 (mod 7)

Resolver problemas como esse é um dos objetivos deste tópico.

Teorema 10.11 (Teorema Chinês do Resto – TCR): sejam m1, m2, ..., mk inteiros positivos

primos entre si dois a dois, isto é, tais que o MDC(mi, mj) = 1 se i j. Nestas condições, o

sistema de congruências lineares:

1 1

2 2

k k

x a (mod m )

x a (mod m )

...

x a (mod m )

tem única solução módulo m = m1, m2, ..., mk , dada por:

x a1.M1.x1 + a2.M2.x2 + ... + ak.Mk.xk (mod m)

Demonstração: para cada k = 1, 2, 3, ..., r, seja:

Seja Mk 1 2 r

k k

m . m . ... . mm

m m. Como os inteiros mi são todos primos entre si dois a dois, o

MDC(Mr, mr) = 1, de modo que a congruência linear Mr . x 1 (mod Mr) tem única

solução x xr (mod mr)

Posto isto, vamos mostrar que o inteiro x a1.M1.x1 + a2.M2.x2 + ... + ak.Mk.xk (mod m) é uma

solução do sistema considerado.

CAPÍTULO 10

FERMAT, WILSON E EULER

162

Com efeito se i r, então mr | Mi e Mi 0 (mod mr), que implica em:

x a1.M1.x1 + a2.M2.x2 + ... + ak.Mk.xk (mod m)

Para demonstrar a unicidade desta solução, suponhamos que x1 é uma outra solução

qualquer do sistema considerado. Então:

e, portanto, mr | (x – x1), r = 1, 2, ..., k.

Mas, o MDC(mi, mj) = 1 implica em (m1 . m2 . … . mk) | (x – x1), isto é, m | (x – x1) e x x1

(mod m), com o que termina a demonstração do TCR.

Teorema 10.12: Sejam m1, m2, ..., mk inteiros positivos primos entre si dois a dois, e sejam

a1, a2, ..., ak inteiros tais que MDC(ar, mr) = 1 para r = 1, 2, ..., k. Nestas condições, o

sistema de congruências lineares:

1 1 1

2 2 2

k k k

a .x b (mod m )

a .x b (mod m )

...

a .x b (mod m )

tem única solução módulo m = m1 . m2 . ... . mk.

Demonstração: como o MDC(ar, mr) = 1, a congruência linear ar.x 1 (mod mr) tem única

solução x ar (mod mr), de modo que:

e, portanto, é equivalente a congruência x ar . br (mod mr).

Assim sendo, o sistema considerado é equivalente ao seguinte sistema de congruências

lineares:

1 1 1

2 2 2

k k k

x a .b (mod m )

x a .b (mod m )

...

x a .b (mod m )

o qual tem, pelo TCR, uma única solução módulo m = m1 . m2 . ... . mk:

xk . Mk 1 (mod mk) (m) 1

k k kx M (mod m )

onde:

ar.ar 1 (mod mr), e ar.ar.x x (mod mr)

x ar (mod mr) x1 (mod mr), r = 1, 2, ..., k.

CAPÍTULO 10

FERMAT, WILSON E EULER

163

(i) Mk k

m

m, k = 1, 2, 3, ...

(ii) xk . Mk 1 (mod mk) (m) 1

k k kx M (mod m ) , ou seja, xk é o inverso de Mk módulo

mk.

Exemplo 10.7: utilizando o TCR, resolver o sistema de congruências lineares:

x 8 (mod 5)

x 5 (mod 3)

x 11 (mod 7)

x 2 (mod 4)

Resolução: os módulos 5, 3, 7 e 4 das congruências lineares que formam o sistema são

primos entre si dois a dois, de modo que pelo TCR este sistema tem uma única solução

módulo m =m1 . m2 . m3 . m4 = 5 . 3 . 7 . 4 = 420. Temos então:

1

1

m 420M 84

m 5,

2

2

m 420M 140

m 3,

3

3

m 420M 60

m 7,

4

4

m 420M 105

m 4

Os inversos xk dos Mk são dados por:

84 x1 1 (mod 5)

140 x2 1 (mod 3)

60 x3 1 (mod 7)

105 x4 1 (mod 4)

Aplicando o método de Euler nas equações acima obtemos as soluções respectivas:

x1 = 4, x2 = 2, x3 = 2, e x4 = 1.

Portanto, temos :

x a1.M1.x1 + a2.M2.x2 + ... + ak.Mk.xk (mod m)

x 8 . 84 . 4 + 5 . 140 . 2 + 11 . 60 . 2 + 2 . 105 . 1 (mod 420)

x 5618 (mod 420) 158 (mod.420),

segue-se que x = 158 é a menor solução positiva módulo 420, do sistema de

congruências lineares dado. Qualquer outra solução é da forma:

Exemplo 10.8:

5.x 11 mod 17  

3.x 19 mod 32

11.x   6 mod 37

x 158 (mod 420) x = 158 + 420.k, k .

CAPÍTULO 10

FERMAT, WILSON E EULER

164

Resolução: Como o MDC(17, 32) = MDC(17, 37) = MDC(32, 37) = 1, o sistema possui

solução.

De 5.x 11 (mod 17), obtemos x 9 (mod 17).

De 3.x 19 (mod 32), obtemos x 17 (mod 32)

De 11.x 6 (mod 37), obtemos x 14 (mod 37)

assim temos o seguinte sistema:

9(mod 17)

17(mod 32)

14(mod 37

x

x

x

Usando o TCR:

a1 = 9, a2 = 17, a3 = 14.

m1 = 17, m2 = 32, m3 = 37.

m = m1 . m2 . m3 = 17 . 32 . 37 = 20128.

1

1

m 20128M 1184

m 17

, 2

2

m 20128M 629

m 32,

3

3

m 20128M 544

m 37

Então 1184.b1 1 (mod 17), 629.b2 1 (mod 32) e 544.b3 1 (mod 37).

De onde concluímos: b1 = 14 , b2 = 29 e b3 = 10.

A solução geral será:

EXERCÍCIOS

1. Resolver os seguintes sistemas de congruências lineares utilizando o TCR:

a)

3 (mod5)

5 (mod 7)

7 (mod11)

x

x

x

b)

1 (mod5)

5 (mod 7)

7 (mod11)

x

x

x

c)

5 (mod 6)

4 (mod11)

3 (mod17)

x

x

x

d)

5 (mod11)

14 (mod 29)

15 (mod 31)

x

x

x

e)

7 (mod 9)

10 (mod 4)

1 (mod 7)

x

x

x

f)

28 (mod 29)

30 (mod 31)

10 (mod11)

x

x

x

g)

(mod 3)

(mod 5)

(mod8)

x a

x b

x c

h)

2 1 (mod 5)

4 1 (mod 7)

5 9 (mod11)

x

x

x

x 9 . 14 . 1184 + 17 . 29 . 629 + 14 . 10 . 544 (mod 20128)

x 535441 12113 (mod 20128)

CAPÍTULO 10

FERMAT, WILSON E EULER

165

10.10. POTENCIAÇÃO: UMA APLICAÇÃO DO TEOREMA DE EULER

Seja m um inteiro positivo tal que mdc(a, m) = 1 . Então:

Aplicamos este resultado ao cálculo de potência módulo m. De uma maneira geral o problema

é o seguinte. São dados a, k e m três inteiros positivos, dos quais sabemos que m é co-primo

com a. Digamos que k é muito grande (o caso difícil) e queremos achar a forma reduzida de ak

(mod m).

Podemos simplificar as contas usando o teorema de Euler. Estamos supondo que k é grande,

na prática precisamos saber apenas que k ( )m . Dividindo k por ( )m , obtemos

k = ( )m .q + r, onde o resto r satisfaz 0 r ( )m – 1. Temos, então, que:

Mas pelo teorema de Euler, a(m)

1 (mod m) . Obtemos, portanto, da equação acima que ak

ar (mod p).

Um exemplo numérico para convencê-lo das vantagens deste resultado tão simples.

Queremos calcular 25432675

(mod 13). Da maneira como vínhamos procedendo teríamos que

efetuar uma quantidade enorme de potenciações módulo 13. Usando a idéia acima, obtemos o

resto da divisão de k = 5432675 por (13), que é r = 11, e assim:

Logo, 25432675

7 (mod 13).

10.11 – POTENCIAÇÃO: UMA APLICAÇÃO DO TEOREMA CHINÊS DO RESTO (TCR)

Podemos aplicar o TCR para simplificar o cálculo de potências módulo n em alguns casos

especiais. Suponhamos que n = p1 . p2 . ... . pk, onde p1 < p2 < ... < pk são números primos.

Assim, estamos supondo que, na fatoração de n, cada fator primo aparece com multiplicidade

1. Neste caso fica muito fácil calcular a forma reduzida de am

(mod n).

Em primeiro lugar, usamos o PTF para achar a forma reduzida de am

módulo cada um dos

primos p1, p2, ..., pk separadamente. Digamos que:

am

r1 (mod p1) e 0 r1 < p1

am

r2 (mod p2) e 0 r1 < p2

am

rk (mod pk) e 0 rk < pk

25432675

(2(13)

)q . 2

11 (mod 13) 2

11 (mod 13) 7 (mod 13)

ak

(m).q+ra (mod m) ((m)a )

q . a

r (mod m)

a(m)

1 (mod m)

CAPÍTULO 10

FERMAT, WILSON E EULER

166

Para achar a forma reduzida de am

(mod n), basta resolver o sistema:

Observe que este sistema sempre tem solução, já que os módulos são primos distintos e,

portanto, o MDC entre dois quaisquer entre eles é sempre igual a 1. Além disto, o teorema nos

garante que o sistema tem uma única solução r (mod n), onde n = p1 . p2 . ... . pk.

Logo am

r (mod n), e obtivemos o que queríamos.

Vejamos um exemplo numérico. Digamos que queremos calcular a forma reduzida de

Fatorando 1155 vemos que é 1155 = 3 . 5 . 7 . 11, todos primos entre si com multiplicidade 1.

Aplicando o PTF a cada um destes primos, temos que:

Assim:

Precisamos, portanto, resolver o sistema:

Solução:

1

1

m 1155M 385

m 3;

2

2

m 1155M 231

m 5;

3

3

m 1155M 165

m 7

; 4

4

m 1155M 105

m 11

m = m1 . m2 . m3 . m4 = 3 . 5 . 7 . 11 = 1155

x 1 (mod 3)

x 4 (mod 5)

x 2 (mod 7)

x 5 (mod 11)

6754 = (3).q1 + r1 6754 = 2 . 3377 + 0, logo 20 1 (mod 3).

6754 = (5).q2 + r2 6754 = 4 . 1688 + 2, logo 22 4 (mod 5).

6754 = (7).q3 + r3 6754 = 6 . 1125 + 4, logo 24 2 (mod 7).

6754 = (11).q4 + r4 6754 = 10 . 675 + 4, logo 24 5 (mod 11).

26754

1r2 (mod 3)

26754

2r2 (mod 5)

26754

3r2 (mod 7)

26754

4r2 (mod 11)

26754

(mod 1155)

x r1 (mod p1)

x r2 (mod p2)

...

x rk (mod pk)

CAPÍTULO 10

FERMAT, WILSON E EULER

167

M1.x1 1 (mod m1) 385.x1 1 (mod 3) x1 385(3)–1

(mod 3) 1 (mod 3)

M2.x2 1 (mod m2) 231.x2 1 (mod 5) x2 231(5)–1

(mod 5) 1 (mod 5)

M3.x3 1 (mod m3) 165.x3 1 (mod 7) x3 165(7)–1

(mod 7) 2 (mod 7)

M4.x4 1 (mod m4) 105.x4 1 (mod 11) x4 105(11)–1

(mod 11) 2 (mod 11)

Assim, temos:

EXERCÍCIOS

1) Construa as tabelas de operações em 8 ,

9 ,

10 , 13 e

17 .

2) Encontre os dois números primos, cujos

produtos, geraram os números abaixo,

utilizando o método da Fatoração de Fermat:

a) 437623

b) 919199

c) 9797

d) 4061

e) 19109

3) Calcule a potências utilizando o método da

exponenciação rápida.

a) 33268

(mod 335)

b) 1774096

(mod 277)

c) 818192

(mod 92)

d) 482048

(mod 20)

4) Calcule as potências utilizando o Pequeno

Teorema de Fermat

a) 52349899

(mod 17)

b) 4511223311

(mod 19)

c) 1001002003007

(mod 281)

5) Calcule as potências utilizando o Pequeno

Teorema de Fermat e o Algoritmo Chinês do

Resto.

a) 4753

(mod 437623)

b) 51773

(mod 919199)

c) 8397

(mod 2926)

d) 313961

(mod 12369)

x a1.M1.x1 + a2.M2.x2 + a3.M3.x3 + a4.M4.x4 (mod m)

x 1 . 385 . 1 + 4 . 231 . 1 + 2. 165 . 2 + 5 . 105 . 2 (mod 1155)

x 3019 (mod 1155) 709 (mod 1155)

CAPÍTULO 10

FERMAT, WILSON E EULER

168

LEITURA COMPLEMENTAR: PSEUDOPRIMOS

De acordo com o teorema de Fermat, se p é primo e a é um inteiro qualquer não divisível por

p, então ap a (mod p). Imagine então a seguinte situação: temos um número ímpar n, e

desejamos saber se é, ou não, composto.

Digamos que descobrimos um inteiro b tal que bn b (mod n). Pergunta-se: n pode ser

primo? A resposta é não, porque isto viola o teorema de Fermat. Logo, isto nos dá uma

maneira indireta de verificar se um número é composto.

Observe que, na prática, só precisamos considerar os inteiros b no intervalo 1 < b < n – 1.

Por quê? Em primeiro lugar, como estamos trabalhando módulo n, qualquer inteiro é

congruente a um inteiro no intervalo de 0 a n – 1. Além disso, a equação bn b (mod n) é

sempre satisfeita quando b é 0, 1 ou n – 1.

Este teste produz uma situação surpreendente. Com ele podemos chegar à conclusão de que

um número é composto mesmo que não nos seja possível determinar seus fatores. Antes de

fazer um exemplo, é conveniente formularmos um teste de uma maneira mais fácil de utilizar

na prática. Para isto usaremos a segunda versão do teorema de Fermat.

Eis o teste: se n > 0 e 1 < b < n – 1 são números inteiros, e bn–1

1 (mod n), então n é um

número composto. O número b é conhecido como uma “testemunha de fato” de n ser

composto.

Será que podemos inverter o teorema de Fermat para verificar se um número é primo? Na

verdade, estamos perguntando se um número ímpar n que satisfaz

bn–1

1 (mod n), para algum 1 < b < n – 1, é primo? Leibniz, famoso pela invenção do cálculo

(quase ao mesmo tempo que Newton), achava que sim, e usou isto como um critério de

primalidade. Na verdade, ele tomava apenas b = 2, que é o caso mais simples de calcular.

Infelizmente isto não é verdade. Por exemplo, 2340

1 (mod 341). Logo, segundo Leibniz,

341 seria um número primo. Mas, 341 = 11 . 31, é composto. Estes “falsos primos” são

conhecidos como pseudoprimos. Isto é, um inteiro positivo n, ímpar e composto, é um

pseudoprimo para a base b (onde 1 < b < n – 1) se bn – 1

1 (mod n). Assim, 341 é um

pseudoprimo para a base 2.

Apesar de às vezes dar errado, o teste de Leibniz é muito útil. Pelo menos para números

pequenos, ele acerta mais do que erra. Por exemplo, entre 1 e 109 existem 50.847.534 primos,

mas apenas 5597 pseudoprimos para a base 2. Logo um número menor que um bilhão que

passa no teste de Leibniz apenas com a base 2 tem uma alta probabilidade de ser primo.

Além disto, usamos apenas a base 2 até agora. Por que não testar para mais de uma base?

Fazendo isto o teste fica ainda mais eficiente. Por exemplo, 3340

56 (mod 341). Logo 3 é

uma testemunha de fato de que 341 é composto. Na verdade, há apenas 1272 pseudoprimos

simultâneos para as bases 2 e 3 no intervalo entre 1 e 109.

Como estamos limitando nossas bases ao intervalo entre 1 e n – 1, podemos considerar a

possibilidade de testar se n é pseudoprimo para todas estas bases. Se n é grande, isto vai ser

impossível, logo o interesse prático é muito limitado. Por outro lado, este problema leva a

CAPÍTULO 10

FERMAT, WILSON E EULER

169

algumas questões surpreendentes, e de real interesse prático, como veremos em seguida com

os números de Carmichael.

Números de Carmichael

Primeiramente, não existem números que sejam pseudoprimos para todas as bases. Isto é fácil

de constatar. Se n é composto, então tem um fator b, ou seja, MDC(b ,n) 1. Como também o

MDC(bn – 1

, n) 1, temos pelo teorema de inversão que b não pode ser inversível módulo n.

Em particular, bn – 1

1 (mod n). Logo n não é pseudoprimo para a base b. Entretanto, pode

ocorrer que um número composto n seja pseudoprimo para todas as bases que são primas a n.

É este tipo de número que desejamos discutir.

É preferível reformular o problema da seguinte maneira: seja n um número inteiro positivo,

como caracterizar os números compostos n que satisfazem à equação bn b (mod n)?

A vantagem desta formulação é que podemos dispensar a condição MDC(b, n) = 1, o que é

muito conveniente. O primeiro matemático a dar exemplos destes números foi R. D.

Carmichael, em um artigo publicado em 1912. Por isso são chamados números de

Carmichael.

Portanto, um número composto ímpar n > 0 é um número de Carmichael se bn b (mod n)

para todo 1 < b < n – 1. Observe que, para um número ser de Carmichael é preciso que seja

composto. Um número primo p também satisfaz à equação bp b (mod p), mas não é um

número de Carmichael.

Como o próprio Carmichael determinou, o menor número de Carmichael é 561. Em princípio

podemos verificar isto usando a definição. Entretanto, mesmo para um número relativamente

pequeno como este, isto é difícil de fazer usando apenas lápis e papel.

Afinal, para mostrar que 561 é um número de Carmichael pela definição, precisamos mostrar

que b561

b (mod 561) para b = 2, 3, 4, 5, ..., 558, 559; o que dá um total de 558 bases a

serem testadas, algumas não tão pequenas. Não há duvida de que esta é uma tarefa para um

computador. Entretanto, mesmo um computador razoável pode ter dificuldade em usar este

método para verificar que, por exemplo:

349407515342287435050603204719587201

é um número de Carmichael.

Felizmente há uma maneira mais simples de verificar que um dado número composto é de

Carmichael. Consideremos 561 mais uma vez. Podemos facilmente fatorá-lo:

561 = 3 . 11 . 17

Seja agora b um número inteiro entre 2 e 559. Queremos mostrar que b561

b (mod 561).

Nossa estratégia será a seguinte: mostraremos que b561

– b é divisível por 3, 11 e 17 para

todos os valores de b entre 2 e 559. Como são primos distintos, segue que o produto destes

primos divide b561

– b. Mas este produto é 561, e dizer que 561 divide b561

– b é equivalente a

dizer que b561

b (mod 561).

CAPÍTULO 10

FERMAT, WILSON E EULER

170

Só nos resta mostrar que b561

– b é divisível por cada um dos fatores de 561, separadamente.

Como estes fatores são primos, o teorema de Fermat vem em nossa ajuda. Vamos efetuar as

contas para o primo 17 e deixar 3 e 11 como exercício. Queremos verificar que b561

– b é

divisível por 17, isto é, b561

b (mod 17).

Há dois casos a considerar. Se 17 divide b então ambos b e b561

são membros de b

congruentes a zero modulo 17, logo a congruência é imediatamente verificada. Mas, digamos

que 17 não divide b. Neste caso, o teorema de Fermat diz que b16

1 (mod 17). Para aplicar

isto, precisamos dividir 561 por (17 – 1), mas isto resulta em 561 = 35 . 16 + 1. Assim, b561

(b16

)35

. b (mod 17) (1)35

. b (mod 17) b (mod 17).

Alguns números de Carmichael (existem infinitos):

Provaremos que, se t é tal que (6.t + 1), (12.t + 1) e (18.t + 1) são todos primos, então o seu

produto é um número de Carmichael. Não se sabe ainda (2005), se existe uma infinidade de

tais números t.

Demonstração: temos que n = (6t + 1).(12t + 1).(18t + 1) é composto.

E que (n – 1) = 1296.t3 + 396.t

2 + 36.t 0 (mod 36)

Seja o MDC(a, n) = 1. Então, a é relativamente primo com (6.t + 1), (12.t + 1) e (18.t + 1).

Pelo PTF:

Logo, a36.t

1 (mod n). Como 36.t divide (n – 1), temos que an–1

1 (mod n).

Como n é composto, n é um número de Carmichael.

a6.t

1 (mod 6.t + 1) a36.t

1 (mod 6.t + 1)

a12.t

1 (mod 12.t + 1) a36.t

1 (mod 12.t + 1)

a18.t

1 (mod 18.t + 1) a36.t

1 (mod 18.t + 1)

561, 1105, 1729, 2465, 2821, 6601, 8911, 10585, 15841, 29341, 41041, 46657, 52633,

62745, 63973, 75361, 101101, 115921, 126217, 162401, 172081, 188461, 252601,

278545, 294409, 314821, 334153, 340561, 399001, 410041, 449065, 488881, 512461,

530881, 552721, 656601, 658801, 670033, 748657, 825625, 838201, 852841, 997633.

171

Capítulo 11

CIFRA DE CÉSAR

INTRODUÇÃO À CRIPTOGRAFIA

A palavra Criptografia é composta por dois termos gregos kryptos (kryptos secreto,

escondido, oculto) e grapho (grapho - escrita grafia).

A criptografia é uma arte ou ciência de escrever ocultamente talvez tão antiga quanto a

própria escrita, hoje em dia é um dos métodos mais eficientes de se transferir informação, sem

que haja a possibilidade de interferência por parte de terceiros. É o estudo de técnicas

matemáticas, relacionadas com os aspectos de segurança e confidencialidade de informação, a

integridade de dados, a autenticação de entidades e a autenticidade de origem de dados, ou

seja, consiste na conversão de dados num código secreto como medida de segurança para que

possam existir comunicações seguras.

A criptografia lida de um modo muito estreito com termos como - encriptação e

desencriptação. A encriptação é a conversão de dados para uma forma que não será

compreendida facilmente por pessoas autorizadas com o objectivo de assegurar a privacidade

mantendo a informação escondida e ilegível mesmo para quem vê os dados encriptados. A

desencriptação é o processo de converter dados encriptados de volta á sua forma original, para

que a mensagem possa ser compreendida e para isso acontecer requer alguma informação

secreta, usualmente denominada chave de desencriptação. A chave de desencriptação é o

algoritmo que desfaz o trabalho do algoritmo de encriptação. Um algoritmo é um programa de

computador que pode ser visto como um algoritmo elaborado. É baseada em chaves, uma

informação pode ser codificada através de algum algoritmo de criptografia, de modo que,

tendo conhecimento do algoritmo e da chave utilizados, é possível recuperar a informação

original fazendo o percurso contrário da encriptação, a desencriptação.

CAPÍTULO 11

CIFRA DE CÉSAR

172

Embora os códigos secretos remontem aos primórdios da comunicação escrita, tem havido um

aumento recente de interesse no assunto devido à necessidade de manter a privacidade da

informação transmitida ao longo de linhas públicas de comunicação. Na linguagem da

criptografia, os códigos são denominados cifras, as mensagens não codificadas, são textos

comuns e as mensagens codificadas são textos cifrados ou criptogramas. O processo de

converter um texto comum em cifrado é chamado cifrar ou criptografar e o processo

inverso de converter um texto cifrado em comum é chamado decifrar.

As cifras mais simples, denominadas cifras de substituição (ou Código de César), são as

que substituem cada letra do alfabeto por uma outra letra.

Por exemplo, na cifra de substituição

Comum A B C D E F G H I J K L M N O P Q R S T U V W X Y Z

Cifra D E F G H I J K L M N O P Q R S T U V W X Y Z A B C

A letra de texto comum A é substituída por D, a letra de texto comum B por E e assim por

diante.

Com esta cifra, a mensagem de texto comum

fica

A Cifra de César - Um dos sistemas criptográficos mais antigos e simples é a chamada

“cifra de César”, em homenagem ao famoso imperador romano . Júlio César usou sua famosa

cifra de substituição para cifrar mensagens governamentais. Atualmente denomina-se

qualquer cifra baseada na substituição cíclica do alfabeto de código de César. Com o uso de

dois discos concêntricos contendo todas as letras do alfabeto, a substituição se tornava

extremamente simples.

A cifra de César baseia-se na seguinte propriedade:

“Seja m>1 um inteiro. Para cada a mZ fixado, temos que f:

m mZ Z definida por f(x) =

x a (mod m) é bijetiva”.

URPD QDR IRL FRQVWUXLGD HP XP GLD

ROMA NÃO FOI CONSTRUÍDA EM UM DIA

CAPÍTULO 11

CIFRA DE CÉSAR

173

Demonstração:

(i) f é injetiva.

De fato:

f(x) = f(y)

x + a = y + a (mod m)

x + a - a = y + a - a (mod m)

x = y (mod m)

x = y em m , donde f(x) = f(y) acarreta x = y

(ii) f é sobrejetiva

Seja x um elemento qualquer de m .

Então x - a está em m e f(x - a) = x, uma vez que em

m x – a = x – a + km, para

todo inteiro k. Logo todo x em m é igual a f(x - a), donde f é sobrejetiva.

Como f é injetiva e sobrejetiva, então f é bijetiva.

Imaginemos, por questão de simplicidade, as 26 letras usuais e o espaço (entre duas palavras)

associados aos elementos de 27Z conforme a Tabela 1 abaixo ( adotaremos o símbolo [ ] para

indicar um espaço entre as palavras):

A B C D E F G H I J K L M

1 2 3 4 5 6 7 8 9 10 11 12 13

N O P Q R S T U V W X Y Z

14 15 16 17 18 19 20 21 22 23 24 25 26

[ ]

0

Tabela 1

Fixado um elemento a mZ ( a é a chave do código – de transmissão e de recepção), a

aplicação f: x x a (mod 27) permuta os elementos de 27Z e, conseqüentemente, os

elementos do conjunto formado pelo símbolo do espaço e as 26 letras. Dessa forma cada

mensagem se transforma em código; o fato de f ser bijetiva garante que mensagens diferentes

são codificadas de maneira diferente e, ainda, a possibilidade da decodificação.

Exemplo 11.1: Vejamos como codificar a frase “EU VOU”, usando como chave a = 14, ou

seja com y = x + 15 (mod 27)

E 5 5 + 14 19(mod 27) S

U 21 21 + 14 8 (mod 27) H

[] 0 0 + 14 14 (mod 27) N

V 22 22 + 14 9 (mod 27) I

O 15 15 + 14 2 (mod 27) B

U 21 21 + 14 8 (mod 27) H

Portanto o código para a frase dada é “ SHNIBH”.

Para decodificar, considerando que o simétrico aditivo de 14 módulo 27 é 13

(pois, 14 + 13 0 (mod 27)), mantendo, portanto, a chave a = 14 ), procede-se assim:

CAPÍTULO 11

CIFRA DE CÉSAR

174

S 19 19 + 13 5 (mod 27) E

H 8 8 + 13 21 (mod 27) U

N 14 14 + 13 0 (mod 27) []

I 9 9+13 22 (mod 27) V

B 2 2+13 15 (mod 27) O

H 8 8+13 21 (mod 27) U

11.1. Funções Polinomiais de Codificação

No exemplo acima usamos a função polinomial f(x) = x + 15 (mod 27) para codificar a

mensagem e usamos a sua inversa f -1

(x) = x + 12 (mod 27) para decodificação.

A pergunta que podemos fazer é podemos usar qualquer função polinomial módulo m para

codificar uma mensagem? A resposta é não!

Como vimos, precisamos da inversa para decodificar a mensagem. Assim se escolhermos uma

função polinomial que não seja bijetiva no domínio trabalhado teremos problemas em

decodificar em situações normais. Além disso, nos casos em que tivermos multiplicações e

divisões da variável x, dependendo de Zm podemos não ter os inversos de x módulo m nos

casos em que m não é primo impossibilitando a decodificação. Vejamos alguns exemplos:

Exp1. Use a tabela abaixo e a função 2( ) 23(mod29)f x x para codificar a palavra DJ

A B C D E F G H I J K L M

1 2 3 4 5 6 7 8 9 10 11 12 13

N O P Q R S T U V W X Y Z

14 15 16 17 18 19 20 21 22 23 24 25 26

Á É [ ]

27 28 0

2(4) 4 23 10(mod29)D f J

2(10) 10 23 7(mod29)J f G

Queremos agora decodificar a mensagem JG. A inversa da função y = x2 + 23 é obtida pelo

método prático de trocar x por y e colocar y em função de x, ou seja: 2

2

2

2

23

23

23 ou

6(mod 29)

y x

x y

y x

y x

Ou seja, para “voltarmos” estamos interessado em saber que número y elevado ao quadrado é

igual a um número x + 6 (mod 29) ( isto é, [f(x)]2 = x + 6 ). Assim,

CAPÍTULO 11

CIFRA DE CÉSAR

175

2 10 6 16(mod29)J y

Que número elevado ao quadrado é igual a 16 módulo 29? Resposta: 4. Logo J é D.

2 7 6 13(mod7)G y

Que número elevado ao quadrado é igual a 13 módulo 29? Temos um problema! Existem dois

valores que elevado ao quadrado são iguais a 13 módulo 29: 10 e 19, letras J e S

respectivamente, pois

10.10 13(mod29) e 19.19 13(mod29)

impedindo-nos, em situações mais complexas, de decodificar a mensagem. Isso se deve ao

fato de f(x) = x2

+ 23 não ser bijetiva.

Exercícios

Use a Tabela 2 abaixo para os seus cálculos

Z31

A B C D E F G H I J K L M

1 2 3 4 5 6 7 8 9 10 11 12 13

N O P Q R S T U V W X Y Z

14 15 16 17 18 19 20 21 22 23 24 25 26

? Á Ã É [ ]

27 28 29 30 0

1. Utilize as funções abaixo para codificar e sua inversa* para decodificar as mensagens

dadas.

a) 2 (mod31)y x ; 7(mod31)y x ; O MAIS QUERIDO

b) 3 (mod31)y x ; 2 30(mod31)y x ; NAVEGAR É PRECISO?

c) 4 (mod31)y x ; 3 2(mod31)y x ; SE VOU NÃO FICO

d) 5 (mod31)y x ;1

(mod31)29

xy

x; ELVIS NÃO MORREU

* Use o método da troca de variáveis para encontrar a função inversa

CAPÍTULO 11

CIFRA DE CÉSAR

176

LEITURA COMPLEMENTAR:

ANÁLISE DE FREQUÊNCIA

Em qualquer língua, alguns sons são utilizados com mais frequência do que outros. Isto

significa que, na linguagem escrita, algumas letras também são mais utilizadas que outras.

Determinar a frequência com que ocorrem determinadas letras em determinada língua, ou

seja, fazer uma análise da frequência de ocorrência de letras.

Apesar de não se saber quem foi o primeiro a perceber que a variação na frequência de letras

poderia ser explorada para se quebrar cifras, a descrição mais antiga de que se tem

conhecimento e que descreve esta técnica data do século 9 e é devida ao cientista Abu Yusuf

Ya 'qub ibn Is-haq ibn as-Sabbah ibn 'omran ibn Ismail al-Kindi.

Conhecido como o filósofo dos árabes, al-Kindi foi o autor de 290 livros sobre medicina,

astronomia, matemática, linguística e música. No entanto, seu maior tratado, o qual foi apenas

redescoberto em 1987 no Arquivo Sulaimaniyyah Ottoman em Istambul, na Turquia, é

intitulado "Um Manuscrito sobre Decifração de Mensagens Criptográficas.

A cifra de substituição monoalfabética parecia inquebrável devido ao número muito grande de

chaves possíveis. Entretanto, havia uma fraqueza que minava sua segurança.

A quebra da cifra de substituição marca o nascimento da criptanálise. Tal fato ocorreu durante

os anos dourados da civilização islâmica, quando muitos manuscritos estrangeiros foram

levados para Bagdá para integrarem as grandes bibliotecas árabes. Alguns destes manuscritos

estavam encriptados, o que motivou os arrombadores de códigos a quebrarem as cifras para

revelar os segredos que continham.

As letras "A" e "I" são as mais comuns em Árabe. No Inglês, as letras mais comuns são o "E",

o "T" e o "A". Já no Português, as mais frequentes são "A", "E", "O" e "S".

Se uma mensagem é cifrada de modo que cada letra seja substituída por uma outra, então a

nova letra assumirá todas os atributos da letra original, inclusive com que frequência é

utilizada.

Desta forma, se a longa mensagem estiver em Português e se a letra mais comum na

mensagem cifrada for G, então G provavelmente representa A. Se a segunda letra mais

frequente na mensagem cifrada for W, então a probabilidade de que esteja substituindo o E é

bastante grande, e assim por diante.

Devemos considerar qual foi a língua utilizada para redigir a mensagem. Esta é uma questão

essencial porque define o padrão da frequência da ocorrência de letras que deve ser usada para

fazer a comparação. É óbvio que SEMPRE existem outras pistas que podem ajudar: o

remetente da mensagem, o destinatário, o possível assunto, etc.

Caso estejamos considerando o Português, é claro que se vai fazer uma análise de frequência

usando esta língua como base. Quanto mais longo for o texto, maior a probabilidade dos

valores encontrados estarem mais próximos dos valores padrão. Além disso, não se deve

esquecer que os valores padrão representam a MÉDIA da frequência de ocorrência. Se, por

exemplo, o padrão para a letra A é de 14.63%, esta frequência pode variar, digamos, de 13% a

17%. Neste caso estamos contando com um desvio de cerca de 2%.

CAPÍTULO 11

CIFRA DE CÉSAR

177

Abaixo estão as tabelas de das frequências relativas das letras nas línguas Portuguesa e

Inglesa.

Frequência relativa das letras no Português

Frequência relativa das letras no Inglês

CAPÍTULO 11

CIFRA DE CÉSAR

178

Exercícios

1. Suponha que você interceptou a mensagem:

JV F UZEYVZIF JFSIRJJV V JV VCR DV RDRJJV

Segundo suas fontes ela foi codificada utilizando a cifra de César de acordo com a tabela

seguinte

A B C D E F G H I J K L M

0 1 2 3 4 5 6 7 8 9 10 11 12

N O P Q R S T U V W X Y Z

13 14 15 16 17 18 19 20 21 22 23 24 25

É claro que você não sabe a chave usada. Como bom estudante de Estatística, você fez

uma análise de frequência ( isso é que é interesse nas coisas alheias!). Suponha que você

já havia interceptado um texto com mais de 1000 letras da mesma origem. De acordo

com a análise de frequência, a letra A é mais comum na língua portuguesa. Você

percebeu que a letra mais comum no texto cifrado foi a R. Com mais uma suposição de

que a chave ainda não foi trocada, decodifique a mensagem. Desconsidere os espaços.

2. Para as questões de 1 a 6, utilize a seguinte tabela de conversão:

A B C D E F G H I J K L M

1 2 3 4 5 6 7 8 9 10 11 12 13

N O P Q R S T U V W X Y Z

14 15 16 17 18 19 20 21 22 23 24 25 26

*O espaço em branco [ ] é representado pelo valor 0.

1. Utilizando a função f(x) = x + 13 (mod 27), codifique a palavra SOFTWARE.

2. Utilizando a função f(x) = 4.x + 11 (mod 27), codifique a palavra UNIVERSO. Encontre a

função inversa de f(x) e verifique se sua resposta está correta.

3. Decodifique a palavra GESIOF sabendo que a função f(x) = x + 14 (mod 27) foi utilizada

para cifragem.

4. Decodifique a palavra BVJZOL sabendo que a função f(x) = 5.x + 17 (mod 27) foi

utilizada para cifragem.

5. Dada a mensagem em texto simples FELIZ, e o seu respectivo texto cifrado YXDAR,

encontre a função afim que foi utilizada na cifragem. Determine a função inversa

6. Dada a mensagem em texto simples TEMPO, e o seu respectivo texto cifrado ORTNG,

encontre a função afim que foi utilizada na cifragem. Determine a função inversa.

179

Capítulo 12

CIFRA DE VIGENÈRE

A cifra de Vigenère tem este nome em homenagem a Blaise de Vigenère, embora realmente

tenha sido inventada antes por Giovan Batista Belaso. O que Vigenère fez foi modificar a

cifra para torná-la mais robusta.

A cifra de Vigenère é um método de encriptação que usa um série de diferentes cifras de

César baseadas em letras de uma senha. Trata-se de uma versão simplificada de uma mais

geral cifra de substituição polialfabética, inventada por Leone Battista Alberti a cerca de

1465.

A cifra de Vigenère foi uma campeã em segurança. Foram precisos 300 anos para que, quase

que simultaneamente, ao redor de 1860, Babbage (na Inglaterra) e Kasiski (na Alemanha)

quebrassem a cifra.

A invenção da cifra de Vigenère é erradamente atribuída a Blaise de Vigenère; encontra-se

originalmente descrita por Giovan Batista Belaso no seu livro datado de 1553 com o título La

cifra del. Sig. Giovan Batista Belaso.

Esta cifra é muito conhecida porque é fácil de perceber e de pôr em prática.

Conseqüentemente, muitos programadores implementaram esquemas de encriptação nas suas

aplicações que são no essencial cifras de Vigenère.

Descrição

Em uma cifra de César, cada letra do alfabeto é deslocada da sua posição um número fixo de

lugares; por exemplo, se tiver uma deslocação de 3, a letra A se torna D, B se torna E, etc. A

cifra de Vigenère consiste na sequência de várias cifras de César com diferentes valores de

deslocamento.

A cifra de Vigenère pode ser vista algebricamente. A encriptação pode ser escrita como

Ci (Pi + ai) (mod m)

e a decriptação como

Pi (Ci – ai) (mod m)

onde Pi corresponde aos valores das letras a serem cifradas, ai aos valores das letras da chave

e Ci aos valores das letras cifradas.

CAPÍTULO 12

CIFRA DE VIGENÈRE

180

Exemplo 12.1: Vamos supor agora, que a palavra-chave escolhida tenha sido "GREGO" e a

mensagem a ser codificada seja “PERXES PREPARA UM ATAQUE”. Para isso utilizaremos

a seguinte tabela de valores módulo 31:

A B C D E F G H I J K L M N O P

1 2 3 4 5 6 7 8 9 10 11 12 13 14 15 16

Q R S T U V W X Y Z ? Á Ã É [ ]

17 18 19 20 21 22 23 24 25 26 27 28 29 30 0

Para codificar a mensagem, podemos escrever as letras da palavra-chave quantas vezes for

preciso acima da frase:

G R E G O G R E G O G R E G O G R E G O G R E G

P E R X E S P R E P A R A U M A T A Q U E

isto equivale a fazer os seguintes cálculos

7 18 5 7 15 7 18 5 7 15 7 18 5 7 15 7 18 5 7 15 7 18 5 7

16 5 18 24 5 19 0 16 18 5 16 1 18 1 0 21 13 0 1 20 1 17 21 5

Somando termo a termo mod 31 (Ci (Pi + ai)), obtemos:

23 23 23 0 20 26 18 21 25 20 23 19 23 8 15 28 0 5 8 4 8 4 26 12

W W W T Z R U Y T W S W H O Á E H D H D Z L

Para decifrar a mensagem WWW TZRUYTWSWHOÁ EHDHDZL

basta fazer a operação inversa (Pi (Ci – ai)).

CAPÍTULO 12

CIFRA DE VIGENÈRE

181

Exercícios:

Para as questões abaixo, utilize a seguinte tabela de conversão:

A B C D E F G H I J K L M N O P

1 2 3 4 5 6 7 8 9 10 11 12 13 14 15 16

Q R S T U V W X Y Z ? Á Ã É [ ]

17 18 19 20 21 22 23 24 25 26 27 28 29 30 0

Tabela de conversão de caracteres para a Cifra de Vigenère (mod 31).

1) Decifre a mensagem abaixo, usando a palavra-chave CERA:

2) Descubra a mensagem a seguir, capturada na transmissão do inimigo.

A palavra-chave utilizada foi TEBAS.

3) Codifique a mensagem “RIO DE JANEIRO” utilizando a chave TRIUNFO.

4) Codifique a mensagem “FRONTEIRAS DO BRASIL” utilizando a chave TIGRE.

5) Decodifique a mensagem “DYDSSX YNYFS” sabendo que a chave BYTE foi utilizada.

6) Decodifique a mensagem “XIDHY OLOWFBRL” sabendo que a chave MISTURA foi

utilizada.

7) Dada a palavra em texto simples “ILUSTRE”, e o seu respectivo texto cifrado “MCI

FYS”, encontre a chave que foi utilizada na cifragem.

8) Dada a mensagem em texto simples “CRISE DOS INFERNOS”, e o seu respectivo texto

cifrado “EEVHYHVGSBW VYZEGS”, encontre a chave que foi utilizada na cifragem.

9) Desafio: dada a mensagem

OSFYSRWXMFTANASWVNSSDMJMVAMVSISINQBLUDB”,decodifique-a e

encontre a chave que foi utilizada na cifragem.

QIOETGZTQHVIZSLARIISKE

TXBQMXWFRSIIMOETVOEKMENAVKYHAVT

182

Capítulo 13

CIFRA DE HILL

Uma das desvantagens de cifras de substituição é que elas preservam as frequências de letras

individuais, tornando relativamente fácil quebrar o código por métodos estatísticos. Uma

maneira de superar este problema é dividir o texto em grupos de letras e criptografar o texto

comum por grupo, em vez de uma letra de cada vez. Um sistema poligráfico é um sistema de

criptografia no qual o texto comum é dividido em conjuntos de n letras, cada um dos quais é

substituído por um conjunto de n letras cifradas. Veremos uma classe de sistemas poligráficos

chamados cifras de Hill (Em 1929 Lester S. Hill publica seu livro Cryptography in an

Algebraic Alphabet, no qual um bloco de texto claro é cifrado através de uma operação com

matrizes).

Daqui em diante, nós vamos supor que cada letra de texto comum e de texto cifrado, tem um

valor numérico que especifica sua posição no alfabeto padrão (Tabela 1). Utilizaremos o

símbolo [ ] para indicar um espaço entre as letras ou palavras.

Tabela 1

A B C D E F G H I J K L M

1 2 3 4 5 6 7 8 9 10 11 12 13

N O P Q R S T U V W X Y Z

14 15 16 17 18 19 20 21 22 23 24 25 0

Iniciemos com o caso mais simples de cifras de Hill que utiliza matrizes 2x2.

Usando a tabela acima em 26 transformaremos pares sucessivos de texto comum em texto

cifrado pelo seguinte procedimento:

Passo 1. Escolha uma matriz A, 2 x 2, com entradas inteiras para efetuar a codificação. A

matriz deve ser inversível módulo 26 .

Passo 2. Agrupe letras sucessivas de texto comum em pares, adicionando uma letra fictícia

para completar o último par se o texto comum tem um número ímpar de letras; substitua cada

letra de texto comum por seu valor numérico.

Passo 3. Converta cada sucessivo p1 p2 de letras de texto comum em um vetor-coluna p, e

forme o produto A.p. Nós chamamos p de vetor comum e Ap o correspondente vetor

cifrado.

CAPÍTULO 13

CIFRA DE HILL

183

Passo 4. Converta cada vetor cifrado em seu equivalente alfabético.

Em todas as operações converter os valores para congruência módulo 26.

Exemplo 13.1: Cifra de Hill de uma Mensagem

Use a matriz A= 1 2

0 3 para obter a cifra de Hill da mensagem de texto comum

PARAFUSO Solução.

Se nós agruparmos o texto comum em pares de letras, obteremos

ou, equivalentemente, usando a Tabela 1,

Para codificar o par PA nós efetuamos o produto matricial

que fornece o texto cifrado RC pela Tabela 1.

Para codificar o par RA nós efetuamos o produto matricial

que fornece o texto cifrado TC.

Os cálculos para os demais vetores cifrados são

Estes vetores correspondem aos pares de textos cifrado VK e WS, respectivamente.

Coletando os pares, obtemos a mensagem cifrada completa

6

21 =

48

63=

22

11 (mod 26)

1 2

0 3

19

15 =

49

45 =

23

19 (mod 26)

1 2

0 3

1 2

0 3

18

1=

20

3 (mod 26)

1 2

0 3

16

1=

18

3 (mod 26)

16 -1 18 - 1 6 -21 19 -15

PA RA FU SO

CAPÍTULO 13

CIFRA DE HILL

184

RCTCVKWS

Como o texto comum foi agrupado em pares e criptografado por uma matriz 2 x 2, dizemos

que a cifra de Hill do Exemplo 1 é uma 2-cifra de Hill. Evidentemente também é possível

agrupar o texto comum em ternos e criptografar com uma matriz 3 x 3 com entradas inteiras;

isto é chamado uma 3-cifra de Hill. Em geral, para uma n-cifra de Hill agrupamos o texto

comum em conjunto de n letras e codificamos com uma matriz codificadora n x n de

entradas inteiras.

Decifrando

Cada cifra útil deve possuir um procedimento para decifrar. Para decifrar as cifras de Hill,

usamos a inversa (mod m) da matriz codificadora. Para ser preciso, se m é um inteiro positivo,

dizemos que uma matriz A com entradas em Zm é inversível módulo m se existir uma matriz

B com entradas em Zm tal que

Suponha agora que

é inversível módulo m e que esta matriz é usada para uma 2-cifra de Hill. Se

é um vetor comum, então

é o correspondente vetor cifrado e

Assim, cada vetor comum pode ser recuperado do correspondente vetor cifrado pela

multiplicação à esquerda por A-1

(mod m).

Em criptografia é importante saber quais matrizes são inversíveis módulo m e como obter

suas inversas. Em seguida investigaremos estas questões.

Em aritmética comum, uma matriz quadrada A é inversível se, e somente se, det (A) 0 ou,

equivalentemente, det (A) tem um inverso. O teorema seguinte é o análogo deste resultado em

aritmética modular.

p = A-1

c

c = A p

p = 1

2

p

p

A = 11 12

21 22

a a

a a

A . B = B . A = I (mod m)

CAPÍTULO 13

CIFRA DE HILL

185

Teorema 13.1 Uma matriz quadrada A em Zm é inversível módulo m se, e somente se, o

detA( mód m) tem um inverso módulo m.

Desse modo, em particular, temos que, se

tem entradas em Zm se o det(A) = ad – bc (mod m) for relativamente primo com m, então a

inversa de det(A) (mod m) é dada por

Onde (ad – bc)-1

(mod m) é o inverso de ad – bc (mod m).

Exemplo 13.2. Decifrando uma 2-Cifra de Hill

Decifre a seguinte 2-cifra de Hill, que foi dada no Exemplo 1: RCTCVKWS

Solução:

Pela Tabela 1, o equivalente numérico do texto cifrado é

para obter os pares de texto comum, multiplicamos cada vetor cifrado pela inversa de A:

temos que o inverso de 3 módulo 26 é igual 9, ou seja 3.9 1(mod 26).

Assim, por ( I ),

A-1

= 9 3 2

0 1=

27 18

0 9=

1 8

0 9(mod 26)

Logo

1 8

0 9

18

3=

42

27=

16

1(mod 26)

1 8

0 9

20

3=

44

27=

18

1 (mod 26)

det(A) = 1.3 – 2.0 = 3

18-3 20-3 22-11 23-19

A-1

= (ad-bc)-1

d b

c a (mod m) ( I )

A= a b

c d

CAPÍTULO 13

CIFRA DE HILL

186

1 8

0 9

22

11=

110

99=

6

21(mod 26)

1 8

0 9

23

19=

175

171=

19

15(mod 26)

Pela Tabela 1, pode-se ver que os equivalentes alfabéticos destes vetores são

PARAFUSO

Matrizes Inversas Módulo m

Teorema: Se M é uma matriz quadrada de ordem n e det M 0, então a inversa de M módulo

m é

onde M é a matriz adjunta de M.

Exemplo 13.4:

Utilizando a matriz

9 0 5

8 7 6

3 2 0

A (mod 29) como chave, codifique e decodifique a mensagem

MATEMÁTICA em 29 , tal que

A B C D E F G H I J K L M

1 2 3 4 5 6 7 8 9 10 11 12 13

N O P Q R S T U V W X Y Z

14 15 16 17 18 19 20 21 22 23 24 25 26

Á É [ ]

27 28 0

Solução: Convertendo a mensagem:

Para codificarmos a mensagem, basta multiplicar cada matriz 3x1 obtida pela matriz

codificadora A:

13 5 20 1

1 ; 13 ; 9 ; _ _ 0

20 27 3 0

MAT EMÁ TIC A

M-1

= (det M )-1

. M (mod m)

CAPÍTULO 13

CIFRA DE HILL

187

9 0 5 13 217 14

8 7 6 1 231 28 (mod 29)

3 2 0 20 41 12

9 0 5 5 180 6

8 7 6 13 293 3 (mod 29)

3 2 0 27 41 12

9 0 5 20 195 21

8 7 6 9 241 9 (mod 29)

3 2 0 3 78 20

9 0 5 1 9

8 7 6 0 = 8 (mod 29)

3 2 0 0 3

A mensagem codificada é: NÉLFCLUITIHC

DECODIFICAÇÃO:

Para decodificar precisamos da matriz inversa de A.

Cálculo da matriz inversa 1A (mod 29):

2- Det (A) = -133, ou seja, Det (A) = 12 ( mod 29). Temos que 1 1

(mod 29) 17( ) 12Det A

.

De fato: (29) 112 17(mod29) 2712 1(mod29) , pois

3 9 3 27 312 17 12 17 12 12 12 17(mod29)

3- Matriz dos Cofatores:

12 18 -5

10 -15 -18

35 -14 63

C

17 18 24

10 14 11 (mod 29)

23 15 5

4- A Matriz Adjunta ( M ) é a transposta da Matriz dos Cofatores: 17 10 23

18 14 15 (mod 29)

24 11 5

tM C

5- Matriz Inversa 1A = 1

.( )

MDet A

(mod 29), temos:

1

17 10 23 289 170 391 28 25 14

17. 18 14 15 306 238 255 16 6 23 (mod 29)

24 11 5 408 187 85 2 13 27

A

Basta, então, multiplicar a mensagem codificada pela matriz inversa para obtermos a

mensagem original.

CAPÍTULO 13

CIFRA DE HILL

188

Exercícios:

Z32

A B C D E F G H I J K L M

1 2 3 4 5 6 7 8 9 10 11 12 13

N O P Q R S T U V W X Y Z

14 15 16 17 18 19 20 21 22 23 24 25 26

Ê ? Á Ã É [ ]

27 28 29 30 31 0

Tabela 1

1. Utilizando a tabela 1 e as matrizes dadas como chave, codifique e decodifique as

mensagens correspondentes, pelo método de Hill

a) 1 3

, O PIOR CEGO É AQUELE QUE NÃO ENXERGA O QUE VÊ2 7

A

b) 9 15, LEÃO AZUL

19 2A

c) 7 8 1

12 23 14 , MATEMÁTICA É LEGAL

22 4 21

B

d) 1 2 7

0 3 1 ,

0 5 2

B CATACLISMÁTICO

Para as questões de 2 a 8, utilize a seguinte tabela de conversão:

A B C D E F G H I J K L M

1 2 3 4 5 6 7 8 9 10 11 12 13

N O P Q R S T U V W X Y Z

14 15 16 17 18 19 20 21 22 23 24 25 0 Tabela de conversão de caracteres módulo 26.

2. Codifique a palavra AFRODITE utilizando a matriz 10 11

9 12

3. Codifique a palavra ELETRICIDADE utilizando a matriz

14 13 11

15 17 18

19 16 12

CAPÍTULO 13

CIFRA DE HILL

189

4. Decodifique a palavra KJQCDYVU sabendo que a matriz 20 21

19 22 foi utilizada na

cifragem.

5. Decodifique a palavra GEIOHSSFGQUD sabendo que a matriz

2 6 3

1 7 4

8 9 5

foi utilizada

na cifragem.

6. Decodifique a mensagem ODNINDTYTXPWVZPQ sabendo que a matriz

2 3 1 4

11 13 5 12

16 9 14 15

8 7 6 10

foi utilizada na cifragem.

7. Dada a mensagem em texto simples MITO, e o seu respectivo texto cifrado UQNI,

encontre a matriz 2 x 2 que foi utilizada na cifragem.

8. Dada a mensagem em texto simples ALGORITMO e o seu respectivo texto cifrado

ODVMFFYWS. Encontre a matriz 3 x 3 que foi utilizada na cifragem, sabendo que todos

os elementos desta matriz são inteiros positivos distintos menores ou iguais a 9.

Para as questões de 9 e 10, utilize a seguinte tabela de conversão:

A B C D E F G H I J K L

1 2 3 4 5 6 7 8 9 10 11 12

M N O P Q R S T U V W X

13 14 15 16 17 18 19 20 21 22 23 24

Y Z Á É Í Ó Ú Â Ô Ã Õ Ç

25 26 27 28 29 30 31 32 33 34 35 36 *O espaço em branco é representado pelo valor 0 (zero).

Tabela de conversão extendida de caracteres módulo 37.

9. Codifique a palavra EQUAÇÕES utilizando a matriz 31 33

29 35

10. Decodifique a palavra PBGEZJÃÁÓÍMJ sabendo que a matriz

26 28 29

21 25 27

22 23 24

foi

utilizada na cifragem.

190

Capítulo 14

RSA

Três americanos desenvolveram um sistema de código secreto, chamado RSA, baseado nas

dificuldades existentes para descobrir os fatores primos de um número muito grande. Criava-

se um novo ramo da Criptografia, a ciência dos códigos, fortemente baseado na Teoria dos

Números. Com o advento dos computadores e da computação algébrica, a Criptografia

ganhou um novo impulso. Neste momento, a proliferação de senhas bancárias e de cartões de

crédito, bem como a crescente necessidade de criptografar dados confidenciais que inundam a

Internet, fazem da Criptografia um dos ramos mais pesquisados da Matemática.

O sistema RSA, batizado em homenagem a seus inventores Ronald Rivest, Adi Shamir e

Leonard Adleman, foi o primeiro criptossistema de chave pública e ainda é o mais importante.

Sua segurança está intimamente relacionada à dificuldade de encontrar a fatoração de um

número inteiro positivo composto, que é o produto de dois primos gigantes.

14. 1. PRÉ-CODIFICAÇÃO

Em primeiro lugar, devemos converter a mensagem em uma sequência de números. Essa

primeira etapa é chamada de pré-codificação. Há várias maneiras de se fazer isso. Aqui

vamos supor que o texto não contém acentuação, pontuação, números etc, apenas as letras A a

Z (maiúsculas). Também vamos adicionar espaços em branco entre palavras, que será

substituído pelo número 99. A letra A será convertida no número 10, B será 11 e assim por

diante, até o Z correspondendo ao número 35. Observe que cada letra corresponde a um

número com exatamente dois algarismos. Isso evita ambigüidades.

A B C D E F G H I J K L M

1 0 11 12 13 14 15 16 17 18 19 20 21 22

N O P Q R S T U V W X Y Z

23 24 25 26 27 28 29 30 31 32 33 34 35

[ ]

99

A chave pública é um número n = p . q, onde p e q são primos. Antes de começar devemos,

então escolher esses números. O último passo da pré-codificação é quebrar a mensagem em

blocos. Esses blocos devem ser números menores que n. A maneira de escolher os blocos não

é única, mas é importante evitar duas situações:

Nenhum bloco deve começar com o número 0 (problemas na decodificação).

Os blocos não devem corresponder a nenhuma unidade lingüística (palavra, letra, etc).

Assim a decodificação por contagem de freqüência fica impossível.

CAPÍTULO 14

RSA

191

14.2 – Codificando e decodificando

Para codificar a mensagem precisamos de n = p . q e de um inteiro positivo e (1 < e < (n))

que seja inversível módulo (n). Em outras palavras,

Note que e é sempre ímpar, dado que p – 1 é par.

Chamaremos o par (n, e) de chave de codificação do sistema RSA.

Codificaremos cada bloco de mensagem separadamente e a mensagem codificada será a

seqüência de blocos codificados.

Vamos agora mostrar como codificar cada bloco b. Chamaremos o bloco codificado de C(b).

Em primeiro lugar, lembre-se que b é menor que n. Então:

C(b) be (mod n)

onde 0 C(b) < n.

Exemplo 14.1: Considere a frase Paraty é linda. Convertendo em números,

Agora devemos escolher n.

Vamos começar com um número pequeno, por exemplo, n = 11.13 = 143.

Podemos então quebrar a mensagem acima em blocos, que devem ter valor menor que 143 e

não devem iniciar com zero.:

Então temos que (143) = 10.12 = 120 e portanto vamos escolher e como o menor primo que

não divide 120. O valor é 7. Logo,

Procedendo dessa maneira com todos os blocos, obtemos a seguinte mensagem cifrada:

Vejamos agora como proceder para decodificar um bloco de mensagem codificada.

64 – 119 – 6 – 119 – 102 – 36 – 130 – 36 – 27 – 79 – 23 – 117 – 10

C(25) 257 (mod 143)

C(25) 254 . 25

2 . 25

1 (mod 143)

C(25) 254 . 53 . 25 (mod 143)

C(25) 532 . 53 . 25 (mod 143)

C(25) 92 . 53 . 25 (mod 143)

C(25) 14 . 25 (mod 143)

C(25) 64 (mod 143)

25 – 102 – 7 – 102 – 93 – 49 – 91 – 49 – 92 – 118 – 23 – 13 – 10

25 10 27 10 29 34 99 14 99 21 18 23 13 10

m.d.c.(e, (n)) = m.d.c. (e, (p – 1).(q – 1)) = 1

CAPÍTULO 14

RSA

192

A informação que precisamos para decodificar está contida no par (n, d), onde d (1<d< (n)) é

o inverso de e módulo (n) (e.d 1 (mod (n)). Chamaremos (n, d) de chave de

decodificação e D(c) o resultado do processo de decodificação. D(c) é dado por:

onde 0 D(c) < n.

Observe que é possível calcular d, desde que (n) e e sejam conhecidos. Entretanto, se não

conhecemos p e q é praticamente impossível calcular d. Voltando ao nosso exemplo, temos

que n = 143 e e = 7. Para calcular d, usamos o Teorema de Euler:

ou seja,

Como exemplo, deciframos o primeiro bloco da mensagem cifrada:

Os outros blocos ficam como exercícios.

A pergunta óbvia que surge agora é: D(C(b)) = b?

Ou seja, decodificando um bloco de mensagem codificada, encontramos um bloco da

mensagem original? Porque senão todo nosso esforço foi sem sentido. Vamos mostrar nessa

seção que a resposta para esta pergunta é: sim!

Consideremos então n = p . q. Vamos provar que D(C(b)) b (mod n)

D(64) 64103

(mod 143)

D(64) 64100

. 643 (mod 143)

D(64) (6410

)10

. 643 (mod 143)

D(64) ((645)2)10

. 643 (mod 143)

D(64) ((642 . 64

3)2)10

. 643 (mod 143)

D(64) ((92 . 25)2)10

. 25 (mod 143)

D(64) ((12)2)10

. 25 (mod 143)

D(64) (144)10

. 25 (mod 143)

D(64) (1)10

. 25 (mod 143)

D(64) 25 (mod 143)

d 7( (143)) – 1

(mod (143))

d 7(120) – 1

(mod 120)

d 732 – 1

(mod 120)

d 731

(mod 120)

d 103 (mod 120)

d.e 1 (mod (n)) d e( (n)) – 1

(mod (n))

D(c) cd (mod n)

CAPÍTULO 14

RSA

193

E por que não a igualdade D(C(b)) = b (mod n)? Observe que D(C(b)) e b são menores que (n

– 1). Por isso escolhemos b menor que n e mantivemos os blocos separados depois da

codificação. Pela definição de D e C temos que

Mas d é o inverso de e módulo (n). Logo, existe inteiro k tal que e.d = 1 + k. (n). Logo,

Se m.d.c.(b, n) = 1, então podemos usar o teorema de Euler:

Se b e n não são primos entre si, observe que n = p . q, p e q primos distintos. Logo,

Se m.d.c.(b, p) = 1, então podemos usar o teorema de Fermat [bp-1

1 (mod p)].

Caso contrário, temos que p | b e portanto

Logo,

qualquer que seja b. Fazemos o mesmo para o primo q, obtendo:

Portanto,

c.q.d.

Exemplo 14.2:

A B C D E F G H I J K L M

1 0 11 12 13 14 15 16 17 18 19 20 21 22

N O P Q R S T U V W X Y Z

23 24 25 26 27 28 29 30 31 32 33 34 35

[ ]

99

Digamos que você está interessado em enviar a mensagem

bed

b (mod p.q)

bed

b (mod n)

bed

b (mod q)

bed

b (mod p)

bed

b 0 (mod p)

bed

b1 + k. (n)

b1 + k.(p-1).(q-1)

(b(p-1)

)k.(q-1)

. b (mod n)

bed

(1)k . b b (mod n)

bed

b1 + k. ( n )

b . (b (n)

)k (mod n)

D(C(b)) (be)d b

ed (mod n)

CAPÍTULO 14

RSA

194

Você deve inicialmente numerar toda a palavra de acordo com a tabela acima:

Agora, suponha que a chave seja (n, e) = (7663, 17)

Então você deve escolher blocos numéricos para codificar que sejam menores que n e que não

comecem com zero.

Escolhamos, por exemplo, os seguintes blocos (veja que isso vai depender da máquina que

você está trabalhando):

Basta agora pegar cada bloco e elevar à potência e módulo n:

A mensagem codificada é

Agora vejamos como fazer se recebemos uma mensagem codificada por alguém utilizando a

chave pública (n, e) = (7663, 17). Suponha que a mensagem recebida seja a mesma obtida

acima:

Para entender o que significa você precisa da chave privada d, que é inversa de e (mod ( n )).

Etapas:

1. Primeiro, fatoramos o número n = 7623 ( veja abaixo a fatoração pelo método de Fermat).

Encontramos os primos 79 e 97.

2. Calculamos (n) = (7663) = (79) . (97) = 78.96 = 7488.

5137 – 3345 – 1635 – 3009 – 2186 – 3890 – 2032

5137 – 3345 – 1635 – 3009 – 2186 – 3890 – 2032

15117

5137 (mod 7663)

15117

5137 (mod 7663)

82317

3345 (mod 7663)

10217

1635 (mod 7663)

12217

3009 (mod 7663)

14217

2186 (mod 7663)

32917

3890 (mod 7663)

1417

2032 (mod 7663)

151 – 823 – 102 – 122 – 142 – 329 – 14

15 18 23 10 21 22 14 23 29 14

FINALMENTE

CAPÍTULO 14

RSA

195

3. Calculamos d e( (n)) – 1

(mod (n)) 17(7488) – 1

(mod 7488) 881 (mod 7488).

4. Agora basta pegar cada bloco que recebemos e elevar à potência d módulo n.

5. Então colocamos os resultados alinhados

depois separamos em blocos de dois números

que corresponde a mensagem enviada

14. 3. Assinatura digital utilizando a criptografia RSA

1º passo: Como a segurança do RSA reside na dificuldade de fatorar números que são

produtos de 2 primos; então é aconselhável escolher primos bem grandes (10 ou mais dígitos).

Mas, para fins didáticos vamos escolher primos relativamente pequenos (menores que 1000),

cuja idéia é a mesma para primos grandes.

Ainda, para efeitos deste exemplo, chamaremos o remetente de Alice (A) e o destinatário de

Bob (B). Para elaborarmos as chaves públicas e privadas do esquema, então escolhemos

adequadamente dois primos p e q para cada participante, e calculamos o produto n = p . q.

Os números primos de Alice são: pA = 859, qA = 547. Que resulta em nA = 469873.

Os números primos de Bob são: pB = 937, qB = 461. Que resulta em nB = 431957. *Observação 1: os primos p e q não devem ser muito próximos para evitar fatorações rápidas como a de Fermat.

2º passo: O número n encontrado desta maneira faz parte tanto da chave pública como da

chave privada de cada participante. Agora, escolhemos um número e que também será público

e que seja relativamente primo à função de Euler (n) MDC( (n), e) = 1.

FINALMENTE

15 18 23 10 21 22 14 23 29 14

151 – 823 – 102 – 122 – 142 – 329 – 14

5137881

151 (mod 7663)

3345881

823 (mod 7663)

1635881

102 (mod 7663)

3009881

122 (mod 7663)

2186881

142 (mod 7663)

3890881

329 (mod 7663)

2032881

14 (mod 7663)

CAPÍTULO 14

RSA

196

Para Alice, (nA) = (859) . (547) = 858 . 546 = 468468.

Uma boa escolha para Alice é o número primo eA = 5, pois o MDC(468468, 5) = 1.

Para Bob, (nB) = (937) . (461) = 936 . 460 = 430560.

Uma boa escolha para Bob é o número primo eB = 7, pois o MDC(430560, 7) = 1. *Observação 2: a chave pública e não é necessariamente um número primo, este exemplo foi apenas uma mera coincidência.

3º passo: Tendo posse de (n) e da chave pública e, podemos calcular a chave secreta d.

Sendo que d é o inverso multiplicativo de e, ou seja, e.d 1 (mod (n)). Pelo teorema de

Euler, ( ( )) 1(mod ( ))nd e n . Fazendo os devidos cálculos encontramos que:

Para Alice, 5. dA 1 (mod 468468) dA = 281081.

Para Bob, 7. dB 1 (mod 430560) dB = 307543. *Observação 3: uma forma computacional rápida para se encontrar a chave privada d é:

int d = 0; for ( d = 2; d < phi; d++ ) {if ( (e * d) % phi == 1 ) ) {System.out.println( “Inverso = ” + d ); break; // d encontrado, sai do laço}}

Pronto, já temos posse das chaves necessárias para cifrar e decifrar mensagens.

O que deve ser disponibilizado nas páginas amarelas ou qualquer diretório público:

Chave pública de Alice (nA, eA) = (469873, 5).

Chave pública de Bob (nB, eB) = (431957, 7).

O que não deve ser disponibilizado, pois é secreto e pertence somente aos seus donos:

Chave privada de Alice (nA, dA) = (469873, 281081).

Chave privada de Bob (nB, dB) = (431957, 307543). *Observação 4: note que tanto os primos p e q, assim como a função (n), também devem ser mantidos secretos!

Pois a partir de p e q, pode-se facilmente calcular (n) = (p – 1) * (q – 1). E vice-versa, a partir de (n) pode-se calcular p e q através de um

sistema linear e uma simples equação do 2º grau. Tendo posse de (n), calcula-se a chave privada d como demonstrado.

4º passo: Finalmente, como fazer para criptografar uma mensagem? Por exemplo, tomemos a

palavra “BRASIL”, que no código ASCII corresponde aos valores:

Concatenando esses valores, obtemos 668265837376.

Reagrupando em blocos (de qualquer tamanho), obtemos 668, 265, 837, 376.

O valor contido em cada bloco não deve exceder o valor da chave pública n. Também se deve

tomar cuidado para que nenhum bloco inicie com o valor zero, com risco de se criar um erro

na cifragem. Os blocos evitam ataques por freqüência, já que se a mensagem fosse codificada

caractere por caractere, o RSA se tornaria uma mera cifra de substituição.

A codificação dos blocos é feita, elevando-se cada valor do bloco ao expoente público e

módulo n do destinatário, obtendo-se o menor valor positivo correspondente. Assim, se Alice

deseja enviar a palavra “BRASIL” para Bob, ela deve utilizar a chave pública (nB, eB) =

(431957, 7) de Bob, e efetuar C(b) = be (mod n), da seguinte forma:

(66, 82, 65, 83, 73, 76)

CAPÍTULO 14

RSA

197

C(668) = 6687 (mod 431957) 331318 (mod 431957)

C(265) = 2657 (mod 431957) 387742 (mod 431957)

C(837) = 8377 (mod 431957) 213180 (mod 431957)

C(376) = 3767 (mod 431957) 121902 (mod 431957)

Alice, então, deve enviar a mensagem criptografada

para Bob.

5º passo: Para decifrar a mensagem, Bob deve proceder de maneira inversa, utilizando a sua

chave secreta (nB, dB) = (431957, 307543), ou seja, D(c) = cd (mod n), da seguinte forma

(geralmente não é uma congruência fácil de resolver...):

D(331318) = 331318307543

(mod 431957) 668 (mod 431957)

D(387742) = 387742307543

(mod 431957) 265 (mod 431957)

D(213180) = 213180307543

(mod 431957) 837 (mod 431957)

D(121902) = 121902307543

(mod 431957) 376 (mod 431957)

Bob reagrupa a mensagem decodificada 668265837376, separa em pares,

e obtém a mensagem original “BRASIL”.

Propriedades matemáticas garantem que aplicando a função de cifragem e depois decifragem,

a mensagem original continua a mesma, isto é, b = D(C(b)).

Assinatura: Assinar uma mensagem é garantir que ela realmente vem de quem diz ser o

remetente.

Como podemos assinar uma mensagem utilizando o RSA?

No exemplo, anterior, perceba que em nenhum momento chegamos a usar as chaves do

remetente (Alice), portanto é chegada a hora.

Devido às propriedades matemáticas das chaves públicas e privadas e e d, Alice pode utilizar

sua própria chave privada dA “decodificar” a mensagem original ao invés de normalmente

utilizá-la para decifrar uma mensagem cifrada com sua chave pública destinada a ela.

Isto faria com que a mensagem ficasse cifrada com a sua chave privada, e como isto seria

útil? Antes de partimos para o exemplo numérico, uma breve teoria.

O que foi feito anteriormente (no exemplo de Bob):

b CB(b) DB(CB(b)) b

(66, 82, 65, 83, 73, 76)

(331318, 387742, 213180, 121902)

CAPÍTULO 14

RSA

198

onde b é a mensagem em texto puro “BRASIL”, CB( ) é a função de cifragem que utiliza a

chave pública de Bob, e DB( ) é a função de decifragem que utiliza a chave secreta de Bob.

O esquema de assinatura digital consiste em

Em outras palavras, Alice utiliza sua chave privada para “decodificar” a mensagem original

(que inicialmente não é uma ação muito útil, pois qualquer pessoa com conhecimento da

chave pública de Alice pode desfazer esta ação). Em seguida, Alice cifra o resultado com a

chave pública de Bob. Neste ponto, o que temos? Uma mensagem-cápsula duplamente

criptografada com a chave privada de Alice e a chave pública de Bob.

O que isto garante? Que Bob pode primeiramente decodificar esta mensagem-cápsula com

sua própria chave privada, e assim obter de volta a mensagem criptografada com a chave

privada de Alice. Mas aqui que surge o grande ponto: se Bob utilizar a chave pública de

Alice, ele obterá a mensagem original!

Como ninguém mais além de Alice possui a chave privada dA, então realmente aquela

mensagem foi codificada inicialmente por Alice. A outra garantia vem de que somente Bob

sabe disto, pois Alice sabiamente também utilizou a chave pública de Bob.

Em resumo, no esquema tradicional (sem assinatura), b = D(C(b)).

Entretanto, devido às propriedades matemáticas, também é verdade que b = C(D(b)).

Ou seja, no esquema assinado temos, b = CA(DB(CB(DA(b)))), onde as funções são aplicadas

de dentro para fora na ordem que aparecem. Agora o exemplo numérico.

Relembrando as chaves públicas de Alice e Bob:

Chave pública de Alice (nA, eA) = (469873, 5).

Chave pública de Bob (nB, eB) = (431957, 7).

Chave privada de Alice (nA, dA) = (469873, 281081).

Chave privada de Bob (nB, dB) = (431957, 307543).

1º passo: Alice utiliza sua chave privada para “decodificar” a palavra “BRASIL” separada em

blocos como no exemplo anterior (as mesmas observações de tamanho de bloco e valores

menores que os módulos também valem para este caso):

DA(668) = 668281081

(mod 469873) 249014 (mod 469873)

DA(265) = 265281081

(mod 469873) 82081 (mod 469873)

DA(837) = 837281081

(mod 469873) 221564 (mod 469873)

DA(376) = 376281081

(mod 469873) 231544 (mod 469873)

CB(DA(b)) = mensagem assinada

b DA(b) CB(DA(b)) DB(CB(DA(b))) DA(b) CA(DA(b)) b

CAPÍTULO 14

RSA

199

2º passo: Em seguida, Alice pega o resultado e aplica na função de codificação com a chave

pública de Bob:

CB(249014) = 2490147 (mod 431957) 273379 (mod 431957)

CB(82081) = 820817 (mod 431957) 45868 (mod 431957)

CB(221564) = 2215647 (mod 431957) 414208 (mod 431957)

CB(231544) = 2315447 (mod 431957) 363416 (mod 431957)

Assim, Alice envia a mensagem criptografada assinada:

3º passo: Bob, ao receber a mensagem criptografada assinada, deve primeiramente aplicar sua

chave privada (é uma calculeira danada, eu sei... Tente usar o computador!):

DB(273379) = 273379307543

(mod 431957) 249014 (mod 431957)

DB(45868) = 45868307543

(mod 431957) 82081 (mod 431957)

DB(414208) = 414208307543

(mod 431957) 221564 (mod 431957)

DB(363416) = 363416307543

(mod 431957) 231544 (mod 431957)

Já aqui neste resultado, Bob deve obter algo que Alice obteve no 1º passo.

4º passo: Finalmente, Bob aplica a chave pública de Alice que garante que foi ela mesma que

enviou esta mensagem (nenhuma outra pessoa além de Alice pode ter a chave privada dela,

lembra?):

CA(249014) = 2490145 (mod 469873) 668 (mod 469873)

CA(82081) = 820815 (mod 469873) 265 (mod 469873)

CA(221564) = 2215645 (mod 469873) 837 (mod 469873)

CA(231544) = 2315445 (mod 469873) 376 (mod 469873)

Novamente, Bob reagrupa a mensagem decodificada 668265837376, separa em pares, (66,

82, 65, 83, 73, 76) e obtém a mensagem original “BRASIL”.

(273379, 45868, 414208, 363416)

CAPÍTULO 14

RSA

200

Exercícios

Para os exercícios seguintes utilize a tabela abaixo ou a tabela ASCII

A B C D E F G H I J K L M

1 0 11 12 13 14 15 16 17 18 19 20 21 22

N O P Q R S T U V W X Y Z

23 24 25 26 27 28 29 30 31 32 33 34 35

[ ]

99

1) Usando as chaves públicas dadas, codifique e decodifique as frases correspondentes:

a) (n, e) = (1037, 7) SONHO

b) (n, e) = (2201, 11) REMO

c) (n, e) = (1577, 5) ESPERANDO

2) A chave pública utilizada pelo Banco Crash para codificar suas mensagens é a seguinte:

(n, e) = (4559, 5)

Os computadores do banco receberam, de local indeterminado, os seguintes blocos de

mensagem: 2621 – 2608 – 3594 – 4261

O que diz a mensagem?

3) A mensagem

96 – 61 – 751 – 9

foi codificada pelo método RSA usando a chave (n, e) = (767, 13). Decodifique a

mensagem.

4) Sabendo-se que n = 27641 é igual ao produto de dois primos e que ( ) 27300n ,

determine os fatores primos de n.

5) Sabendo-se que n = 3552377 é igual ao produto de dois primos e que ( ) 3548580n ,

determine os fatores primos de n.

6) A mensagem

6802 – 8728 – 9451

foi codificada pelo método RSA usando a chave (n, e) = (16517, 5). Além disso, sabe-se

que ( ) 16236n . Decodifique a mensagem.

7) (Assinatura Digital)

Considere que o Banco Bandit possua a seguinte chave pública:

(nB, eB) = (20099, 7)

E a Empresa Explorit possua a seguinte chave pública:

(nE , eE) = (61823, 11)

Demonstre como a empresa enviaria a mensagem MONEY assinada ao banco e como o

banco confirmaria a autenticidade da assinatura.

201

Capítulo 15

PARTILHA DE SENHAS

O Teorema Chinês do Resto é utilizado em sistemas de partilha de senhas entre várias

pessoas, de modo que para cada pessoa seja dado um elemento distinto em função da senha s

a ser compartilhada. A grande vantagem do método de partilha de senhas é que cada pessoa

contém uma chave diferente das outras chaves e uma pessoa sozinha não consegue decifrar a

senha. Outra vantagem é que não é necessária a presença de todas as pessoas com suas

respectivas chaves, bastam que k ou mais pessoas estejam presentes. A chave para cada

pessoa é escolhida dentro um conjunto S composto por n pares de inteiros positivos de forma

que, para cada inteiro positivo k n previamente escolhido, tem-se que:

Qualquer subconjunto de S com k elementos permite determinar s facilmente;

É muito difícil determinar s conhecendo menos que k elementos de S.

O primeiro passo é escolher um conjunto apropriado L de n inteiros positivos distintos, dois a

dois primos entre si. Seja N o produto dos k menores números de L, e M o produto dos k – 1

maiores números de L, os elementos de L devem ser escolhidos com cuidado de forma que M

< N, e a senha s possa ser escolhida arbitrariamente dentro do intervalo M < s < N.

Os elementos até aqui mencionados são:

L = {p1, p2, ..., pn}; onde p1 < p2 < ... < pn são primos distintos entre si.

k = número de pessoas que se deseja estarem presentes para a decifragem da senha.

N = p1.p2. ... .pk = produto dos k menores elementos de L.

M = px.px+1. ... . pn = produto dos k – 1 maiores elementos de L.

s = senha aleatória que não tem relação alguma com L, mas deve ser escolhida de forma que

M < s < N.

Posteriormente, o conjunto gerador de chaves S será constituído pelos pares da forma (p, sp)

onde p L e sp é a forma reduzida de s (mod p). Um limite k 2 implica em s > p para

qualquer p L. Logo sp < s para qualquer p L.

Supõe-se que sejam conhecidos, em um dado momento, t k pares de elementos de S, ou

seja, existem t pessoas presentes para a decifragem da senha s. Denota-se esses pares por (p1,

s1), (p2, s2), ..., (pt, st).

Para se chegar à senha s é necessário resolver o seguinte sistema de congruências:

Pelo Teorema Chinês do Resto, obtém-se x0 como solução, tal que:

x0 s (mod p1.p2. ... .pt)

x s1 (mod p1)

x s2 (mod p2)

...

x st (mod pt)

S = {(p1, sp1), (p2, sp2), ..., (pt, spt)}

CAPÍTULO 15

PARTILHA DE SENHAS

202

É sabido que como t k, (p1.p2. ... .pt) N > k, e o sistema de congruências tem uma única

solução menor que (p1.p2. ... .pt).

Observações:

É possível escolher os módulos de s de modo que seja impraticável

encontrar s através de uma busca, conhecendo-se apenas uma das chaves;

É sempre possível escolher um conjunto L que satisfaça todas as condições.

A seguir, um exemplo prático para efeito de esclarecimento.

Exemplo 15.1:

No banco “Golden Luck” há 5 funcionários responsáveis pela manutenção da senha de um

cofre, e pelo menos 2 pessoas (k = 2) têm que estar presentes para a abertura do mesmo.

Logo, o conjunto L deve possuir 5 elementos e seu limiar deve igual a 2. Uma possível

escolha para L envolvendo somente primos pequenos é

A partir do qual se calcula os valores dos limites N e M:

N = 143 = 11.13 = produto dos (k = 2) menores elementos de L.

M = 23 = produto dos (k – 1 = 2-1=1) maiores elementos de L.

O valor da senha s pode ser escolhido aleatoriamente como qualquer inteiro no intervalo que

vai de 23 a 143 (M < s < N). Por exemplo, suponha que a senha seja s = 50. Então o conjunto

S que contêm os elementos da senha é:

O segundo termo de cada elemento de S, sm, é o resto da divisão de s = 50 por cada termo

correspondente de L, ou seja, s(mod p). Se os funcionários que possuem as senhas (17, 16) e

(23, 4), por exemplo, estão no banco, para obter a senha seria preciso resolver o sistema:

Solução:

m = 17. 23 = 391

1

1

mM = = 23

m;

2

2

mM = = 17

m

Os inversos x1 e x2 de M1 e M2 são dados por:

23 x1 1 (mod 17) e 17x1 1 (mod 23)

x 16 (mod 17)

x 4 (mod 23)

S = {(11, 6), (13, 11), (17, 16), (19, 12), (23, 4)}

L = {11, 13, 17, 19, 23}

CAPÍTULO 15

PARTILHA DE SENHAS

203

Aplicando o método de Euler nas equações acima obtemos as soluções respectivas:

Portanto, temos

Assim, determina-se que 50 é o menor valor inteiro positivo congruente a x, que é a senha

correta.

Exercícios

1. Por motivo de segurança o banco “ Golden Luck” trocou a senha do cofre. Dois

funcionários possuem as chaves (13, 5) e (19, 17). Qual a nova senha?

2. Após uma nova troca de senhas no banco “Golden Luck”, 2 funcionários tem agora as

chaves (31, 6) e (41, 20). Qual é a senha agora?

3. Como duas pessoas só sabem guardar um segredo se uma delas já estiver morta, o banco

“Golden Luck” resolveu fazer uma mudança completa no esquema de segurança e

novamente a senha foi trocada, além disso ficou estabelecido que estejam presentes, no

mínimo, 3 pessoas para que o cofre possa ser aberto. Três funcionários estão com as

chaves (53, 21), (61, 9) e (71, 35). Qual é a senha?

x a1M1x1 + a2M2x2 (mod m)

x 16 . 23 . 3 + 4 . 17 . 19 (mod 391)

x 2396 (mod 391)

x = 50

x1 = 3 e x2 = 19 .